You are on page 1of 138

1

MỤC LỤC
Đề Lời giải
1 Trường THPT chuyên Hà Nội - Amsterdam 105
Practice Test 1 3
Practice Test 2 8
Practice Test 3 12
Practice Test 4 16
Practice Test 5 20
2 Trường THCS Ngoại Ngữ 112
Practice Test 1 24
Practice Test 2 27
Practice Test 3 30
Practice Test 4 33
Practice Test 5 36
3 Trường THCS chất lương cao 117
Practice Test 1 39
Practice Test 2 43
Practice Test 3 47
Practice Test 4 51
Practice Test 5 54
Practice Test 6 57
Practice Test 7 61
Practice Test 8 65
Practice Test 9 68
Practice Test 10 71
4 Trường THCS Lương Thế Vinh 128
Practice Test 1 73
Practice Test 2 76
Practice Test 3 78
Practice Test 4 81
Practice Test 5 84
Practice Test 6 87
Practice Test 7 91
Practice Test 8 94
Practice Test 9 97
Practice Test 10 101

2
ĐỀ THI THỬ
TRƯỜNG THPT CHUYÊN
HÀ NỘI - AMSTERDAM
PRACTICE TEST 1
I. PHONETICS
1.1. Choose the word whose underlined part is differently pronounced from the others.
1. A. skating B. status C. stadium D. statue
2. A. definitions B. documents C. combs D. doors
3. A. worked B. moved C. stopped D. brushed
1.2. Choose the word whose main stress is different from the others.
1. A. relax B. wonder C. problem D. special
2. A. popularity B. conscientious C. apprenticeship D. personality
II. READING COMPREHENSION
2.1. Read the following passage and mark the letter A, B, C, or D to indicate the correct answer to each
of the questions.
Why are there so many grandmothers and so few grandfathers? In other words, why do men die
younger than women? Is this because men are afraid of getting old and helpless and so they prefer to die
before that happens? Perhaps they fear to be left alone by their women and so decide to do the leaving first.
Many explanations are given for the fact that men die earlier than women. Men are stronger
physically, yet women can hang on longer to life. Both men and women are emotional creatures but women
are not afraid or ashamed to cry while men refuse to do so. They are afraid of being thought "soft". Some
men, when they are upset, play loud music or dig in the garden to relieve their feelings. Many men like an
orderly life so that they bury themselves in their work or want a wife with some children or long to take part
in wars where there is a definite chain of command. There is a leader to give orders and a known enemy to
fight against and defeat. All these make men feel they live in a rational world.
So the majority of men like a rational world. Rationality is fine but it does not include everything that
makes life joyful and fun or even messy and frustrating. When a man refuses to cry, he is refusing to accept
that his emotions are part of him. Of course, some men do not follow this pattern. Bob Hawke is capable of
crying in public whereas Margaret Thatcher is probably incapable of crying at any time.
We need to cry because that shows our ability to suffer. If we do not suffer, we are not really alive at
all. Suffering can be creative or destructive. If we can all learn to cry and laugh and shout and dance openly,
we are living creatively and adding something to the human race. We often say men suffer more from stress
and therefore they die earlier. Are we not saying in another way that they do not know how to suffer in the
right way - with tears and laughter instead of silence - and so they are miserable and just give up? That is just
my theory of course. Have you a better one?
1. What does the word “that” in the passage refer to?
A. The fact that men get old and helpless.
B. The fact that they fear to be left alone.
C. The fact that men die younger than women.
D. The fact that they decide to do the leaving first.
2. What's the main question the article is intended to discuss?
A. Why women are more emotional than men.
B. Why suffering is different in men and women.
C. Why men die earlier than women.
D. Why men like a rational world.
3. What is found the same for both men and women?
3
A. They like to cry. B. They are physically strong.
C. They are emotional creatures. D. They want to play loud music.
4. The phrase “to do the leaving” is closest in meaning to………….
A. set off B. go down C. pass away D. depart
5. What's the writer's opinion toward crying?
A. It is a good thing for people to cry.
B. It is all right for women to cry but shameful for men.
C. Tears are not real signs of suffering.
D. To be silent is better than to cry.
2.2. Read the text below and decide which answer A,B,C or D fits each space.
Earth is the only (1)…………..we know of in the universe that can support human life. (2)…………..
human activities are making the planet less fit to live on. As the western world (3) …………..on consuming
two-thirds of the world's resources while half of the world's population do so (4)…………..to stay alive, we
are rapidly destroying the very resource we have by which all people can survive and prosper. Everywhere
fertile soil is (5)…………..built on or washed into the sea. Renewable resources are exploited so much that
they will never be able to recover completely. We discharge pollutants into the atmosphere without any
thought of the consequences. As a (6)………….., the planet's ability to support people is being reduced at the
very time when rising human numbers and consumption are (7)…………..increasingly heavy demands on it.
The Earth's natural resources are there for us to use. We need food, water, air, energy, medicines, warmth,
shelter and minerals to (8)…………..us fed, comfortable, healthy and active. If we are sensible in how we
use the resources, they will (9)…………..indefinitely. But if we use them wastefully and excessively, they
will soon run (10)…………..and everyone will suffer.
1. A. situation B. place C. position D. site
2. A. Although B. Still C. Yet D. Despite
3. A. continues B. repeats C. carries D. follows
4. A. already B. just C. for D. entirely
5. A. sooner B. neither C. either D. rather
6. A. development B. result C. reaction D. product
7. A. doing B. having C. taking D. making
8. A. hold B. maintain C. stay D. keep
9. A. last B. stand C. go D. remain
10. A. out B. off C. over D. down
III. GRAMMAR AND VOCABULARY
3.1. Choose the best option to complete the sentences.
1. A new study group has been set…………….by the government.
A. out B. up C. away D. down
2. We would rather Helen…………….us all the information we needed. We should have been well informed.
A. sent B. send C. had sent D. have sent
3. Only because she had to support her family,…………….to leave school.
A. that Alice decides
B. so Alice decided
C. Alice decided
D. did Alice decide
4. The majority of primary school teachers…………….women.
A. is B. are C. includes D. including
5.…………….today, there would be nowhere for them to stay.
A. Were they to arrive B. If they arrive
C. Had they arrive D. Provided they arrived
4
6. You can use my phone if yours……………..
A. won't be worked B. won't work C. isn't worked D. doesn't work
7. Generally…………….., learning a foreign language is interesting, but not easy.
A. spoken B. speaking C. speak D. speaks
8. Ben would have studied medicine if he……………..to a medical school.
A. was admitted B. had been admitted
C. had admitted D. would be able to enter
9.……………..having a well-paid job, she never has any money.
A. Let alone B. Despite C. For D. Even though
10.……………..does it take you to do the washing?
A. How fast B. What time C. How often D. How long
3.2. Conversation
Mark the letter A, B, C, or D on your answer sheet to indicate the most suitable response to complete
each of the following exchanges.
1. Linda: “It's been a tough couple of months, but I think the worst is behind us now.”
Jill: “……………..”
A. Good morning B. Good luck! C. Good! D. Goodness me!
2. Linda: “What a new nice style of hair you have it cut!” “……………..”
A. No, I think it's suitable for me B. It's from Italy
C. Thank you D. Yes, please
3.3. Synonyms & Antonyms
(a) Mark the letter A, B, C, or D on your answer sheet to indicate the word(s) CLOSEST in meaning to
the underlined word(s) in each of the following questions.
1. In remote communities, it's important to replenish stocks before the winter sets in.
A. empty B. remake C. repeat D. refill
2. Nearly all weather occurs in the troposphere, the lowest layer of the earth's at-mosphere.
A. Closely to B. Barely C. Almost D. After
(b) Mark the letter A, B, C, or D on your answer sheet to indicate the word(s) OPPOSITE in meaning
to the underlined word(s) in each of the following questions.
1. They have not made any effort to integrate with the local community.
A. cooperate B. put together C. separate D. connect
2. There has been insufficient rainfall over the past two years, and farmers are having trouble.
A. adequate B. unsatisfactory C. abundant D. dominant
3.4. Give the correct form of the words in brackets.
1. Housework has……………..been regarded as women's work. (TRADITION)
2. We will live happier and……………..life if we keep our environment clean. (HEALTH)
3. It is……………..of you to cheat in the exam. (HONEST)
4.……………..is now a serious problem in many countries. (FOREST)
IV. WRITING
4.1. Rewrite each of the following sentences in such a way that it means the same as the sentence
printed before it. Use the bold words given in brackets. Do not alter the given words in any way.
1. It took us three hours to find a room for the night. (SPENT)
We…………………………………………………………………………………………………………
2. Would you rather I stayed with you during the holidays? (PREFER)
Would you …………………………………………………………………………………………………
3. The English test was not easy enough for me to do well. (SO)
The English test ……………………………………………………………………………………………
4. The police said Jim had stolen the money. (ACCUSED)
5
The police ………………………………………………………………………………………………….
5. Because of his illness, he could not work effectively. (IMPOSSIBLE)
His illness ………………………………………………………………………………………………….
4.2. Write meaningful sentences using the given words.
1. when/hot/he/go/swim/river/front/his house.
……………………………………………………………………………………………………………...
2. she/usually/listen/music/night.
……………………………………………………………………………………………………………...
3. this coffee/hot/that/I/not/drink it.
……………………………………………………………………………………………………………...
4. you/know/ who/best/English/your grade?
……………………………………………………………………………………………………………...
5. air pollution/serious problem/many/big city.
……………………………………………………………………………………………………………...
4.3. Write a paragraph (about 120 - 150 words) about what we should do to stay healthy.
The following suggestions may be useful:
- Having rational nutrition.
- Doing exercises and sports.
- Avoiding smoking and drinking too much.
……………………………………………………………………………………………………………...
……………………………………………………………………………………………………………...
……………………………………………………………………………………………………………...
……………………………………………………………………………………………………………...
……………………………………………………………………………………………………………...
……………………………………………………………………………………………………………...
……………………………………………………………………………………………………………...
……………………………………………………………………………………………………………...
……………………………………………………………………………………………………………...
……………………………………………………………………………………………………………...
……………………………………………………………………………………………………………...
……………………………………………………………………………………………………………...
……………………………………………………………………………………………………………...
……………………………………………………………………………………………………………...
……………………………………………………………………………………………………………...

6
PRACTICE TEST 2
I. PHONETICS
1.1. Choose the word whose underlined part is differently pronounced from the others.
1. A. threaten B. earth C. healthy D. breathe
2. A. passion B. discussion C. expression D. decision
1.2. Choose the wordthat differs from the other three in the position of primary stress in each of the
following questions.
1. A. employment B. remember C. concentrate D. position
2. A. express B. effort C. office D. comment
3. A. incidence B. tendency C. difference D. importance
II. READING COMPREHENSION
2.1. Read the text below and decide which answer A,B,C or D fits each space.
What is the world largest desert? I'm sure the first that (1)……………to mind is the legendary Sahara - but
that's 3rd. It's actually the desert on the Antarctic (2)……………, measuring just under 14,000,000kms2,
closely followed by the Arctic desert. Most people living away from deserts associate this kind of (3)………
with sand, but only 10% of deserts are actually made up of sand (4).……………
The term ‘desert' in fact describes a (5)……………which receives almost no (6) ……………, meaning
rainfall, snow, ice or hail. The term can also apply to regions where there is greater evaporation of (7)………
than rainfall. In other words, more water is absorbed back into the (8)……………than stays on or within the
ground. So, in (9)……………deserts, you're mainly talking about ice sheets and a little rock, not sand, of
course. The surface of many other deserts is comprised of loose rock where the finer particles of dust and
sand have been (10)……………away.
1. A. jumps B. leaps C. rises D. springs
2. A. country B. continent C. territory D. pole
3. A. terrain B. plain C . horizon D. nature
4. A. hills B. dunes C. piles D. slopes
5. A. base B. landscape C. ground D. soil
6. A. wildlife B. agriculture C. precipitation D. alteration
7. A. moisture B. drops C. dampness D. drizzle
8. A. setting B. atmosphere C. environment D. surroundings
9. A. extreme B. Mediterranean C. typical D. polar
10. A. left B. disappeared C. blown D. removed
2.2. Read the following passage and mark the letter A, B, C or D to indicate the correct answer to each
of the questions.
The lion dance has a long history in China. There are many different stories about how the lion dance
first started, but no one knows for sure. One story tells us that long time ago, a lion appeared in a small
village in China and attacked the people and animals there. The villagers didn't know what to do and offered
ten pieces of gold to anyone who could help them.
A kung-fu fighter heard about this and went to the village. He fought with the lion three times but
couldn't kill it. So he taught kung-fu to some of the villagers to help him kill the lion. A few months later,
they killed the lion. After this the villagers celebrated the event every year by following the steps of those
who fought the lion in a dance.
Another story tells us there was once an emperor who used to have bad dreams every night and
couldn't sleep. One night he saw a lion dance in his dream and the bad dreams just disappeared.
When he woke up that morning, he showed his guards how to do the dance. From then on, they did
the dance on that same day every year and the emperor never had bad dreams again.
Lion dances take place at people's homes during Chinese New Year. Chinese people believe that the
dancers will bring good luck to the homes they visit. Two people do the dance, one at the head and another at
7
the tail of the lion. They move the lion's eyes, mouth and ears to make it look alive during the dance. Loud
music is played and sometimes firecrackers are fired during the dance. This loud noise helps to keep evil
spirits away and to bring good luck. The lion dance is one of the most colorful events of the Chinese New
Year celebration.
1. How did the lion dance first start?
A. When the kung-fu fighter killed the lion.
B. When the emperor had a dream.
C. When the emperor taught his guards how to do the dance.
D. No one knows for sure.
2. Why did the kung-fu fighter go to the village?
A. He wanted to get some gold.
B. He wanted to teach kung-fu to the villagers.
C. He wanted to kill the lion.
D. He wanted to teach the people how to do the lion dance.
3. How did the villagers celebrate the kil li ng of the lion?
A. They taught kung-fu to everyone in the village.
B. They gave gold to all the people who killed the lion.
C. They followed the steps of the fighters every year in a dance.
D. They gave ten pieces of gold to the kung-fu fighter every year.
4. What happened when the emperor saw the lion dance in his dream?
A. He woke up quickly. B. The bad dreams disappeared.
C. His guards woke him up. D. He couldn't sleep after that.
5. Why do people light firecrackers during the lion dance?
A. The loud noise keeps evil spirits away.
B. The loud noise brings good luck.
C. Both these reasons.
D. Neither of these reasons.
III. GRAMMAR AND VOCABULARY
3.1. Choose the correct answer to each of the following questions.
1. In many countries, it is……………..for children to receive primary education.
A. methodical B. compulsory C. general D. typical
2. So far they……………..hands to create a cozy family atmosphere.
A. join B. joined C. have joined D. are joining
3. The survey will cover various ways of……………..the different attitudes toward love and marriage of
today youth.
A. determination B. determine C. determined D. determining
4. Be careful not to……………..any mistakes in your letter of application.
A. do B. make C. have D. cause
5. The man to……………..you were talking is a music teacher.
A. that B. who C. whom D. which
6. The boss…………….. to his secretary using the office phone for personal calls.
A. disapproves B. criticizes C. disagrees D. objects
7. He'd prefer……………..chicken soup rather than……………..eel soup.
A. have - have B. having - to have
C. to have - have D. to have - having
8. After the test papers……………..to the students in class tomorrow, the students……………..their next
assignment.
A. will return - will be given B. are returned - will be given
8
C. are returning - are giving D. will be returned - are given
9. As a millionaire who liked to show off her wealth, Mrs. Smith paid……………..we asked.
A. four time as many as B. four times much as
C. four times as much as D. four time much than
10. Peter is not used……………..late for school. He prefers punctuality.
A. to being B. to be C. being D. been
3.2. Antonyms & Synonyms
(a) Choose the word(s) CLOSEST in meaning to the underlined word(s) in each of the following
questions.
1. I'm becoming increasingly absent-minded. Last week, I locked myself out of my house twice.
A. being considerate of things B. remembering to do right things
C. forgetful of one's past D. often forgetting things
2. Sports and festivals form an integral part of every human society.
A. informative B. delighted C. exciting D. essential
(b) Choose the word(s) OPPOSITE in meaning to the underlined word(s) in each of the following
questions.
1. Although it's a long way for us, we feel we are contented with what we do.
A. interested B. dissatisfied C. excited D. shocked
2. I can't stand people who treat animals cruelly.
A. gently B. cleverly C. reasonably D. brutally
3.3. Conversation
Choose the most suitable response to complete each of the following exchanges.
1. A: “How are you getting on?” - B: “…………….”
A. All right B. Not bad C. It's Ok D. All are correct
2. A: “I've passed my driving test.” - B: “…………….”
A. Congratulations! B. That's a good idea.
C. It's nice of you to say so. D. Do you?
3.4. Give the correct form of the given words
1. Recently, health foods have increased in…………….. (POPULAR)
2. Honesty is an…………….quality. (ADMIRE)
3. Sorry about the mistakes, I…………….the instruction you gave me. (UNDERSTAND)
4. He is interested in the…………….of old buildings. (PRESERVE)
IV. WRITING
4.1. Rewrite each of the following sentences in such a way that it means the same as the sentence
printed before it.
1. New Year comes near. People are busy.
The nearer………………………………………………………………………………………………..
2. Both Peter and John don't like tea.
Neither …………………………………………………………………………………………………..
3. Mary will only come to the party if Peter is invited.
Unless ……………………………………………………………………………………………………
4. In spite of the bad weather, we went swimming.
Although …………………………………………………………………………………………………
5. He is very sorry he didn't come to the meeting yesterday.
He apologizes …………………………………………………………………………………………….
4.2. Build complete sentences with the given words.
1. factory/close/workers/go/strike/last week.
……………………………………………………………………………………………………………
9
2. It/important/brother/stop/smoke/once.
……………………………………………………………………………………………………………
3. policeman/order/children/not/play football/street.
……………………………………………………………………………………………………………
4. She/ another attempt/climb/mountain/next year.
……………………………………………………………………………………………………………
5. It/wrong/you /not/give/her/chance/talk.
……………………………………………………………………………………………………………
4.3. Write a paragraph (about 120 - 150 words) about your future dreams.
……………………………………………………………………………………………………………
……………………………………………………………………………………………………………
……………………………………………………………………………………………………………
……………………………………………………………………………………………………………
……………………………………………………………………………………………………………
……………………………………………………………………………………………………………
……………………………………………………………………………………………………………
……………………………………………………………………………………………………………
……………………………………………………………………………………………………………
……………………………………………………………………………………………………………
……………………………………………………………………………………………………………
……………………………………………………………………………………………………………
……………………………………………………………………………………………………………
……………………………………………………………………………………………………………

10
PRACTICE TEST 3
I. PHONETICS
1.1. Mark the letter A, B, C or D to indicate the word whose underlined part differs from the other
three in pronunciation in each of the following questions.
1. A. question B. division C. television D. decision
2. A. blood B. fool C. food D. pool
3. A. worked B. hoped C. moved D. brushed
1.2. Mark the letter A, B ,C or D to indicate the word that differs from the other three in the position
of the primary stress in each of the following questions.
1. A. indifferent B. comparative C. conservative D. sympathetic
2. A. contestant B. promotion C. implement D. ambition
II. READING COMPREHENSION
2.1. Read the following passage and mark the letter A, B, C, or D to indicate the correct answer to each
of the questions.
Early humans saw a variety of natural colours around them, from the browns and greens of the soil
and plants to the deep blues and red of the sky. They painted their bodies with colours from nature to signal
aggression towards an enemy, or to make themselves attractive to a mate.
A girl and a rainbow of crushed ice flavours at Chowpatty Beach, India. Over the centuries, the
sources of colours such as blue, purple and red were highly valued and they were often worth as much as
gold. In the 19th century, a young chemistry student manufactured the first synthetic dye, and suddenly the
world became a much more colourful place, in the 20th century, scientists discovered the psychological
effects of colours, and people found ways to use this discovery to influence our feelings and behaviour.
RED
Red, the colour of blood, symbolises fire, love and anger. In Eastern cultures, people believe it brings
luck, wealth and success. In humans, the colour red can send different messages. Some people redden, for
example, when they are angry or embarrassed. Researchers have discovered that in sports the team that is
wearing red is more likely to win. Why? Because red seems to be the colour that signals dominance, giving
those dressed in red an advantage in sporting events. In many animal species (including humans), contact
with this bold colour causes the heart rate to increase. However, one of red’s lighter shades, pink, can have
the opposite effect on people. Men in prisons are less aggressive when the walls are a specific shade of pink.
YELLOW
Yellow, the colour that comes to mind when we think of sunshine, is found throughout nature and the
man-made world as a colour that commands attention; indeed, it is one of the easiest colours to see. This
highly visible shade is found on everything from school buses to traffic signs and pens that we use to
highlight important information in a text. The colour is also used to caution people; football players, for
example, are shown a yellow card as a reminder to behave. It can be used as a stimulant as well: in a number
of studies, yellow has been found to help children focus on their work and do better at school.
BLUE
Blue, the colour of the sky and sea, is associated in many cultures with water, religious objects, and
protection against evil. Its darker shades represent calm, stability and power. Dark blue, for example, is the
colour of the business suit or police uniform; it tells others, ‘I am in control,’ or ‘I am trustworthy.’ Blue is
also associated with sadness. It’s common in English, for example, when you are feeling sad or depressed, to
talk about ‘feeling blue’, while in Iran, blue is the colour of mourning, worn when a person dies.
Like pink, blue has a calming effect on people. Rooms painted blue help people to relax or sleep.
Sleeping pills are often coloured blue to suggest exactly this idea. This colour also seems to reduce feelings
of hunger. Blue food is rarely seen in nature, and when it is, such food is usually no longer healthy to
consume. It’s just one more example of the power that colour can hold over us.
1. According to the text, the colour red…………….
11
A. is believed to embarrass people
B. gives an advantage to team members who wear it
C. symbolises bad luck
D. makes people less aggressive
2. Yellow is used to highlight information in a text because……………..
A. it is an important colour
B. it can be used to caution people
C. people prefer this colour to other colours
D. it is a highly visible colour
3. What is this text mainly about?
A. how views of colour have changed B. how colour influences people
C. the sources of colour D. how colour has a calming effect
4. What are English speakers referring to when they talk about "feeling blue " (paragraph 5, line 5)?
A. being in control B. being upset when someone dies
C. being calm D. being sad
5. The first man-made colours were produced……………..
A. only recently B. in the 20th century
C. in the 19th century D. centuries ago.
2.2. Read the text carefully and choose the correct answer A,B,C or D:
Because writing has become so important in our (1)……………, we sometimes think of it as more
real than speech. A little thought, (2) ……………, will show why speech is primary and writing secondary to
language. Human beings have been writing for (3) …………… 5000 years; but they have been talking for
much longer, doubtless ever since there have been human (4) …………… When writing (5) ……………
develop, it was derived (6) ……………and represented speech, albeit imperfectly. Even today there are
spoken languages that have no written form. (7) ……………, we all learn to talk well before we learn to
write; any human child who is not severely handicapped physically or mentally will learn to talk: a normal
human being cannot be prevented from doing so. (8) ……………, it takes a special (9) ……………to learn
to write; in the past many intelligent and useful members of society didn’t acquire the skill, and even today
many who speak languages with writing (10) ……………never learn to read or write, while some who learn
the rudiments of those skills do so only imperfectly.
1. A. culture B. education C. training D. competition
2. A. though B. nevertheless C. however D. furthermore
3. A. at least B. at last C. totally D. finally
4. A. people B. personality C. beings D. being
5. A. rather B. quite C. has D. did
6. A. with B. of C. about D. from
7. A. However B. Otherwise C. Furthermore D. But
8. A. On behalf of B. On the other hand
C. Another way D. By the way
9. A. affect B. affair C. effort D. effect
10. A. sign B. signal C. systems D. net
III. GRAMMAR AND VOCABULARY
3.1. Choose the correct answer to each of the following questions.
1. Armed terrorists are reported to have……………..the embassy.
A. taken up B. taken to C. taken over D. taken into
2. Newspapers vary greatly in their……………..to the government.
A. opinion B. attitude C. view D. bias
3.……………..that increasing numbers of compact-disc players will be bought by consumers in the years to
12
come.
A. They are anticipated B. It is anticipated
C. Anticipating D. In anticipation
4. You shouldn’t go to the meeting……………..you’ve got an invitation.
A. except B. if C. without D. unless
5. He couldn’t afford to……………..his car repaired.
A. pay B. make C. do D . get
6. I would……………..you didn’t leave just at the moment.
A. rather B. like C. ask D. advise
7. I don’t remember……………..the front door when I left home this morning.
A. to lock B. locking C. locked D. to have locked
8.……………..the fact that it was raining heavily, he went out without a raincoat.
A. Despite of B. In spite of C. However D. Although
9.……………..that we were late, we began to run.
A. Knowing B. To knowing C. We know D. As knowing
10. She was……………..right when she criticized you.
A. reasonably B. reasonable C. very D. reason
3.2. Put the correct form of words.
1. The boy was very violent and his parents found him…………….. (MANAGE)
2. Jim is one of the most……………..members of the committee. (SPEAK)
3. In particular, parents are afraid of changes in their children’s…………….. (PERSON)
4. She studied……………..at university. (ECONOMY)
3.3. Antonyms &: Synonyms
(a) Mark the letter A, B, C or D on your answer sheet to indicate the word(s) CLOSEST in meaning to
the underlined word(s) in each of the following questions.
1. I don’t think I can ever live on my own in a big city.
A. poorly B. freely C. hungry D. alone
2. We had waited for two hours and in the end they arrived.
A. luckily B. eventually C. suddenly D. gradually
(b) Mark the letter A, B, C or D on your answer sheet to indicate the word(s) OPPOSITE in meaning
to the underlined word(s) in each of the following questions
1. The milk is delivered at 6 a.m., so we always have fresh milk for breakfast.
A. sour B. new C. dirty D. old
2. There has been a significant shortage of new homes in the region.
A. poverty B. lack C. fall D. abundance
3.4. Conversation
Choose the most suitable response to complete each of the following exchanges.
1. Peter: “Is it important?” Thomas: “……………..”
A. Not on your life! B. It’s a matter of life and death!
C. No worry, that’s nothing. D. It’s ridiculous.
2. Ms. Janet: “I’d like to try on this dress, please.” Shop assistant: “……………..”
A. You’re welcome. B. You’re so nice.
C. By all means. D. That’s right.
IV. WRITING
4.1. Complete the second sentence so that it has a similar meaning to the first sentence, using the word
given. Do not change the word given. You must use between two and five words, including the word
given.
1. Tim looks nothing like his father. (TAKE)
13
Tim…………………………………………………………………………..his father at all.
2. I regret giving Dennis my phone number. (DENNIS)
I wish…………………………………………………………………………..my phone number.
3. “Is there anything you want from the shops?” Alison asked her mother. (THERE)
Alison asked her mother if………………………………………from the shops.
4. Oh bother, we don’t have any milk left. (RUN)
Oh bother, we’ve………………………………………milk.
5. We advise customers to buy their tickets in advance. (ADVISED)
Customers………………………………………………their tickets in advance.
4.2. Build complete sentences with the given words.
1. Many people/fond/go/countryside/weekends.
……………………………………………………………………………………………………………..
2. It/take/her/two hours/finish/such/difficult test.
……………………………………………………………………………………………………………..
3. I/invite/party/occasion/her birthday/last week.
……………………………………………………………………………………………………………..
4. Tet/time/Vietnamese people/celebrate/beginning/spring.
……………………………………………………………………………………………………………..
5. Telephone/invent/Alexander Graham Bells.
……………………………………………………………………………………………………………..
4.3. Write a paragraph (about 120 - 150 words) about a festival you know using the following
suggestions:
- The name of the festival
- What the festival is
- Where the festival is held
- When the festival is held
- How the festival is held
- Why the festival is held
- What you think about the festival
……………………………………………………………………………………………………………..
……………………………………………………………………………………………………………..
……………………………………………………………………………………………………………..
……………………………………………………………………………………………………………..
……………………………………………………………………………………………………………..
……………………………………………………………………………………………………………..
……………………………………………………………………………………………………………..
……………………………………………………………………………………………………………..
……………………………………………………………………………………………………………..
……………………………………………………………………………………………………………..
……………………………………………………………………………………………………………..

14
PRACTICE TEST 4
I. PHONETICS
1.1. Choose the word whose main stress is different from the others.
1. A. solar B. image C. danger D. oasis
2. A. reference B. interview C. government D. understand
3. A. scholarship B. develop C. equipment D. discourage
1.2. Choose the word whose underlined part is differently pronounced from the others.
1. A. tough B. rough C. through D. enough
2. A. exhibition B. holiday C. childhood D. hilarious
II. READING COMPREHENSION
2.1. Choose the word or phrase which best completes each blank in the following passage:
My home is in the air - I do an enormous amount of travelling. It is a fast life and (1)…………..of
work, but I like it and that is the only way (2) …………..me . Everything is tiring - music, travelling - but
what can I do? I am not (3) …………..to complaining. It is hard to imagine now (4) …………..I will ever be
very long in one place. My home town is on the Caspian Sea. There is sea, wind, sun, and (5)…………..
many tourists and hotels. I have my own flat with four or five rooms, but I am seldom there. If I am there for
a day or two, I prefer to (6) …………..with my mother and grandmother. They live in a small house, (7)..…
it is very comfortable and my mother cooks for me. I like good, simple food. I have no wife, no brothers or
sisters and my father (8) …………..when I was seven. He was an engineer and I don’t (9). …………..him
very well. He liked music very much and wanted me to (10) …………..a musician.
1. A. most B. full C. complete D. more
2. A. for B. to C. in D. by
3. A. wanted B. taken C. used D. known
4. A. and B. so C. while D. that
5. A. far B. too C. much D. more
6. A. stay B. go C. do D. spend
7. A. but B. since C. even D. which
8. A. killed B. gone C. passed D. died
9. A. know B. remember C. remind D. see
10. A. become B. turn C. develop D. grow
2.2. Read the following passage and mark the letter A, B, C, or D to indicate the correct answer to each
of the questions.
Thanksgiving is celebrated in the US on the fourth Thursday in November. For many Americans, it is
the most important holiday apart from Christmas. Schools, offices and most businesses close for
Thanksgiving, and many people make the whole weekend a vacation.
Thanksgiving is associated with the time when Europeans first came to North America. In 1620, the
ship the May flowers arrived, bringing about 150 people who today are usually called Pilgrims. They arrived
at the beginning of a very hard winter and could not find enough to eat, so many of them died. But in the
following summer, Native Americans showed them what foods were safe to eat, so that they could save food
for the next winter. They held a big celebration to thank God and the Native Americans for the feet that they
had survived.
Today people celebrate Thanksgiving to remember these early days. The most important part of the
celebration is a traditional dinner with foods that come from North America. The meal includes turkey, sweet
potathes (also called yams) and cranberries, which are made into a kind of sauce or jelly. The turkey is filled
with stuffing or dressing, and many families have their own special recipe. Dessert is pumpkin made into a
pie.
On Thanksgiving, there are special television programs and sports events. In New York there is the
Macy’s Thanksgiving Day Parade, when a long line of people wearing fancy costumes march through the
15
streets with large balloons in the shape of imaginary characters.
Thanksgiving is considered the beginning of the Christmas period, and the next day many people go
out to shop for Christmas presents.
1. According to the passage, Pilgrims are……………
A. native Americans who live in North America
B. people who left their home and went to live in North America in 1620s
C. people who traveled to America by ships
D. trips that religious people make to a holy place
2. Which of the following is NOT true?
A. In the US, Thanksgiving is not a national holiday; it’s a religious holiday
B. Christmas comes less than a month after Thanksgiving
C. The Macy’s Thanksgiving Day Parade is colourful and exciting
D. Thanksgiving was originally celebrated by the first Europeans in North America to thank God for
theft survival
3. In the United States, Thanksgiving is…………….
A. celebrated as a public holiday
B. a religious celebration held by Christians only
C. apart from Christmas
D. more important than Christmas
4. All of the following statements are mentioned EXCEPT…………….
A. People usually have traditional dinners on Thanksgiving
B. There are lots of entertainments on Thanksgiving
C. People celebrate Thanksgiving to thank God
D. People go to churches for religious services on Thanksgiving
5. Which of the following statements is not TRUE about Thanksgiving……………?
A. People go out to shop for Christmas presents
B. People wear colored costumes marching through the streets
C. Turkey, yams and pumpkin pies are served
D. People join in the Macy’s Thanksgiving Day Parade
III. GRAMMAR AND VOCABULARY
3.1. Choose the correct answer to each of the following questions.
1. They have just found the couple and their car……………were swept away during the heavy storm last
week.
A. that B. which C. whose D. when
2. I was brought……………in the countryside by my aunt after my parents had passed……………
A. on/over B. for/on C. on/off D. up/away
3. Since I……………a child, I have liked to solve maths puzzles.
A. am B. was C. have been D. had been
4. The zookeeper wanted us……………near the bars.
A. that we didn’t put our hands B. don’t put out hands
C. not putting our hands D. not to put our hands
5. ……………we had planned everything carefully, a lot of things went wrong.
A. Because B. Because of C. Despite D. Although
6. No one can predict the future exactly. Things may happen…………….
A. expected B. unexpected C. expectedly D. unexpectedly
7. It’s our responsibility to contribute to……………our own lives.
A. growing B. heightening C. bettering D. increasing
8. Oliver used to go fishing and……………
16
A. so did I B. I did not C. so I did D. so did me
9. It would be hard to name areas……………computers are not being used.
A. how B. which C. what D. where
10.……………, he felt so unhappy and lonely.
A. Rich as was he B. Rich as he was
C. In spite of his being wealth D. Despite his wealthy
3.2. Antonyms &: Synonyms
(a) Choose the word(s) CLOSEST in meaning to the underlined word(s) in each of the following
questions.
1. Lucy will be like a dog with two tails if she gets into the team.
A. very exhausted B. extremely pleased C. very proud D. extremely dazed
2. I’ll take the new job whose salary is fantastic.
A. reasonable B. wonderful C. pretty high D. accept
(b) Choose the word(s) OPPOSITE in meaning to the underlined word(s) in each of the following
questions.
3. Our traditions are very ancient and our people are very proud of them.
A. modern B. real C. old D. young
4. It is quite incredible that he is unaware of such basic facts.
A. difficult B. unbelievable C. imaginable D. disappointed
3.3. Conversation
Choose the most suitable response to complete each of the following exchanges.
1. Theo: “Do you mind if I switch the light off?” Nuttel: “……………..”
A. Yes, I mind it, sorry.
B. What if I don’t mind it?
C. I’d rather you didn’t, if you don’t mind.
D. Yes, please do it.
2. Hana and Jenifer are talking about a book they have just read.
Hana: “The book is really interesting and educational.” Jenifer: “……………..”
A. I’d love it. B. Don’t mention it.
C. That’s nice of you to say so. D. I couldn’t agree more.
3.4. Give the correct form of the given words
1. David has been a bit……………..today. TROUBLE
2. My boss was so angry that he was absolutely…………….. SPEECH.
3. These clothes are attractive but entirely…………….. PRACTICE.
4. Thank you for your …………….. GENEROUS.
IV. WRITING
4.1. Rewrite the sentences without changing their meaning using the given words.
1. It took us three hours to open the door.
We ………………………………………………………………………………………………………...
2. I don’t really like her, even though I admire her achievement.
In spite …………………………………………………………………………………………………….
3. My English friend finds using chopsticks difficult.
My English friend isn’t ……………………………………………………………………………………
4. "I’ve seen the film three times, Mary", said George.
George told…………………………………………………………………………………………………
5. My brother studies now harder than he used to.
My brother …………………………………………………………………………………………………
4.2. Build complete sentences with the given words.
17
1. You/not pass/coming exam/unless/work/hard.
……………………………………………………………………………………………………………..
2. James Watt/Scottish scientist/invent/steam engine.
……………………………………………………………………………………………………………..
3. sometimes/a country/refuse/take part/the Olympics.
……………………………………………………………………………………………………………..
4. I/apologise/him/not able/arrive/on time.
……………………………………………………………………………………………………………..
5. Mars/be/140 million miles/know/red planet.
……………………………………………………………………………………………………………..
4.3. Write a paragraph (about 120 - 150 words) about your leisure activities
……………………………………………………………………………………………………………..
……………………………………………………………………………………………………………..
……………………………………………………………………………………………………………..
……………………………………………………………………………………………………………..
……………………………………………………………………………………………………………..
……………………………………………………………………………………………………………..
……………………………………………………………………………………………………………..
……………………………………………………………………………………………………………..
……………………………………………………………………………………………………………..
……………………………………………………………………………………………………………..
……………………………………………………………………………………………………………..

18
PRACTICE TEST 5
I. PHONETICS
1.1. Choose the word whose main stress is different from the others.
1. A. mosquito B. document C. literature D. business
2. A. magazine B. preference C. cigarette D. engineer
1.2. Choose the word whose underlined part is differently pronounced from the others.
1. A. healthy B. whom C. honest D. heal
2. A. amuses B. repeats C. attacks D. coughs
3. A. stomach B. change C. watch D. church
II. READING COMPREHENSION
2.1. Read the following passage and mark the letter A, B, C, or D to indicate the correct answer to each
of the questions.
Whales are the largest animals in the world, and the gentlest creatures we know. Although the whale
is very huge, it is not hindered at all by its size when it is in the water. Whales have tails that end like
flippers. With just a gentle flick, it can propel itself forward. The skin of a whale is so smooth that it does not
create any friction that can slow the whale down. A whale’s breathing hole is located on the top of its head,
so it can breathe without having to completely push its head out of the water. Whales are protected from the
cold seawater by body fat that is called blubber.
Whales live in the ocean but, in terms of behaviours, they are more similar to humans than fish. They
live in family groups and they even travel in groups when they have to migrate from cooler to warmer
waters.
The young stay with their parents for as long as fifteen years. Whales are known not to desert the ill
or injured members; instead, they cradle them.
When whales are in danger, there are people who go to great lengths to help them. One such case
occurred in 1988, when three young whales were trapped in the sea. It was close to winter and the sea had
begun to freeze over. Whales are mammals that require oxygen from the air, so the frozen ice was a great
danger to them. All they had then was a tiny hole in the ice for them to breathe through. Volunteers from all
over soon turned up to help these creatures. They cut holes in the ice to provide more breathing holes for the
whales. These holes would also serve as guides for the whales so that they could swim to warmer waters.
1. Which of the following best describes the main idea of this passage?
A. Whales as the only animals to live in warm water.
B. Successful attempts to rescue whales all over the world.
C. Some remarkable similarities of whales to humans.
D. Whales as the largest, gentlest but vulnerable creatures.
2. Whales can move easily in water thanks to their…………...
A. tail and blubber B. skin and head
C. size and head D. tail and skin
3. According to the passage, why was the frozen ice on the sea surface a danger to whales?
A. Because they couldn’t eat when the weather was too cold.
B. Because whales couldn’t breathe without sufficient oxygen.
C. Because they couldn’t swim in icy cold water.
D. Because the water was too cold for them as they were warm-blooded.
4. According to paragraph 2, the author mentions all of the following to show that whales “are more similar
to humans” EXCEPT…………...
A. the young stay with their parents for almost fifteen years
B. they do not migrate from cooler to warmer waters
C. they do not desert the ill or injured members
D. they live in family groups and travel in groups
19
5. The word “tiny” in paragraph 3 probably means…………....
A. very deep B. very ugly C. very small D. very fat
2.2. Read the following passage and mark the letter A, B, C, or D indicate the correct word(s) for each
of the blanks
Along (1)…………….jogging and swimming, cycling is one of the best all-round forms of
exercise. It can help to increase your strength and energy, giving you more (2) ……………muscles and
stronger heart. But increasing your strength is not the only advantage of cycling. Because you are not (3)
……………the weight of your body on your feet. It is a good form of exercise for people with painful feet or
backs. However, with all forms of exercise, it is important to (4) ……………slowly and build up gently.
Doing too much too quickly can damage muscles that are not (5)…………...to working. If you have any
doubts about taking( 6) ……………cycling for health reasons, talk to your doctor and ask his or her
advice.
Ideally you should be cycling at (7) ……………two or three times a week. For the exercise to be
doing you (8) ……………, you should get a little out of breath. Don’t worry that if you begin to lose your
breath, it could be dangerous and there must be something wrong with your heart. This is simply not true,
shortness of breath (9) ……………that the exercise is having the right effect. However, if you find you are in
pain, (10) ……………you should stop and take a rest.
1. A. on B. at C. by D. with
2. A. confident B. efficient C. better D. reliable
3. A. bringing B. carrying C. lifting D. arising
4. A. make B. take C. start D. do
5. A. have B. ought C. used D. made
6. A. in B. up C. out D. on
7. A. best B. all C. least D. times
8. A. good B. well C. nice D. fine
9. A. tells B. shows C. points D. appears
10. A. then B. though C. even D. yet
III. GRAMMAR AND VOCABULARY
3.1. Choose the correct answer to each of the following questions.
1. This room…………….since I was born.
A. has been painted B. was painted C. painted D. has painted
2. Have you seen the Titanic yet? - No, I haven’t. I…………….it next Saturday.
A. would see B. will see C. am going to see D. see
3. After…………….dinner, I often watch TV.
A. ate B. eat C. eaten D. eating
4. Tom said that he…………….his motorbike the day before.
A. had lost B. lost C. has lost D. lose
5. Overwork is…………….to cause increased stress.
A. likely B. possible C. possibly D. obviously
6. The husband is ill in……………hospital, so she has to stay at…………..home to look after…………..
children instead of going to work.
A. Ø / Ø / Ø B. the/the/the C. Ø / Ø /the D. Ø/the/the
7. The beautiful woman has a busy…………..life.
A. society B. socialize C. social D. socializing
8. English has become the main language of .…………..
A. communication B. communicate C. communicant D. communicative
9. I spoke very slowly…………..he didn’t understand English very well.
A. to B. so that C. because D. so
20
10. Can we find…………..at the hotel for the night?
A. house B. room C. accommodation D. stay
3.2. Put the correct form of words.
1. It is…………..for our students to make mistakes in English. AVOID
2. Teenagers are now…………..dressed. FASHION
3. Novelists are among the most…………..people in the world. IMAGINE
4.…………..is sometimes a very bad habit. CURIOUS
3.3. Antonyms & Synonyms
(a) Mark the letter A, B, C or D on your answer sheet to indicate the word(s) CLOSEST in meaning to
the underlined word(s) in each of the following questions.
1. If people’s interference with the environment decreases, more species will survive and produce offspring.
A. result B. descent C. children D. ancestor
2. Hunting for meat and burning forests for soil cause destruction to wildlife.
A. protection B. damage C. organization D. contamination
(b) Choose the word(s) OPPOSITE in meaning to the underlined word(s) in each of the following
questions.
3. Love is supposed to follow marriage, not precede it.
A. take out B. find out C. happen D. come after
4. The city of Hue is very famous for its Royal Tombs.
A. infamous B. popular C. little - known D. notorious
3.4. Conversation
Choose the most suitable response to complete each of the following exchanges.
1. Tim: ’’Make yourself at home.” Mai: “…………..”
A. Yes. Can I help you? B. Thanks! Same to you.
C. Not at all. Don’t mention it. D. That’s very kind. Thank you.
2. Alusa: “I got 8.0/9.0 for the IELTS test!" Liu: “……………”
A. It’s OK. I’m proud of you.
B. Good for you. Thank you.
C. Well done, son! I’m very proud of you.
D. You can do it.
IV. WRITING
4.1. Make all the changes and additions necessary to produce from the following sets of words and
phrases that together make a complete paragraph.
1. Bill Gates/born/1955/Washington State. He/grow up/rich family.
…………………………………………………………………………………………………………….
2. His parents/send/he/private school. There/he/meet/business partner/Paul Allen.
…………………………………………………………………………………………………………….
3. When/they/be/8th grade, they/write/programs/business/computers.
…………………………………………………………………………………………………………….
4. 1973/Gates/be accepted/Harvard University. Parents/happy.
…………………………………………………………………………………………………………….
5. Two years later/Gates/drop out/of Harvard/work/computer program/his friend Allen.
…………………………………………………………………………………………………………….
6. 1975 /they/create /company/Microsoft/sell/their product.
…………………………………………………………………………………………………………….
7. A few years later/Microsoft/become/giant company.
…………………………………………………………………………………………………………….
8. By 1990/Gates/the youngest/billionaire/the United States/age of 34.
21
…………………………………………………………………………………………………………….
9. He/achieve/success/a lot of hard work. He/be/“ King of Software ”
…………………………………………………………………………………………………………….
10. By 1997, he/the richest/man/United States.
…………………………………………………………………………………………………………….
4.2. Rewrite the sentences so that they are nearest in meaning to the sentences printed before them.
1. He can’t lift the table because he is not strong enough.
If……………………………………………………………………………………………………………
2. It is a pity her brother can’t speak English as fluently as a native speaker.
Her brother wishes…………………………………………………………………………………………
3. She doesn’t usually drive a car very fast.
She isn’t used ………………………………………………………………………………………………
4. It is reported that the city has increasingly developed in the past few years.
The city…………………………………………………………………………………………………….
5. My sister prefers staying at home and watching TV to going to the cinema.
My sister would……………………………………………………………………………………………
4.3. Write a paragraph in about 140 words about the importance of learning English.
……………………………………………………………………………………………………………..
……………………………………………………………………………………………………………..
……………………………………………………………………………………………………………..
……………………………………………………………………………………………………………..
……………………………………………………………………………………………………………..
……………………………………………………………………………………………………………..
……………………………………………………………………………………………………………..
……………………………………………………………………………………………………………..
……………………………………………………………………………………………………………..

22
TRƯỜNG THCS NGOẠI NGỮ
PRACTICE TEST 1
Part I. Look and read. Choose the correct words and write them on the lines
baby-sitter/mother tongue/brother - in- law/science fiction/traffic jam/box office/dining room/income
tax
1. Stories about the future are called………...
2. Your sister’s husband or your husband’s brother is your………....
3. Your first language is also called your………....
4. The place where you buy tickets for the theatre or cinema is the………....
5. A long line of cars moving slowly is a………....
Part II. Choose the correct answer to complete the sentence.
1.………...the weather forecast, it will rain heavily later this morning.
A. Because of B. Due to C. On account of D. According to
2. Sharon did not attend the meeting because she was………...the weather.
A. in B. under C. on D. at
3. We didn’t………...to the station in time to catch the train.
A. get B. reach C. make D. arrive
4. The pay was not brilliant but I could………...and there were many aspects of the job that I enjoyed.
A. earn high salary B. make a living
C. do a living D. get high salary
5.………...the problem of water pollution gets more serious, the government is searching for a way to deal
with it.
A. Because B. Although C. However D. Providing
6. Do you think he is………...of doing the job?
A. suitable B. capable C. able D. competent
7. Mary always takes great care………...her children.
A. with B. for C. of D. to
8. More and more investors are pouring………...money into food and beverage start-ups.
A. the B. a C. an D. Ø
9. I really eat vegetables now, but I………...them when I was a child.
A. eat B. used to eat C. would eat D. didn’t use to eat
10. The policeman is happy………...the kids safe and sound now.
A. finding B. be found C. to find D. have found
Part III. Choose sentences (A-J) to complete the following conversation
Nam: What do we need to cook eel soup for family members?
Mai: (1)………...
Nam: Can you tell me how to cook eel soup, please? What should I do first?
Mai: (2)………...
Nam: What should I do when the rice soup is ready?
Mai: (3)………...
Nam: What should I do with the eel?
Mai: (4)………...
Nam: OK, and after that?
Mai: (5)………...
Nam: What should I do when the eel is cooked?
Mai: (6)………...
Nam: What should I do at last?
23
Mai: (7)………...
Nam: How can we serve the eel soup?
Mai: (8)………...
Nam: (9)………...
Mai: (10)………...
A. First, clean rice, pour into the pot and cook until it’s well done, add a little salt.
B. We need 300 grams of eel, 150 grams of rice, fish sauce, ginger, and green onion.
C. Thank you very much. Your instructions are very clear.
D. Finally, when rice is well done, add meat eel into the pot, put the fish sauce, sugar, and pepper to suit your
taste.
E. Clean the eel carefully: first use water of lemon, apply it on the body of the eel and rub it.
F. I suggest you should eat when it is hot. Add some sliced green onion on the surface of the dish.
G. After steam, separate meat and bone eel carefully. Use the meat only.
H. You’re welcome. I hope eel soup will be the best.
I. Next, you clean green onion, ginger and then smash the ginger.
J. After that, put the eel in a plate and steam it until it’s cooked.
Part IV. Read and choose a word from the box. Write the correct word next to numbers 1-10.
it matter who likely other languages on pronunciation
from because differences there countries they common meet

English is the first language of many people in (1)……………outside the United Kingdom. When
you (2) ……………speakers of English from around the world, you can notice that they do not all speak in
the same way. There are also some (3) ……………in the words they use, including the names of (4) ……
objects that are part of everyone’s daily life. Although (5) ……………and vocabulary are not the same
everywhere, it is interesting that English speakers (6) ……………opposite sides of the world can understand
each (7) ……………quite easily. It does not seem to (8) ……………where they learnt the language. And of
course this is one reason why speakers of other (9) ……………are keen on learning English, too. If you
know English, you are more (10) ……………to be able to study or work in all sorts of exciting places, such
as the United States or Australia.
Part V. Read the following passage and choose the correct answer A,B,C or D.
Inventions That Changed Our World
People have been inventing things for years. Paper was invented about 2,000 years ago. The wheel was
invented more than 5,000 years ago. What would life be like without paper to write on or bikes to ride?
Those inventions have made life easier.
Airplane
The Wright Brothers invented the first airplane in December 1903. That makes 2003 the 100th
anniversary! Before the airplane was invented, most people traveled by cars, boats, and trains. Today,
airplanes help people travel faster. How do airplanes help you?
Telephone
Alexander Graham Bell invented the telephone in 1876. Before the telephone was invented, people
kept in touch by writing letters or talking in person. Today the telephone makes it easier for people to talk to
one another. How do telephones help you?
Automobile
The first four - wheeled gas - powered car was invented by Karl Benz in 1891. Before people had
cars, they couldn’t travel easily. They walked or rode horses for short trips. They took trains or boats for long
trips. Today cars are one of the main ways people travel. How do cars help you?
Computer
People worked for years to make the first computer. It was built in 1946. The first computer was
24
called ENIAC. Today computers are an important part of many people’s lives. People use computers as tools
to write, get information, and much more. How do computers help you?
1. Which of the following are two inventions in the passage that were invented before 1900?
A. the telephone and the car B. the car and the computer
C. the computer and the airplane D. the airplane and the telephone
2. What do all the inventions described in this text have in common?
A. All these inventions were invented in the twentieth century.
B. All these inventions were invented by Americans.
C. All these inventions made people’s lives easier.
D. All these inventions changed how people travel.
3. It can be inferred from the passage that……………
A. the Wright Brothers spent a long time trying to invent the airplane
B. someone had the idea to invent the computer in the 1940s or early 1950s
C. Karl Benz didn’t like to walk or ride horses for short trips
D. Alexander Graham Bell wrote many letters in his lifetime
4. What does the word “anniversary” in the second paragraph most nearly mean?
A. an interesting idea B. an event
C. a statement of fact D. a celebrated date
5. The primary purpose of this passage is to describe…………….
A. the invention of the airplane in December 1903
B. some inventions that have made a big difference
C. some ways that computers help make our lives easier
D. the reasons that Karl Benz invented the gas - powered car
Part VI. Rewrite each sentence so that the meaning stays the same.
1. Sue is excited about going to Spain on holiday.
Sue is looking …………………………………………………………………………………………….
2. I haven’t decided to continue my study in a foreign country.
I haven’t made ……………………………………………………………………………………………
3. He was a fool to say that.
It was ……………………………………………………………………………………………………..
4. He couldn’t afford to buy the car.
The car ……………………………………………………………………………………………………
5. We weren’t surprised by his success.
It came ……………………………………………………………………………………………………
Part VII. Write a paragraph about your daily routine, (about 120 words)
.……………………………………………………………………………………………………………
.……………………………………………………………………………………………………………
.……………………………………………………………………………………………………………
.……………………………………………………………………………………………………………
.……………………………………………………………………………………………………………
.……………………………………………………………………………………………………………
.……………………………………………………………………………………………………………
.……………………………………………………………………………………………………………
.……………………………………………………………………………………………………………
.……………………………………………………………………………………………………………

25
PRACTICE TEST 2
Part I. Look and read. Choose the correct words and write them on the lines
factory/suburbs / environment / commercial centre / shopping centre/continent/car park/library /
1. You can borrow books or study there:……………
2. The area outside the city center where people live:…………….
3. An area where there are company offices and banks in a town:…………….
4. The air, land and water around us:…………….
5. A place where goods (washing machines, furniture, etc.) are made:…………….
Part II. Choose the correct answer to complete the sentence.
1. It’s a good idea in theory, but it’s going to be hard to put it into……………
A. trial B. examination C. practice D. test
2. Thanks for the meal.……………, how’s your brother getting on?
A. By the way B. Moreover
C. To put it another way D. For example
3. She often…………..her own clothes.
A. makes B. does C. produces D. cuts
4. It wasn’t an awful experience. It was the worst thing…………..has ever happened to me.
A. what B. why C. that D. which
5. A good…………..of English will help you find a job more easily.
A. condition B. expectation C. satisfaction D. knowledge
6. It is raining hard. We had better…………..at home tonight.
A. to stay B. staying C. stay D. to be staying
7. The computer’s memory is the place where information…………..and calculations are done.
A. kept B. keep C. are kept D. is kept
8. I like his essay because it’s very…………...
A. imaginary B. imaginative C. imagination D. imaginable
9. As an………….., Mr. Pike is very worried about an increase in teenager crimes.
A. educate B. educational C. education D. educator
10. With greatly increased workloads, everyone is…………..pressure now.
A. under B. above C. upon D. out of
Part III. Choose sentences (A-J) to complete the following conversation
A B
1. I have spots. A. Wear a sun hat.
2. I get sunburnt. B. Wear warm clothes and drink hot water.
3. I have toothache. C. Eat less junk food.
4. I feel sad. D. Go outside and play with friends.
5. I’m putting on weight. E. Stay at home and take a rest.
6. My eyes are getting weaker. F. Stop eating too many sweets.
7. I feel tired. G. Spend less time on computer.
8. I have a cold. H. Don’t shout, sing or chew gum.
9. I have a temperature. I. Cool down your room.
10. I have a sore throat. J. Wash your face regularly.

Part IV. Read and choose a word from the box. Write the correct word next to numbers 1-10.
oldest best age for her and loves mother’s
is have carefully to lives mine but outside
Of all my relatives, I like my aunt Emily the (1)…………….She’s my (2) …………….youngest
26
sister. She has never married and (3) …………….alone in a small village near Bath. She’s fifty years old, (4)
…………….she’s quite young in spirit. She has a fair complexion, thick brown hair and dark brown eyes.
She has a kind face, and when you meet (5) …………….the first thing you notice (6) …………….her smile.
Her face is a little wrinkled now, but I think she is still rather attractive. She is the sort of person you can
always go to if you (7) …………….a problem and she’s extremely generous. She likes reading (8) …………
gardening, and she still goes (9) …………….long walks over the hills. I hope I’m as active as she is when
I’m at her (10) ……………..
Part V. Read the following passage and choose the correct answer to each of the questions.
The idea of a driverless car is not new. Now there are already cars on the road that can park
themselves. But a truly self-driving car, one that can drive itself, is being tested by scientists and engineers.
Self-driving cars have already tested on public roads. These cars not only record images of the road,
but their computerised maps view road signs, find alternative routes and see traffic lights even before they
are seen by a person. By using radars and cameras, the cars can analyse information about their surroundings
faster than a human can.
In some of the tests, the car learns the details of a road by driving on it several times, and when it is
time to drive itself, it can identify when there are pedestrians crossing and then stops to let them pass by.
Self-driving cars could make transport safer for all of us by eliminating the cause of most of today’s
accidents.
Although self-driving cars may seem far off, some people believe that there will be driverless cars in
showrooms in about 10 years.
1. What is the main idea of the passage?
A. Self-driving car: a far off image B. The prospect of driverless cars
C. Computerised maps in cars D. Safer transport in the future
2. Now there are already cars on the road that can…………...
A. park themselves B. record images of the road
C. learn the details of a road D. actually drive themselves
3. How can the tested cars analyse information about their surroundings?
A. By viewing road signs. B. By seeing traffic lights quickly.
C. By using radars and cameras. D. By driving on it several times.
4. The underlined word “them” in the passage refers to…………...
A. tests B. details C. times D. pedestrians
5. In about a decade’s time, we may see…………...
A. self-driving cars being tested B. driverless cars in a far-off place
C. self-driving cars in showrooms D. much safer cars
Part VI. Rewrite each sentence so that the meaning stays the same.
1. Keeping the environment clean is very important.
It’s ……………………………………………………………………………………………………
2. You’ll certainly meet lots of people in your new job.
You are ………………………………………………………………………………………………
3. “I’m working in a restaurant, and I don’t care much for it.’’she said.
She said …………………………………………………………………………………………………
4. In spite of the bad weather, they had a wonderful holiday.
Although ………………………………………………………………………………………………..
5. They suggested banning advertisements on TV.
They suggested that …………………………………………………………………………………….
Part VII. Write a paragraph (about 100 - 120 words) about your future job.
.……………………………………………………………………………………………………………
.……………………………………………………………………………………………………………
27
.……………………………………………………………………………………………………………
.……………………………………………………………………………………………………………
.……………………………………………………………………………………………………………
.……………………………………………………………………………………………………………
.……………………………………………………………………………………………………………
.……………………………………………………………………………………………………………
.……………………………………………………………………………………………………………
.……………………………………………………………………………………………………………

28
PRACTICE TEST 3
Part I. Look and read. Choose the correct words and write them on the lines
bandage / crutch/ambulance / conscious/first-aid / stretcher/handkerchief/scale/
1. Awake and able to see, hear and think:…………...
2. Treatment given to an injured person before a doctor comes:…………....
3. A vehicle equipped to carry sick or injured people to hospital:…………....
4. An instrument used for weighing people or things:…………....
5. A long thin piece of cloth used for tying round and protecting a wound:…………...
Part II. Choose the correct answer to complete the sentence.
1. This shirt is…………...that one.
A. a bit less expensive B. as expensive as
C. not so many expensive as D. much far expensive than
2. Linda…………...her identity card again. This is the second time this…………....
A. lost - has happened B. has lost -happened
C. has lost - has happened D. lost - happened
3.…………...is enjoyable.
A. Playing cards B. Play cards C. Cards D. Cards play
4. Our project was successful…………...its practicality.
A. regardless B. in terms of C. on behalf of D. with a
view to
5. There is…………...in my bedroom.
A. an old square wooden table B. a square old wooden table
C. a square wooden old table D. an old wooden square table
6. Mary wondered…………...her father looked like now, after so many years away.
A. how B. that C. whose D. what
7. You…………...forget what I told you. It’s very important.
A. mustn’t B. needn’t C. don’t have to D. can’t
8. Let’s go out for a walk,…………..?
A. do we B. did we C. shall we D. have we
9. If you do not want to…………..the chance,…………..a decision soon.
A. miss/makes B. take/make C. miss/make D. miss/take
10. He is looking…………..his daughter because she is ill.
A. for B. after C. into D. about
Part III. Choose sentences (A-J) to complete the following conversation.
A. Oh, good. Maybe you can give me some tips. What are the things I shouldn’t miss?
B. Yes, there are lots of really good restaurants in Connaught Place. I remember one called, Shanti - the food
there is delicious. We ate there every night.
C. Yes, I’ve heard that before. Thanks a lot, you’ve been really helpful.
D. No problem.
E. Guess what? I’m going to Delhi next week.
F. Like most places, don’t drink the water. Buy bottle water instead. And I wouldn’t eat anything that’s sold
in the street. You can get ill quite easily there.
G. Thanks, that’s really useful. Have you got any other tips?
H. That’s great! I’ve been there twice.
I. You should definitely see the Red Fort, in Old Delhi - It’s absolutely huge.
J. Great, I’ll definitely go there. And, what about the food?
Do you know any good place?
K. What are you going to do next week?
29
Write your answers here:
0. K 1….. 2….. 3….. 4.….. 5….. 6.….. 7…... 8.….. 9.….. 10.…..
Part IV. Read and choose a word from the box. Write the correct word next to numbers 1-10.
most dream like give are one well next
them their door laugh when festival put enjoyment
In Canada and the United States,…………… (1) of the most popular days in the year is Halloween.
Halloween is on October 31st. It’s a day…………… (2) some people dress up in strange or unusual
costumes. For example, they may dress up to look…………… (3) an animal, a person from a book or film or
a famous person from history. In some places, children go to school in …………… (4) Halloween costumes.
After dark, many young children…………… (5) on their costumes and visit their neighbours. They knock on
the …………… (6) and shout “Trick or treat!”. Then the neighbours …………… (7) them some candy, and
the children go on to the …………… (8) house. Adults also enjoy dressing up for Halloween. There………
(9) usually Halloween parties in the evening and usually there is a prize for the best or…………… (10)
unusual costume.
Part V. Read the passage and answer the following questions by choosing A,B,C or D.
Many people like to eat pizza, but not everyone knows how to make it. Making the perfect pizza can
be complicated, but there are lots of ways for you to make a more basic version at home. When you make
pizza, you must begin with the crust. The crust can be hard to make. If you want to make the crust yourself,
you will have to make dough using flour, water, and yeast. You will have to knead the dough with your
hands. If you do not have enough time to do this, you can use a prepared crust that you buy from the store.
After you have chosen your crust, you must then add the sauce. Making your own sauce from scratch
can take a long time. You have to buy tomatoes, peel them, and then cook them with spices. If this sounds
like too much work, you can also purchase jarred sauce from the store. Many jarred sauces taste almost as
good as the kind you make at home.
Now that you have your crust and your sauce, you need to add the cheese. Cheese comes from milk,
which comes from cows. Do you have a cow in your backyard? Do you know how to milk the cow? Do you
know how to turn that milk into cheese? If not, you might want to buy cheese from the grocery store instead
of making it yourself.
When you have the crust, sauce, and cheese ready, you can add other toppings. Some people like to
put meat on their pizza, while other people like to add vegetables. Some people even like to add pineapple!
The best part of making a pizza at home is that you can customize it by adding your own favorite ingredients.
1. The author’s main purpose in writing this passage is to…………..
A. outline steps to make a basic pizza at home
B. describe the history of pizza
C. provide tips about how to make your pizza especially delicious
D. teach a healthier way to make pizza
2. As used in paragraph 2, which is the best synonym for purchase?
A. forget B. cook C. buy D. ask
3. In paragraph 3, the author asks a series of questions in order to…………...
A. reinforce the idea that most people probably live on farms
B. prove that store-bought cheese tastes better than homemade cheese
C. emphasize the superiority of homemade cheese over store bought cheese
D. support the idea that most people cannot make homemade cheese
4. Which of the following words best describes how the author feels about making a pizza from scratch?
A. enthusiastic B. understanding C. negative D. helpful
5. Which of the following conclusions would work best at the end of this passage?
A. Once you have prepared the crust, sauce, cheese, and toppings, you are ready to bake your pizza. I
think you will see that making pizza at home can be a good alternative to purchasing it from the store.
30
B. It can be understood that making your pizza from scratch should be avoided at all costs. Use store
bought ingredients and save yourself a heap of trouble.
C. Although the crust, sauce, and toppings are all important ingredients in pizza, it is clear that the
cheese is most important. Therefore, be sure your cheese is homemade.
D. As you can see, cooking a pizza can be fun, but it can also be very expensive. But, as you can see,
the best things are worth paying for.
Part VI. Rewrite each sentence so that the meaning stays the same.
1. The bread was so stale that the children couldn’t eat it.
The bread was too ……………………………………………………………………………………….
2. There is no point in crying over spilt milk.
It’s no ……………………………………………………………………………………………………
3. We couldn’t find George anywhere.
George was ……………………………………………………………………………………………
4. Unless he takes these pills, he won’t be better.
If………………………………………………………………………………………………………….
5. We were late for school because of the heavy rain.
Because it ………………………………………………………………………………………………..
Part VII. Write a paragraph (about 100 - 120 words) about the benefits of living in the countryside.
.……………………………………………………………………………………………………………
.……………………………………………………………………………………………………………
.……………………………………………………………………………………………………………
.……………………………………………………………………………………………………………
.……………………………………………………………………………………………………………
.……………………………………………………………………………………………………………
.……………………………………………………………………………………………………………
.……………………………………………………………………………………………………………
.……………………………………………………………………………………………………………
.……………………………………………………………………………………………………………

31
PRACTICE TEST 4
Part I. Match each word to its meaning
WORD MEANING
1. forest A. a community of people smaller than a town
2. stream B. a small house in the country
3. village C. the trees and other plants in a large densely wooded area
4. lake D. a steep descent of the water of a river
5. cottage E. domesticated bovine animals such as cows, buffaloes
6. field F. grass mowed and cured for use as fodder
7. waterfall G. a piece of land cleared of trees and usually enclosed
8. hay H. land covered with grass and other low plants suitable for grazing animals
9. pasture I. a body of (usually fresh) water surrounded by land
10. cattle J. a natural body of running water flowing on or under the earth

Write your answers here:


0.K l…… 2.……3.…… 4.…… 5……. 6.……7.……8.……9.…… 10……
Part II. Choose the correct answer to complete the sentence.
1. “How tall is Ralph?” “He’s…………..than you are.”
A. slight taller B. slight tall C. slightly taller D. slightly tall
2. We cleaned up the room as soon as the guests…………..
A. have left B. had left C. left D. were leaving
3. The town…………..we are living is noisy and crowded.
A. at which B. where C. in where D. which
4. For the last 20 years, we……………significant changes in the world of science and technology.
A. have witnessed B. witness C. witnessed D. are witnessing
5. Ms. Brown asked me……………in my class.
A. how many students were there B. were there how many students
C. how many students there were D. there were how many students
6. They are always……………of what their children do.
A. supportive B. support C. supportively D. supporting
7. Housewives find it easier to do domestic chores thanks to……………invention of labour-saving devices.
A. some B. a C. the D. an
8. It was……………that we decided to stay indoors.
A. such a hot day B. so a day hot
C. such a day hot D. so a hot day
9. I was made……………when I was at school.
A. study hard B. to study hard C. to studying hard D. studying hard
10. In the evening, there is a football match……………8.00 and 10.00.
A. in B. before C. after D. between
Part III. Fill in each gap with the missing question or sentence in order to finish the conversation
between the reporter (R) and the villager (V)
• What do you think of them?
• Would you like them to come back?
• Did they give you anything?
• Well, they’ll come back soon, I guess. Well, I have to go. Have a good day.
• Do you still get wet after the repair?
V: My house had a hole in the roof and we got wet when it rained.
32
R: 1……………………………..
V: Well, it’s good now. We don’t get wet any more.
R: 2.……………………………..
V: Yes. They gave us a big carton of 30 packets of instant noodles, 10 kilograms of rice, and some canned
fish.
R: 3.……………………………..
V: They were very kind and nice. I love them.
R: 4.……………………………..
V: Do you know when they come back?
R: 5.……………………………..
V: Sure. Thank you very much. Goodbye.
R: Goodbye.
Part IV. Fill in each blank in the following passage with ONE suitable word from the box. Write the
correct word next to numbers 1-10.
students protect scientists off energy
gold natural urban save wastes
electricity bulbs coal beautiful rural

With the development of science and technology, more and more (1)……………..is used every day.
Nowadays, many (2) …………….areas in Vietnam have been electrified, and more (3) …………….is
needed. If everyone (4) …………….energy, it will quickly run out. (5) …………….at school should learn
how to (6) …………….energy, for example, they should turn (7)……………the electric lights when they
leave their classrooms.
Moreover, we may waste electricity at home if we use a lot of light (8) …………….. By not wasting
energy, we will save a lot of (9) …………….resources, such as (10) ……………. and oil.
Part V. Read the passage and answer the following questions by choosing A,B,C or D.
Different cultures follow their own special customs when a child’s baby teeth fall out. In Korea, for
example, they have the custom of throwing lost teeth up on the roof of a house. According to tradition, a
magpie will come and take the tooth. Later, the magpie will return with a new tooth for the child. In other
Asian countries, such as Japan and Vietnam, children follow a similar tradition of throwing lost teeth onto the
roofs of houses.
Birds aren’t the only animals thought to take lost teeth. In Mexico and Spain, tradition says a mouse
takes a lost tooth and leaves some money. However, in Mongolia, instead of mice, dogs are responsible for
taking teeth away. Dogs are highly respected in Mongolian culture and are considered guardian angels of the
people. Tradition says that the new tooth will grow in good and strong if the baby tooth is fed to a guardian
angel. Accordingly, parents in Mongolia will put their child’s lost tooth in the fat of a piece of meat and feed
it to a dog.
The idea of giving lost teeth to an angel or fairy is also a tradition in the West. Many children in
Western countries count on the Tooth Fairy to leave money or presents in exchange for a tooth. The exact
origins of the Tooth Fairy are a mystery, although the story probably began in England or Ireland centuries
ago. According to tradition, a child puts a lost tooth under his or her pillow before going to bed. In the wee
hours, while the child is sleeping, the Tooth Fairy takes the tooth and leaves something else under the pillow.
In France, the Tooth Fairy leaves a small gift or candy. In the United States, however, the Tooth Fairy
usually leaves money. These days, the rate is 1 to 5 per tooth. That can add up to a lot of money from the
Tooth Fairy!
1. Where did the story of the Tooth Fairy probably come from?
A. North America B. Western Africa
C. Eastern Asia D. Northern Europe
33
2. Why do some people throw a tooth on the roof?
A. To get a larger house B. To get a new tooth
C. To get rich D. To have a long life
3. It can be inferred from paragraph three that………….
A. the Tooth Fairy began in England or Ireland centuries ago
B. many children in Western countries look forward to leaving money or presents
C. when the Tooth Fairy appeared are still unknown nowadays
D. children always put their lost teeth under their pillows while going to bed.
4. What is NOT traditionally put under a pillow in exchange for a tooth?
A. Candy B. Money C. A gift D. A new tooth
5. What is the main idea of this reading?
A. Some tooth customs involve animals.
B. Many Asian countries have similar traditions concerning teeth.
C. Different cultures have different traditions about teeth.
D. The Tooth Fairy leaves money for teeth.
Part VI. Rewrite each sentence so that the meaning stays the same.
1. Study hard or you will get bad marks.
Unless ………………………………………………………………………………………………….
2. Please don’t play your music so loudly.
Would you mind ………………………………………………………………………………….
3. They spend millions of dollars on advertising every year.
Millions …….………………………………………………………………………………………….
4. He is the best football player in his team.
No one……….…………………………………………………………………………………………
5. The plan may be ingenious, but it will never work in practice.
Ingenious.……………………………………………………………………………………………….
Part VII. Write a paragraph (about 100 - 120 words) about your favourite subject at school.
.……………………………………………………………………………………………………………
.……………………………………………………………………………………………………………
.……………………………………………………………………………………………………………
.……………………………………………………………………………………………………………
.……………………………………………………………………………………………………………
.……………………………………………………………………………………………………………
.……………………………………………………………………………………………………………
.……………………………………………………………………………………………………………
.……………………………………………………………………………………………………………

34
PRACTICE TEST 5
Part I. Find the word which fits the definition
1. You get washed in it:…………..
2. You keep plates in it:…………..
3. You can see yourself in it:…………..
4. You bite the food with them:…………..
5. You keep your clothes in it:…………..
Part II. Choose the correct answer to complete the sentence.
1. My brother has no…………..in football.
A. interest B. interestingly C. interesting D. interested
2. My mother advised me…………..an apple every day to stay healthy.
A. eat B. eating C. ate D. to eat
3. Fire safety in family houses,…………..most fire deaths occur, is difficult.
A. why B. how C. where D. when
4. Thanks for helping me out. I’ll…………..the favor some time.
A. return B. pay C. send D. do
5. I decided to go to the library as soon as I…………...
A. finished what I did B. finished what I was doing
C. would finish what I did D. finish what I did
6. After you…………..all your homework, you will be allowed to go out.
A. had done B. are doing C. did D. have done
7. I bought this grammar book…………..I could go over all the things we have studied this year.
A. that B. with a view to C. so that D. in order to
8. Today, many serious childhood diseases…………..by early immunization.
A. prevent B. can be prevented C. can prevent D. are preventing
9. Oliver used to go fishing and…………...
A. so did I B. I did not C. so I did D. so did me
10. I am…………..tired to think about that problem now.
A. much more B. nearly C. far too D. very
Part III. A girl is talking about food. Match the short passages (1-5) with the topics (a-f), and write the
answers a,b,c,d,e, or f in the blanks.
There is one extra topic.
a. the international food she likes best
b. the traditional food she enjoys most
c. how good she is at cooking
d. how healthy her eating habits are
e. her favourite restaurant
f. the food she doesn’t like
1. …………I’m lucky. My mum is a great cook and she loves cooking. She likes teaching me and my brother
too, so I can cook quite a lot of dishes. My mum’s dishes are better but my dishes are OK.
2. …………I eat most things. I was a vegetarian for a few months last year but I love meat so I stopped. I
don’t like food with a lot of salt or fat in it. Oh, I know, mushrooms. I hate them. They are awful.
3. …………That’s a difficult question. We always go on holiday to a different country and every year I come
home and say that the food was the best in the world! Our last holiday was to Japan so now I love Japanese
food but I love Vietnamese food, too.
4. …………I try to eat good food and I try not to eat unhealthy snacks. If I’m hungry, I eat carrots, not
sweets, but at the weekend, my mum always makes cakes and I always eat a lot. I love them.
5. …………Last week, my parents took me to a very nice restaurant for my birthday. The food was great but
35
the restaurant was very formal. Next year, I want to go to Beach Restaurant. They are very friendly and the
food is good. I think that’s my favourite.
Part IV. Fill in each blank in the following passage with ONE suitable word from the box. Write the
correct word next to numbers 1-10.
that so in what where for have interest
is which because house however nice built not

The country is more beautiful than a town and pleasant to live in. Many people think so, and go to the
country (1)…………….the summer holidays though they can not live there all the year round. Some have a
cottage built in a village (2) …………….that they can go there whenever they can find the time. English
villagers are not all alike, but (3) ……………. some ways they are not very different from each other.
Almost every village has a church, the round or square tower of (4) ……………. can be seen for many miles
around. Surrounding the church is the churchyard, (5) …………….people are buried. The village green (6)
……………. a wide stretch of grass, and houses or cottages are (7) ………… round it. Country life is now
fairly comfortable and many villages (8) ……………. water brought through pipes into each (9) …………
Most villages are so close to some small town that people can go there to buy (10) …………….they can’t
find in the village shops.
Part V. Read the passage and answer the following questions by choosing A,B,C or D.
In the primary school, a child is in a comparatively simple setting and most of the time forms a
relationship with one familiar teacher. On entering secondary school, a new world opens up and frequently it
is a much more difficult world. The pupil soon learns to be less in the way he speaks to teachers and even to
his fellow pupils. He begins to lose gradually the free and easy ways of the primary school, for he senses the
need for a more cautious approach in the secondary school where there are older pupils. Secondary staff and
pupils suffer from the pressures of academic work and seem to have less time to stop and talk. Teachers with
specialist roles may see hundreds of children in a week and a pupil may be able to form relationships with
very few of the staff.
He has to decide which adults are approachable; good schools will make clear to every young person
from the first year what guidance and personal help is available but whether the reality of life in the
institution actually encourages requests for help is another matter.
Adults often forget what a confusing picture school can offer to a child. He sees a great deal of
movement, a great number of people - often rather frightening-looking people - and realises that an
increasing number of choices and decisions have to be made. As he progresses through the school, the
confusion may become less but the choices and decisions required will increase. The school will rightly
expect the pupil to take the first steps to obtain the help he needs, for this is the pattern of adult life for which
he has to be prepared, but all the time the opportunities for personal and group advice must be presented in a
way which makes them easy to understand and within easy reach of pupils.
1. The teachers at secondary school do not talk much to the pupils because ……………
A. they want to keep a certain distance with the pupils
B. they are too busy with their academic work
C. it is the regulation of the school
D. the pupils are afraid of them
2. In this passage about secondary schools, the author is mainly concerned about……………
A. the training of the individual teachers
B. academic standards
C. the personal development of pupils
D. the role of specialist teachers
3. In secondary schools, every pupil having problems should…………….
A. know how to ask for help
36
B. be able to discuss his problems in class
C. be freed from any pressure of academic work
D. be able to discuss his problems with any teacher
4. What will the school rightly expect the pupils to do?
A. they think carefully B. they understand well
C. they study hard D. they firstly obtain the help
5. According to the passage, one of the problems for pupils entering secondary school is that…………….
A. the teachers do not want to be friendly
B. they are taught by many different teachers
C. they do not attend lessons in every subject
D. the teachers give most attention to the more academic pupils
Part VI. Rewrite each sentence so that the meaning stays the same.
1. I only recognized him when he came into the light.
It was …………………………………………………………………………………………………..
2. It is pointless to have that old typewriter repaired.
That old typewriter is not ……………………………………………………………………………..
3. I have never seen such beautiful pictures before.
These pictures …………………………………………………………………………………………
4. Although he didn’t speak Dutch, Bob decided to settle in Amsterdam.
In spite …………………………………………………………………………………………………
5. He looks older when he wears glasses.
Glasses make …………………………………………………………………………………………..
Part VII. Write a paragraph (in about 100 - 120 words) about your favorite foods and drinks.
………………………………………………………………………………………………………….
………………………………………………………………………………………………………….
………………………………………………………………………………………………………….
………………………………………………………………………………………………………….
………………………………………………………………………………………………………….
………………………………………………………………………………………………………….
………………………………………………………………………………………………………….
………………………………………………………………………………………………………….
………………………………………………………………………………………………………….

37
TRƯỜNG THCS CHẤT LƯỢNG CAO
PRACTICE TEST 1
I. Choose the word whose main-stressed syllable is different from the rest.
1. A. mosquito B. elephant C. dinosaur D. buffalo
2. A. protect B. settle C. relate D. compose
3. A. government B. understand C. interview D. reference
4. A. tonight B. control C. purpose D. involve
5. A. enough B. aware C. support D. swallow
II. Choose the word whose underlined part is different from the rest in pronunciation.
1. A. leather B. tea C. lead D. leave
2. A. autumn B. summer C. sunny D. much
3. A. Valentine B. determine C. examine D. heroine
4. A. dreams B. thinks C. hours D. gardens
5. A. contributed B. eradicated C. developed D. needed
III. Reorder the words to form sentences.
1. I/by Daniel Defoe /like/Robinson Crusoe,/the main character/in/a book/who is/.
A. I like Robinson Crusoe, who is the main character in a book by Daniel Defoe.
B. Who is the main character I like Robinson Crusoe, in a book by Daniel Defoe.
C. in a book by Daniel Defoe I like Robinson Crusoe, who is the main character.
D. I like Robinson Crusoe by Daniel Defoe, who is the main character in a book.
2. so/enjoyed/much/it’s/years/I/myself/since/.
A. It’s years since I enjoyed myself so much.
B. Since I enjoyed myself so much it’s years.
C. I enjoyed myself since it’s years so much.
D. It’s so much years since I enjoyed myself.
3. was/person/the/Mary/pass/this/only/test /to/.
A. The only person to pass this test Mary was.
B. Mary was the only person this test to pass.
C. Mary was the only person to pass this test.
D. Mary was this person to only pass the test.
4. for/nowhere/the/train/sit/was/crowded/that/was/Mary/to/so/there/.
A. The train was crowded so there was nowhere for Mary to sit that.
B. The train was that crowded so nowhere was for Mary to sit there.
C. The train was so crowded nowhere there was for Mary to sit that.
D. The train was so crowded that there was nowhere for Mary to sit.
5. humour/always/people/his/laugh/sense/of/makes/.
A. His sense of humour always makes laugh people.
B. His sense of humour always makes people laugh.
C. His humour always laugh makes sense of people.
D. People always makes laugh his sense of humour.
IV. Choose the sentence that is closest in meaning to each of the following questions.
1. It is English pronunciation that puzzles me most.
A. Puzzling me most is how to pronounce English.
B. Pronouncing English words is not complicated.
C. English pronunciation is difficult for me.
D. I was not quick at English pronunciation at school.
2. “Stop smoking or you’ll be ill,” the doctor told me.
38
A. The doctor advised me to give up smoking to avoid illness.
B. I was ordered not to smoke to recover from illness.
C. I was warned against smoking a lot of cigarettes.
D. The doctor suggested smoking to treat illness.
3. They couldn’t climb up the mountain because of the storm.
A. The storm made them impossible to climb up the mountain.
B. The storm discouraged them from climbing up the mountain.
C. Their climbing up the mountain was unable due to the storm.
D. The storm made it not capable of climbing up the mountain.
4. Eight years ago we started writing to each other.
A. We have rarely written to each other for eight years.
B. We have been writing to each other for eight years.
C. Eight years is a long time for us to write to each other.
D. We wrote to each other eight years ago.
5. We’ve run out of tea.
A. There’s not much more tea left.
B. We didn’t have any tea.
C. There’s no tea left.
D. We have to run out to buy some tea.
V. Choose the best answer.
1. It rained very hard…………., we got home safely.
A. As a result B. So C. However D. Therefore
2. You will have to work hard if you want to…………..
A. success B. succeed C. successfully D. successful
3. It is of great importance to create a good impression………….your interviewer.
A. for B. on C. at D. about
4. Although I spoke to him many times, he never paid any………….to what I said.
A. compliment B. informality C. attention D. notice
5. I’d like to have my shoes………….at once.
A. repair B. repairing C. repaired D. being repaired
6. It was………….we could not go out.
A. such dark that B. so dark that C. very so dark D. too dark that
7. Isn’t it easy to learn English well? - ………….
A. No, it is B. No, it does not C. Yes, it is D. Yes. It does
8. There are two restaurants in the town and………….of them are extremely old.
A. each B. all C. both D. most
9. If he eats all that food, he eats…………..
A. quite much B. too many C. quite many D. too much
10. Mr Robinson speaks Vietnamese very…………..
A. skillful B. good C. fluently D. best
11. They received her letter after she …………..to the new house.
A. has moved B. had moved C. was moving D. would move
12. There were many scientists at the meeting, several of…………..were very young.
A. that B. which C. who D. whom
13. Turn off the light,…………..?
A. will you B. shall we C. can you D. shall I
14. Would you mind…………..the door for me?
A. open B. opening C. opens D. opened
39
15. It’s interesting ……………to some music now.
A. to listen B. listening C. has listened D. listened
16. I can’t believe it ! My purse……………
A. has been stolen B. is stolen C. was stolen D. is being stolen
17. Public……………in this city is quite good, and it’s not expensive.
A. journey B. travel C. vehicle D. transport
18. Bicycles……………in the driveway.
A. must not be leaving B. must not leave
C. must not be left D. must not have left
19. The film was so……………. However, my father saw it from beginning to end.
A. interesting B. exciting C. boring D. fascinating
20. She is a……………girl with…………….
A. green-eyes/red hair B. green-eyed/red hairs
C. green-eye/a red hair D. green-eyed/red hair
21.……………is not very far from here to the harbour.
A. This B. It C. There D. That
22. -“Do you like the weather here?” - “I wish it…………….”
A. doesn’t rain B. didn’t rain C. won’t rain D. hadn’t rain
23. - “More coffee? Anybody?” - “……………”
A. I don’t agree, I’m afraid B. I’d love to
C. Yes, please D. It’s right, I think
24. A: “Those girls are very good at dancing!” - B: “……………..”
A. So do I! B. I know, they are
C. Yes, they do! D. They were fine, thanks
25. The old building…………….20 years ago.
A. is built B. were build C. was built D. were built
VI. Read the text below. Choose the correct answer.
Most sore throats are caused by an infection which treatment with antibiotics cannot cure. But with
simple remedies, the patient normally gets better in 4 or 5 days.
Tonsillitis, however, usually starts with a sore throat which causes pain on swallowing. With children
- and some adults - there may be a fever and the patient is obviously not feeling well. It may be possible to
see white spots on the back of the throat.
Sometimes a sore throat may occur with the common cold, and with influenza, there may be dryness
of the throat, pain on coughing and loss of voice.
Treatment:
Aspirin: To help relieve the pain on swallowing and the fever (if there is one). Use aspirin tablets
dissolved in water so that the patient can gargle before swallowing. Repeat the treatment every 4 hours.
Drink: Encourage the patient to drink plenty.
Food: Food should not be forced on a patient who does not want to eat.
Steam: If there is pain in the throat on coughing, breathing in steam may help.
Children: Young children, who may not be able to gargle, should be given aspirin dissolved in water
every 4 hours in the right dose for their age.
At one year: A single junior aspirin.
At five years: Half an adult aspirin.
At eight years: One whole adult aspirin.
When to see the doctor:
If the sore throat is still getting worse after 2 days.
If the patient complains of earache.
40
If the patient’s fever increases.
If the parent of patient is worried.
1. According to the passage, it would appear that most sore throats……………..
A. require an immediate treatment with an antibiotic
B. respond to treatment with an antibiotic
C. rarely turn out to be serious illnesses
D. result in tonsillitis even when treated
2. One of the signs of tonsillitis can often be……………..
A. difficulty in swallowing food B. pain in the chest when coughing
C. white spots on the neck D. earache during the first days
3. In order to treat a sore throat, one should……………..
A. prevent the patient from eating too much
B. give the patient 4 aspirin tablets every hour
C. make certain the patient drink plenty of liquid
D. make the patient gargle with hot liquid
4. You should call the doctor if……………...
A. the infection spreads to other people
B. swelling occurs around the ears
C. the patient’s throat is still sore after two days
D. the patient’s condition continues to worsen
5. What difference is there in the way adults and young children should be treated with aspirin?
A. Young children should not be allowed to gargle with it.
B. Adults should be given tablets to swallow whole.
C. Young children should be given aspirin more often than adults.
D. Adults should be given larger doses of aspirin than children.

41
PRACTICE TEST 2
I. Choose the word whose underlined part is pronounced differently from that of the others
1. A. chairs B. sofas C. rooms D. sinks
2. A. meat B. reading C. bread D. seat
3. A. never B. often C. when D. tennis
4. A. Thursday B. thanks C. these D. birthday
5. A. surround B. counting C. country D. about
II. Choose the word whose main stress is different from the others.
1. A. receive B. copy C. differ D. listen
2. A. dangerous B. energy C. television D. expensive
3. A. exist B. avoid C. support D. notice
4. A. happy B. easily C. Vietnamese D. plentiful
5. A. recycle B. description C. contribute D. atmosphere
III. Choose the best answer to complete the following sentences
1. In order to buy his house, he had to obtain a large………….from his bank.
A. finance B. capital C. loan D. debt
2. You are………….to clean the bathroom.
A. supposed B. declared C. spoken D. imagined
3. If the weather………….good, I………….camping with classmates.
A. is/go B. is/will go C. will be/go D. will be/will go
4. ………….little we may like it, old age comes to most of us.
A. In spite of B. So C. However D. As
5. Rob eats a lot of fast food and he………….on a lot of weight.
A. takes B. spends C. puts D. brings
6. You are not allowed to smoke in this hall.
A. forbidden B. asked C. let D. made
7. “They listen to the music with great interest” means…………..
A. They enjoy the music B. They hate the music
C. They dislike the music D. They never listen to that music
8. If people turn off all electric appliances in one hour all over the world, they can save lots of…………..
A. electric B. electrical C. electrician D. electricity
9. He………….English in this school since he………….here.
A. has taught/moved B. taught/moved
C. taught/has moved D. has taught/has moved
10. I would rather you………….me the truth.
A. telling B. to tell C. told D. tell
IV. Mark the letter A, B, C or D to indicate the word(s) CLOSEST in meaning to the underlined
word(s) in each of the following questions.
1. This organization was established to meet the needs of those who wanted to travel into space.
A. found B. founded C. set in D. set off
2. Let’s wait here for her; I’m sure she’ll turn up before long.
A. arrive B. return C. enter D. visit
3. Young people are now getting more and more concerned about environmental problems.
A. worried B. nervous C. hopeless D. uneasy
4. In Vietnam, it is customary to choose a favourable day for occasions such as weddings, funerals, or
house-moving days.
A. beautiful B. favourite C. suitable D. whole
5. The situation seems to be changing minute by minute.
42
A. again and again B. from time to time
C. very rapidly D. time after time
V. Read the passage and choose the correct answer to each question.
Pets are kept by many people for pleasure or for pastime. They keep and nurse them at home as a
hobby. Some people have special breeds of dogs or cats as pets while others rear birds in cages.
In keeping pets we need to take special care. We have to feed them with the right food at proper
times. Any failure on our part could endanger their life. We have to protect them from their enemies.
Furthermore, the pets and their cages have to be kept clean. This is to prevent the attack of any diseases from
unhygienic surroundings.
If we understand their habits, it is easy for us to handle them. Keeping pets gives us an opportunity to
be kind to these creatures. We might learn more about the life and needs of pets. Those who keep birds like
parrots are likely to read books about rearing them. Such people do not hesitate to spend money, time and
energy for the well-being of their pets.
1. What is the main idea of the passage?
A. Ways of feeding pets B. Keeping pets as a hobby
C. Preventing attack of diseases D. Reading books about rearing birds
2. Which of the following is not true, according to the passage?
A. Keeping dogs and cats is a hobby
B. Feeding pets properly is important
C. We should know about our pets’ life and needs
D. If we fail to take care of pets, their life is dangerous
3. Our pets may get sick if their surroundings are ………….
A. attacked B. prevented C. not clean D. not special
4. Keeping pets is easy if we………….
A. understand their habits B. give them an opportunity
C. read books about them D. spend more money for them
5. The word ‘rearing’ in the passage is closest in meaning to …………..
A. giving food to B. taking care of
C. looking for D. learning about
VI. Choose the underlined word or phrase that needs correcting.
1. Meat must keep in a refrigerator or it will spoil.
A B C D
2. There is a cat sit in the middle of the road.
A B C D
3. Because of tired , we could not go to school yesterday
A B C D
4. I found it wonderfully to travel abroad.
A B C D
5. We object to people to make private calls on the office phone.
A B C D
VII. Choose the most suitable response to the following exchanges.
1. Daisy: "What an attractive hair style you have got, Mary!" Mary: "……………"
A. Thank you for your compliment! B. I don’t like your sayings
C. You are telling a lie D. Thank you very much! I am afraid.
2. Daisy: "You must be Jane’s sister. Glad to meet you.” Mary: "…………..”
A. I am, either B. So I am. I’m glad C. Me too D. What do you do
3. Ann: "…………..?" Bill: "About ten miles before we met him."
A. How long did he drive B. How often did he drive
43
C. How far did he drive D. How fast did he drive
4. Tom: " Sorry, I forgot to phone you last night." Mary: "……………"
A. You were absent-minded B. Oh. Poor me!
C. I have nothing to tell you D. Never mind!
5. Anna: "………….." Bob: "I’m glad you enjoyed it. Hope to see you again soon."
A. My uncle is coming soon. Would you like to come to our dinner?
B. I highly appreciate for the houses you’ve rebuilt for our village
C. I don’t mind going to the party sometimes but when I know everybody there then I enjoy it
D. That’s a great dinner. Thanks a lot. You’ll have to come to our place sometime.
VIII. Choose the sentence that is closest in meaning to each of the fllowing questions.
1. Start at once or you will be late.
A. If you started at once, you wouldn’t be late
B. Unless you start immediately, you won’t be late
C. You would be late if you didn’t start at once
D. If you don’t start immediately, you will be late
2. "You have just got a promotion, haven’t you? Congratulations! " Peter said to his friend.
A. Peter dream of getting promotion
B. Peter congratulated his friend on getting a promotion
C. Peter asked his friend on getting a promotion
D. Peter told his friend if his friend getting a promotion
3. No one but the seven-year-old boy saw the accident.
A. The seven-year-old boy saw no one in the accident
B. Only the seven-year-old boy saw the accident
C. No one in the accident saw the seven-years old boy
D. No one at all saw the seven-years-old’s accident
4. She smokes all the time. I can’t stand that about her.
A. I can’t put up with what she smokes
B. What I can’t stand about her is the way she smokes all the time
C. That she smokes all the time I can’t stand
D. I can’t bear her smoke
5. "I’ll pay back the money, Gloria, " said Ivan.
A. Ivan offered to pay Gloria the money back
B. Ivan apologized to Gloria for borrowing her money
C. Ivan suggested paying back the money to Gloria
D. Ivan promised to pay back Gloria’s money
IX. Mark the letter A, B, C, or D to indicate the sentence that best uses the words to form sentences in
the following questions.
1. last time/I/go/swim/when/we/be/Spain.
A. Last time I went swimming is when we were in Spain.
B. The last time I went for swim was when we was in Spain.
C. Last time I went to swim was when we were in Spain.
D. The last time I went swimming was when we were in Spain.
2. bad/drive/often/cause/accident.
A. Bad drivers often causes lots of accident.
B. Bad driving often causes a lot of accidents.
C. Driving bad often cause accidents a lot.
D. Badly drive causes often a lot of accident.
3. our grandparents/visit/Ha Long Bay/last summer.
44
A. Our grandparents paid a visit to Ha Long Bay last summer
B. Our grandparents made a visit at Ha Long Bay last summer
C. Our grandparents took visit in Ha Long Bay last summer
D. Our grandparents had a visit for Ha Long Bay last summer
4. remember/turn off/light/before/go out.
A. Remember to turn off the light before you go out
B. Remember turning off the light before to go out
C. Remember to turning off the light before go out
D. Remember turn off the light before going out
5. it/necessary/everybody/clean up/ neighborhood/once/week.
A. It’s necessary for everybody clean up their neighborhood once every week.
B. It’s necessary to everybody to clean up their neighborhood once the week.
C. It’s necessary for everybody to clean up their neighborhood once a week.
D. It’s necessary for everybody cleaning up their neighborhood once a week.

45
PRACTICE TEST 3
I. Choose the word which is pronounced differently from the others.
1. A. chemistry B. chair C. couch D. children
2. A. student B. bus C. duty D. unit
3. A. house B. about C. country D. our
4. A. clothes B. watches C. benches D. classes
5. A. clock B. come C. class D. city
II. Choose the word that is different from the rest in each group.
1. A. teacher B. brother C. farmer D. doctor
2. A. watches B. brushes C. teaches D. goes
3. A. meat B. fish C. beef D. tea
4. A. knee B. gray C. nose D. toe
5. A. have B. eat C. is D. play
III. Read the following passage and choose the correct answer to the questions.
There are different ways to keep fit. One of the most important ways is by exercising. Exercising
must be done regularly. The amount of exercise done each time is dependent on the free time that a person
has. Exercises can be done from twenty minutes to an hour. Sometimes, daily five-minute exercises are just
as effective.
Exercising can take place both indoors and outdoors. Those who exercise indoors often do so in the
comfort of gymnasiums. They train on equipment that has been specifically designed for the different parts of
the body. Sometimes, games can also be played indoors. Badminton courts and even some swimming pools
are found indoors.
There are many different types of outdoor exercises. These include tennis, cycling and even rock
climbing. Most people enjoy outdoor activities because they like being in the sun or are attracted to its sense
of peace and quiet. Yet there are others who like the wind blowing against them. Some of these activities are
only suitable for people who love adventure.
The type of exercise a person takes up often depends on his lifestyle and personality. It is important
that a person find an exercise that is suitable for him.
1. According to the passage, why do some people spend less time exercising?
A. Five minutes is effective for them B. They do not have much free tim
C. Gymnasiums are expensive D. They keep fit not by exercising
2. Which of the following is not a place for indoor exercise as mentioned in the passage?
A. Tennis courts B. Badminton courts
C. Swimming pools D. Gymnasiums
3. Which statement is TRUE according to the passage?
A. People should exercise from time to time
B. Outdoor exercises are more popular than indoor exercises
C. Rock climbing is suitable for more adventurous people
D. Facilities in gymnasiums are not effective
4. The highlighted phrase “do so” in the passage refers to.
A. take place B. exercise indoors
C. exercise outdoors D. train on equipment
5. Which of the following influences the particular type of exercise a person takes up?
A. Health B. Income C. Occupation D. Lifestyle
IV. Complete the following passage with these words
fire works parades festivities dancing costumes
“Carnaval” is the most famous holiday in Brazil, and the biggest carnival in the world. Except for industrial
production, retail establishments such as malls, and carnival- related businesses, the country stops completely
46
for almost a week and (1)…………….take place, day and night, mainly in coastal cities. There are lots of
street (2) …………….with amazingly decorated lorries (called floats) and people dressed in colourful (3)
…………….. Everywhere you go there are samba bands playing and people (4) …………….. Right through
the night the festivities continue with more dancing and music and (5) …………….displays which light up
the night sky. The carnival attracts millions and millions of people, including foreigners, every year.
V. Read the passage and choose one word or phrase to fill in each blank
In the past, it was common for three or more (1) …………….to live together in the same roof.
Nowadays, most elderly people live (2) …………….their own. They generally stay in contact with their
children or grandchildren, but might live in the different part of the country. People also live longer, often 20
years after they have (3) …………….from their job. Modern American culture tends to value youth rather
than age. This creates an interesting challenge for older people and for the country.
American families also have troubles. They are the divorce rate, the fact that working mothers might
have (4) ……………. time with their children, and the problems that parents and children sometimes
misunderstand each other. (5) ……………., family is still at the centre of most people’s lives.
1. A. generations B. peoples C. classes D. nations
2. A. at B. on C. with D. by
3. A. stopped B. saved C. retired D. given up
4. A. a few B. fewer C. less D. much
5. A. Although B. Otherwise C. However D. Therefore
VI. Choose the sentence that is closest in meaning to the sentence given in each of the following
questions.
1. “You damaged my laptop, Tom!” said John.
A. John accused Tom of damaging his laptop
B. John accused to Tom for damaging his laptop
C. John accused with Tom to have damaged his laptop
D. John accused Tom to damage his laptop
2. This is the first time she has attended such an enjoyable birthday party.
A. She has never attended such an enjoyable birthday party before
B. Her attendance at the first birthday party was enjoyable
C. She had the first enjoyable birthday party
D. She has ever attended such an enjoyable birthday party before
3. John is studying hard. He doesn’t want to fail the next exam.
A. John is studying hard in order to not to fail the next exam
B. John is studying hard in order that he not fail the next exam
C. John is studying hard in order not to fail the next exam
D. John is studying hard so as to fail the next exam
4. Father has been working all day. He must be tired now.
A. Father must work all day and tired now
B. I think Father was tired all day working
C. I’m sure that Father is tired after working all day
D. Father thinks he is tired now because he has been working all day
5. The test we did last time was more difficult than this one.
A. The test we have done this time is not difficult at all
B. This test is not as difficult as the one we did last time
C. We did an easy test last time and a difficult one this time
D. This time we have to do the most difficult test of all
VII. Choose the best answer
1. Could you move your car? You’re…………..the way of the entrance!
47
A. on B. in C. at D. to
2. Three of the words below go together with “HEAD” to make new words. Which word does NOT?
A. ache B. man C. master D. line
3. What does the following mobile text (cell phone SMS) abbreviation mean: LOL?
A. I am crying B. Laughing out loud
C. I’m going to be late D. I love you lots
4. “Did you know Jim’s getting married next week?” - “…………..Who to?”
A. Really? B. Is he? C. Never D. All are correct
5. - How do you say the following time? - 2:57
A. It’s nearly 3 o’clock B. It’s just before 3 o’clock
C. It’s just coming up to 3 o’clock D. All are correct
6. Which option is NOT correct? “………….., I can’t hear myself think.”
A. Turn it down B. Turn the music down
C. Turn down the music D. Turn down it
7. Which of the following ways of saying 6x3 is NOT correct?
A. Six times three B. Six multiplied by three
C. Six add three D. Six by three
8. “When can you get it all done? ” - “……………”
A. “How does next Monday sound? B. Two hours ago
C. I used to do it on Monday D. How much time do I need?
9. Which of the following is the function of this sentence: “I’d love to come, but I’m already going out that
evening.”
A. Offering to do something B. Apologizing
C. Declining an invitation D. Giving advice
10. “Jane didn’t look well today, did she? “ - “…………….”
A. I am afraid not B. No, she seemed ill
C. Yes, she seemed sick D. A and B
11. “ I am very much obliged to you for helping my family.” - “…………….”
A. My pleasure B. That’s all right
C. You’re welcome D. A & C are correct
12. Which of these is the opposite of straight?
A. beautiful B. crooked C. definite D. self - conscious
13. “Would you mind turning your stereo down?” - “……………”
A. I’m really sorry! I’m not paying attention
B. Oh, I’m sorry! I didn’t realize that
C. No, I don’t
D. Yes, I do
14. Which of these is the synonym of necessary?
A. essential B. difficult C. expensive D. wasteful
15. Where there is a will, there is a…………..
A. road B. way C. money D.
happiness
VIII. Choose the most suitable response to complete each of the following exchanges.
1. Mike: “………….” Maria: “Thank you. We are proud of him.”
A. This gift is for your kid B. Can we take a photo of your child?
C. Your kid is naughty D. Your child is just adorable!
2. John: “I’ve passed my final exam.” Tom: “…………….”
A. Good luck B. Congratulations!
48
C. That’s a good idea D. It’s nice of you to say so
3. Tim: “I don’t think I can do this one.” John: “…………….”
A. Not at all B. Oh, come on! Have a go! C. No way D. I hope not
4. George and Frankie are talking about their hobbies.
George: “In my opinion, action films are exciting.” Frankie: “…………….”
A. There’s no doubt about it B. Yes, you can do it
C. What an opinion! D. Your opinion is exactly
5. Thomas and Peter are meeting after a long time.
Thomas: “How have you been recently?” Peter: “…………….”
A. Pretty busy, I think B. I am working here
C. By car, usually D. I am going on holiday next week

49
PRACTICE TEST 4
I. Match the halves to make sentences.
1. Our Internet connection has been cut…………..
2. The telephones will photograph distant galaxies…………..
3. The distance from the Earth to the Sun is,…………..
4. Be careful! You might give yourself…………..
5. You may get connected to him on the facebook…………..
A. an electric shock!
B. but I can’t be sure.
C. off, so I’m afraid you’ll have to wait until tomorrow.
D. in an attempt to understand their past.
E. on average, about 149 million kilometres.
II. Choose one word that has the underlined part pronounced differently from the others
1. A. facility B. society C. necessity D. economy
2. A. stamped B. indulged C. accomplished D. practised
3. A. friends B. clubs C. tunes D. lamps
4. A. whistle B. battle C. gentle D. little
5. A. food B. school C. root D. wood
III. Choose the word which best completes each of the following sentences.
1. Going into the city by the expressway takes ………….. longer than the local roads because of heavy
traffic.
A. even B. so C. too D. more
2. Every employee is required to attend…………..one of the training workshops each month.
A. so that B. at least C. only if D. as much
3. Several important methods were…………..for preserving food in the nineteenth century.
A. seen B. covered C. prepared D. invented
4. What does the poster say?
Mom,
Anna wants you to phone her. I said you’d be back very late but didn’t mind. I’ve gonne out. Back about
midnight.
Pattrick

A. Pattrick wanted to go out with Anna


B. Pattrick’s mother will arrive home late
C. Anna didn’t agree with Pattrick’s suggestion
D. Pattrick will be home tomorrow night
5. It is said that a person’s personality is more important than……………appearance.
A. your B. his C. its D. my
6. People play golf on a……………
A. court B. pitch C. ball D. course

7.……………energy is the energy that we get from the sun.


A. Wind B. Tidal C. Hydro D. Solar
8. We finally……………our teacher to go on a camping trip with us.
A. persuaded B. argued C. reminded D. made
9. What does the poster say?
ALPHA
FITNESS & SPA
50
50% OFF
for MIDDLE-AGED WOMEN

A. 50% of women in the city come here


B. This place is for middle-aged women only
C. Ladies in their forties pay half price
D. 50% of the customers are middle-aged women
10. I ’ve to see the dentist for a check-up tomorrow; I just hope I don’t need to…………..
A. have anything done B. be done something to
C. let anything be done D. make something done
IV. Read the passage and fill the suitable word in the blank:
We are destroying the Earth. The seas and rivers are (1) …………..dirty to swim in. There is too (2)
…………..smoke in the air in many cities in the world. We are cutting (3) …………..too many trees. We (4)
…………..burning the forests. We are also destroying (5) …………..of plants and (6) …………..Many areas
(7) …………..Earth are wasteland. Farmers in many parts of the world cannot (8)…………..enough to eat.
In some countries (9) …………..is too little rice. The Earth is in (10) …………... Save the Earth!
V. Read the following story and choose the best answer for each question.
Once upon a time, there lived in Morocco one of the richest men on earth. He was King Jodas. He loved gold
more than anything else in the world except his lovely, young daughter with silky, black hair and sparkling
eyes. Whenever King Jodas walked among his fruit trees and flower bushes, he wished he could turn them all
into gold.
Early one morning, a god called Terrus came to visit King Jodas in his palace. He came to reward
King Jodas for doing him a favour. Terrus offered to grant King Jodas whatever he wanted on earth. The
King immediately asked that everything he touched be turned into gold. Terrus unwillingly granted King
Jodas his wish and vanished.
Everything was fine till one fateful day. The King wept aloud in despair as he hugged a golden
figurine he loved so much. He did not mean to turn her into gold. He begged Terrus to take away the curse of
the golden touch.
1. Which of the following was King Jodas’ most favoured?
A. gold B. fruit trees C. flower bushes D. his daughter
2. King Jodas’ love for gold tells me that he was…………..
A. greedy B. selfish C. rich D. cruel
3. Terms was in Morocco because he …………...
A. wanted to visit King Jodas
B. wanted to return King Jodas a favour
C. decided to marry King Jodas’ daughter
D. heard about King Jodas’ wish
4. King Jodas cried because…………..
A. Terr us was not going to grant him his wish
B. Terms vanished
C. his daughter was turned into gold
D. Terrus took away his wish
5. Which of the following titles is the most suitable for the story?
A. The golden king B. A reward
C. The king and his daughter D. The golden touch
VI. Rearrange the given words to make meaningful sentences.
1. I/first/ meals/go/ate / for / night / in/usually/but now / the/hotel / my / out/I / the/ .
………………………………………………………………………………………………………………..
51
2. I/get/all/have/of/taken/I/photos/show/back/when/you/the/will/I/.
………………………………………………………………………………………………………………..
3. Today/I/ an / gallery/been/to/with/exciting/beautiful / art / pictures/have /.
………………………………………………………………………………………………………………..
4. I/selling/discovered/lots / stalls / everything/a/of/with / market/.
………………………………………………………………………………………………………………..
5. The / meal / are/less/great /I/can / and / for / restaurants/£25/than/a/get /.
………………………………………………………………………………………………………………..
VII. Each line in the following paragraph has ONE mistake, find out, underline and correct it.
The culture of Vietnam are one of the oldest in the Southeast Asia region. Although Vietnam lies
geographical in the Southeast Asia, long periods of Chinese domination and influence has resulted from the
emergence of East Asian characteristics in Vietnamese culture, and Vietnam is said to be a part of the East
Asian cultural sphere, knowing widely as Chinese cultural sphere. Despite considerable foreign influent,
Vietnamese people have managed to retain much distinct customs. While Chinese culture has the larger
influence on traditional Vietnamese culture, there is also a much smaller influence from the Cham and later
Western cultures, most of French, Russia and the United States. In terms of prehistory, most Vietnam
historians consider the ancient Dong Son Culture to be one of the defining aspect of early Vietnamese
civilization. Most person live in or near the densely populous Red River or Mekong deltas.
0. is 1……… 2.……… 3.……… 4.……… 5.………
6.……7……… 8.……… 9.……… 10.………
VIII. Find a suitable preposition and fill in the blank.
1. If you have time, come to visit our town. I’ll show you……….
2. My hotel was amazing and it lived up………all my expectations.
3. Finally, the riot was brought………control.
4. We should provide any child ……… a chance to go to school.
5. The doctors say it takes him a long time to get………the shock.

52
PRACTICE TEST 5
I. Choose the word whose stress pattern is different
1. A. weather B. idea C. winter D. movies
2. A. morning B. number C. routine D. ruler
3. A. dangerous B. accident C. telephone D. engineer
II. Choose one word that has the underlined part pronounced differently from the others
1. A. invitation B. intersection C. station D. question
2. A. lives B. misses C. languages D. watches
III. Read the passage and choose the best answer to each question.
The last two decades have seen enormous changes in the way people’s lives are affected by IT.
Twenty years ago, few people had access to a computer while today most people use them at work, home or
school and the use of e- mail and the Internet is an everyday event.
These developments have brought many benefits to our lives. E-mail makes communication much
easier and more immediate. This has numerous benefits for business, commerce, and education. The World
Wide Web means that information on every conceivable subject is now available to us. Clearly, for many
people, this has made life much easier and more convenient.
However, not all the effects of the new technology have been beneficial. Many people feel that the
widespread use of e-mail is destroying traditional forms of communication such as letter writing, telephone
and face-to-face conversation. In addition, the huge size of the Web means it is almost impossible to control
and regulate. This has led to many concerns regarding children accessing unsuitable websites and viruses.
Unfortunately, this kind of problem might even get worse in the future at least until more regulated systems
are set up.
In conclusion, developments in IT have brought many benefits, yet developments relating to new
technology are likely to produce many negative effects in the future, and they must be addressed if we are to
avoid damaging impacts to individuals and society.
1. According to the passage, the World Wide Web………….
A. brings only benefits to people
B. has both advantages and disadvantages
C. is difficult for children to get access to
D. causes a busy and exciting life
2. The writer of the passage believes that e-mail…………..
A. reduces face -to -face contact B. is used only in business
C. can be checked quickly and easily D. takes a lot of time to transmit
3. It is believed that future IT developments will probably be………….
A. more rapid B. more negative C. more positive D. unexpected
4. What does the word “they” in the last paragraph refer to?
A. Benefits B. Developments
C. Negative effects D. Damaging impacts
5. It can be inferred from the passage that children’s use of the computer should be………….
A. increased B. reduced C. stopped D. controlled
IV. Fill in the blanks with the words/phrases from the box. There are more words/ phrases than
needed.
hobby age disabled cooking constructed playwright
speciality comedies poet best sellers artists voluntary service

1. Is your best friend the same……………as yours?


2. After retiring, my grandmother became involved in……………in the local community.
3. I prefer to see……………at the theatre because they make me laugh.
53
4. My brother and I have the same……………. We like to collect stickers.
5. Her mother is a……………person. I sometimes help her with the housework.
6. Moliere is a famous French……………. One of his most popular plays is The Miser.
7. The Imperial Academy - the first university in Viet Nam - was……………under Emperor Ly Nhan Tong
in 1076.
8. John Keats was a British……………. His most famous poem is To Autumn.
9. Airport bookshops often sell……………, the most famous and widely read books.
10. Five - coloured sticky rice is a……………in the mountainous regions of North Viet Nam.
V. Choose the underlined part which needs correcting in each of the following questions.
1. Not only the athlete but also his wife are going to the party tonight.
A. are B. but C. Not only D. athlete
2. Could you mind telling me the way to the nearest restaurant?
A. Could B. nearest C. telling me D. to
3. They asked me what did happen last night, but I was unable to tell them.
A. but B. asked
C. to tell D. what did happen
4. There were so much books in the library that I didn’t know which one to choose.
A. that B. There were C. much D. to choose
5. The Earth is the only planet with a large number of oxygen in its atmosphere.
A. oxygen B. the C. number D. its
VI. Choose the best answer
1. The relationship between fast food and child obesity…………..very close indeed.
A. are B. is C. to be D. were
2. There are only two things that one should do…………..a healthy and fit body: eating sensibly and
exercising well.
A. keeping B. to keep C. for keeping D. keep
3. They are hanging their clothes to make them…………...
A. to dry B. dried C. dry D. drying
4. Dairy products…………..butter, cheese and milk.
A. are including B. included C. includes D. include
5. You shouldn’t eat too much sugar and don’t forget about exercise,…………...
A. too B. so C. either D. neither
6. Which sentence is correct? .…………..
A. Our school is having the celebration anniversary on 5 September
B. Our school is having the celebration of anniversary on 5 September
C. Our school is having the anniversary celebration on 5 September
D. Our school is having the celebration’s anniversary on 5 September
7. He came into the room,…………..the light and…………..the door behind him
A. turned on / closing B. turning on / closing
C. turned on / closed D. turning on / closed
8. We…………..stop when the light is red.
A. can’t B. can C. mustn’t D. must
9. What is your favourite food?
A. I favourite chicken B. My favourite food chicken
C. Chicked favourite me D. I like chicken best
10. How many minutes are there in…………..hour?
A. a B. an C. the D. these
VII. Choose the sentence that best combines each pair of sentences in the following questions.
54
1. I think I should have cooked more food. There’s nothing left now.
A. I should not have cooked so much food
B. I regrets cooking too much food now
C. I did not cook much food and I think it is ok now
D. I did not cook much food and I think it was a mistake
2. You need not have washed the sheets. The hotel staff does the cleaning.
A. It is good that you washed the sheets
B. It was not necessary to wash the sheets, even though you did
C. It was necessary to wash the sheets, but you did not do it
D. It was necessary to wash the sheets, and you did not do it
3. What has happened? You look as if you have been in the wars.
A. You look like an old soldier
B. You are wearing many medals
C. You look as though something unpleasant has happened to you
D. You look as though you have been fighting
4. The agreement ended six-month negotiation. It was signed yesterday.
A. The agreement which ends six-month negotiation was signed yesterday
B. The agreement which was signed yesterday lasted six months
C. The negotiation which lasted six months was signed yesterday
D. The agreement which was signed yesterday ended six-month negotiation
5. “ Cigarette?”, he asked. “ No, thanks.” I said.
A. He asked for a cigarette ,and I immediately refused
B. He mentioned a cigarette, so I thanked him
C. He offered me a cigarette, but I promptly declined
D. He asked if I was smoking, and I denied at once
VIII. Give the correct form of the words in brackets
1. Going to work abroad can prove to be quite a (VALUE)…………….experience.
2. The wheel might seem to be the most (SIMPLICITY)…………….invention.
3. (SCIENCE) ……………., there are no such so-called ghosts.
4. She should practice speaking English regularly in order to (GOOD)…………….her speaking skill.
5. There is a (VARY)…………….of fruits in Viet Nam.
IX. Supply the correct form of the verb in brackets
1. Give me a word (begin)…………….with “S”.
2. I really love Ha long Bay. I (visit)…………….it twice when I was small.
3. She gets used to (live)…………….on her own.
4. He suggested that we (leave)…………….the following morning.
5. I wish I (have)…………….time to go on holiday now.

55
PRACTICE TEST 6
I. Odd one out
1. A. Holland B. Scottish C. Australia D. Korea
2. A. New York B. Big Ben C. Ha Long Bay D. Sahara
3. A. post office B. grocery C. place D. hospital
4. A. rooster B. baker C. runner D. writer
5. A. important B. success C. peaceful D. necessary
II. Choose the word whose underlined part is pronounced differently from that of the others
1. A. couches B. houses C. shoulders D. buses
2. A. about B. country C. surround D. counting
3. A. cities B. centre C. celebrate D. candidate
4. A. chemistry B. church C. check D. chin
5. A. activity B. creative C. continuous D. international
III. Choose the best answer to fill in the blank.
1. I can’t go to your party now. I……………after I finish my housework.
A. am coming B. could come C. will come D. come
2. You should bring a……………with you when you walk into a forest because you may lose your way.
A. compass B. torch C. phone D. camera
3. When New Year comes, people in different countries always wear their ……………clothes.
A. traditional B. normal C. ordinary D. tradition
4. Children in Viet Nam always……………lucky money from their parents or relatives on New Year’s Day.
A. bring B. get C. give D. take
5. Life in the city is more……………than that in the countryside.
A. busier B. quieter C. convenient D. boring
6. Could you……………me the……………to the nearest hotel?
A. tell/road B. show/way C. show/road D. tell/street
7. When the New Year comes, people always wear traditional Korean dress and wish each other a…………
life.
A. healthy B. unhappy C. unhealthy D. health
8. January 1st is a day when people in Europe and America……………new year.
A. remember B. see C. celebrate D. spend
9. Which country is……………, Viet Nam or Japan?
A. large B. largest C. larger D. the largest
10. You can buy fruit and vegetables in a…………….
A. post office B. cinema C. hospital D. grocery
11. They are……………that their son won the championship.
A. pleasing B. bad C. delighted D. interesting
12. Do you mind if I……………your car to work?
A. drive B. driving C. to drive D. drove
13. There are a lot of amusement parks……………our neighborhood.
A. in B. on C. at D. in front of
14. You can improve your health by doing……………activities.
A. outside B. inside C. indoor D. outdoor
15. There is a theatre……………Nguyen Du Street. First go straight, then……………left.
A. on/turn B. in/take C. by/take D. at/turn
IV. Identify the mistake in each sentence
1. How many childs does your mother have ?
A B C D
56
2. Vy is really excited with her first day at school.
A B C D
3. We have geography every Wednesdays.
A B C D
4. If you climb the mountain, you will sees a lot of flowers .
A B C D
5. There are five bridge across Han River in Da Nang.
A B C D
V. Read the text. Some of the lines are incorrect. Find them and write the correct answers in the space
provided. (If a line is correct, put a tick √).
Miss Lien live in a small house on Hanoi. She teaches English at Line 1 live → lives
a school there. She usually is breakfast in seven in the morning Line 2 …………..
and she has dinner at twelve o’clock in the canteen of the school. Line 3 …………..
She teaches his students in the morning. She teaches them Line 4 …………..
dialogues on Wednesdays to Fridays. On Mondays, she teach them Line 5 …………..
grammar. On the evening, she usually stays at home and listens Line 6 …………..
books. She sometimes goes to the movie theatre. She always go Line 7 …………..
to bed at ten o’clock. Line 8 …………..

VI. Read the passage and decide which answer A, B, C or D best fits each space
Spring is a time when there are many festivals in our country. Among them, Hoa Ban Festival is the most
beautiful and interesting and it takes place in Lai Chau. It is typical of the (1)………….life of the Thai
people. In the second lunar month of the year, when it (2)………….warmer and Hoa ban - a kind of beautiful
flower in the northwest mountainous area - blossoms, Hoa Ban Festival is (3)………….. This is a great time
for everyone, (4)………….for boys and girls. The boy picks the most beautiful flower and gives it to his
girlfriend. This is not only a time for love but also for the Thai people to (5)………….for good crops, for
happiness, and express their special thanks to the Gods and ancestors. The festival is always full of songs and
prayers.
1. A. cultural B. festival C. minor D. custom
2. A. gets B. goes C. runs D. comes
3. A. observed B. seen C. celebrated D. arranged
4. A. largely B. especially C. typically D. generally
5. A. look B. ask C. call D. pray
VII. Read the following text and choose the best answer for the questions below.
Sharks
Mention the word shark to someone on the beach and you are bound to get a reaction. Sharks have been the
most feared predator of the ocean. Sharks are fish. They have been around since the time of dinosaurs. They
can be found in oceans all over the world, and they have also been found in some rivers and lakes.
One difference between most other fish and a shark is that most fish have bones and a shark has
cartilage. Cartilage is tough, but it is not as strong as bone. Another difference is that the shark is only able to
swim forward, while most fish can swim forwards and backwards. Fish also generally have slippery scales,
while a shark has rough scales that feel like sandpaper.
There are many different types of shark. In fact, there are about 368 different types. The differences
include color, habitat, behavior, and size. Sharks are also known for their very sharp teeth. They do not use
their teeth to chew their food. Instead they swallow large chunks of food. They have five rows of teeth. If a
tooth is broken, it is replaced by another tooth.
Sharks are carnivores, meaning that they eat meat. Sharks do not normally attack people. It seems like
sharks attack people a lot, but the chances of getting stung by a bee or getting hit by lightning are greater.
57
Even so, experts say it is important to keep an eye out for sharks when swimming in the ocean.
1. Sharks are greatly feared by humans because they…………..
A. are good swimmers B. are carnivores
C. are the right size D. eat only plants
2. According to the passage, how are sharks different from most other fish?
A. They do not have scales
B. They are slow moving
C. They are camouflaged
D. They are not able to swim backwards
3. Why did the author include the first paragraph?
A. To generally introduce sharks
B. To generate questions about sharks
C. To clear up misconceptions about sharks
D. To identify the food eaten by sharks
4. Sharks have very sharp teeth. If a tooth is broken, it is replaced…………...
A. by no teeth B. by another tooth
C. by other teeth D. by every tooth
5. Sharks are carnivores, meaning that …………...
A. they eat meat B. they eat other fish only
C. they eat plants D. they never eat the others
VIII. Choose the correct sentence among A, B, C or D which has the same meaning as the given one
1. I walk to school in 15 minutes every morning.
A. It takes me fifteen minutes to walk to school
B. I need less than 10 minutes to walk to school
C. It takes I a quarter to walk to school
D. It’s a 10 minute walk from my house to school
2. The recess lasts thirty minutes.
A. It is a thirty minutes recess B. It is a recess thirty minutes
C. It is a thirty- minute recess D. It is a recess thirty minute
3. What’s your favorite subject?
A. What subject do you like best? B. What subjects do you have?
C. What subject do you want? D. What subject do you need?
4. My apartment is behind a supermarket.
A. There is a supermarket behind my apartment
B. The supermarket is in front of my apartment
C. The supermarket isn’t in front of my apartment
D. There isn’t a supermarket in front of my apartment
5. How many books does the Library of Congress have?
A. How many books the Library of Congress has?
B. How many books there are in the Library of Congress
C. How many books are there in the Library of Congress?
D. All are correct
IX. Complete the second sentence so that it has the similar meaning to the first one.
1. Huong is sitting to the left of Nam in the classroom at the moment.
Nam is………………………………………………………………………………………………
2. My father’s working room has a desk, an armchair, two computers and a printer.
There ……………………………………………………………………………………………….
3. His computer is on the table.
58
On …………………………………………………………………………………………………
4. My house is next to a big supermarket.
There is ……………………………………………………………………………………………
5. Jane is intelligent, kind-hearted and helpful. Many people in my class love her.
Because ……………………………………………………………………………………………

59
PRACTICE TEST 7
I. Choose the word that is stressed differently from that of the other words.
1. A. furniture B. apricot C. relative D. tradition
2. A. Chinese B. Spanish C. Scottish D. English
3. A. exciting B. delicious C. beautiful D. reliable
4. A. celebrate B. decorate C. fascinate D. unfortunate
5. A. religion B. neighborhood C. apartment D. eraser
II. Choose the word whose underlined part is pronounced differently from that of the others
1. A. overseas B. compass C. judo D. homework
2. A. students B. phones C. bedrooms D. gardens
3. A. love B. above C. Monday D. notebook
4. A. school B. stomach C. schedule D. chimpanzee
5. A. bowl B. show C. now D. snow
III. Choose the best answer to fill in the blank.
1. The book is…………the chair. Please put it on the table.
A. in B. at C. under D. from
2. I am so…………, Mum. Can I eat this cake?
A. sweet B. thirsty C. hungry D. angry
3. The championships are…………every two years.
A. taken B. held C. had D. made
4. Is Peter going to…………the meal?
A. feed B. pay for C. pay D. bite
5. Which…………do you speak ?
A. language B. nationality C. country D. nation
6. Don’t be too hasty and draw the wrong…………
A. conclusion B. judgment C. opinion D. outcome
7. In the middle of all the panic, Susan…………phoned the police.
A. peacefully B. mildly C. silently D. calmly
8. I…………like some orange juice.
A. can B. would C. will D. could
9. …………do you go to the movies ?…………Twice a month.
A. How many B. How often C. How long D. How much
10. He…………his wife and children and went abroad.
A. won B. demanded C. deserted D. gained
11.…………me to buy some fruits when I go downtown.
A. Remind B. Mention C. Make D. Remember
12. They took the baby to church to be…………
A. baptism B. christening C. named D. christened
13. You’ll get a better…………of exchange at a bank.
A. rate B. charge C. value D. worth
14.…………going to HaLong Bav?
A. Why don’t we B. What are
C. A and B are right D. How about
15. You…………..go to the dentist’s before your toothache gets worse.
A. ought B. rather C. better D. ought to
16. Henry drew all his money…………….the bank before he left.
A. to B. out of C. of D. off
17. He used to find…………….with his son’s friends.
60
A. mistake B. blame C. lack D. fault
18. Paul’s…………….in his work is very amazing.
A. interest B. thought C. attention D. concern
19. He who…………….the piper calls the tune.
A. makes B. gives C. pays D. does
20. He acted promptlv. He…………….the child from drowning.
A. brought B. saved C. took D. kept
IV. Identify the mistake in each sentence
1. There are a living room, three bedrooms , a bathroom and two toilets in my house.
A B C D
2. I like watching TV. There is a good film in TV tonight.
A B C D
3. There are many good restaurant and amusement parks in our neighborhood.
A B C D
4. Tom is a talkative student. He never talks in class.
AB C D
5. I’m going to HoanKiem Lake watching fireworks tonight.
A B C D
V. Read the passage and decide which answer A, B, C or D best fits each space
The cottontail rabbit gets its name (1)…………..its tail looks like a ball of cotton. The color of the cottontail
rabbit is reddish brown (2)…………..a white tail. The adult grows to be about 12 to 18 inches long and (3)
…………..anywhere from two to four pounds. The cottontail rabbit likes to live in the brush in order to hide.
It makes its home in a burrow near prickly bushes. This rabbit is unaffected by the prickly bushes, but living
near these prickly bushes deters some predators (4)…………..attacking the rabbit. Coyotes, red foxes,hawks,
and owls are all predators of the cottontail rabbit. A cottontail will have multiple litters of babies each year. It
is not uncommon for a rabbit (5)…………..many babies. The babies are born without fur and they cannot
see. After about a week, the fur begins to grow in. The mother rabbit builds a nest out of grass, fur, and hay
to keep the babies warm.
1. A. because B. so C. but D. because of
2. A. like B. unlike C. with D. without
3. A. weigh B. weighs C. weighing D. weight
4. A. about B. for C. of D. from
5. A. to have B. have C. has D. having
VI. Read the following text and choose the best answer for the questions below.
Franklin D. Roosevelt
On January 30, 1882, Franklin D. Roosevelt (F.D.R.) was born. He would go on to become the 32nd
president of the United States. At an early age, he was taught by his parents and private tutors. In 1896, he
attended a prep school. He went to college to study history and the law.
F.D.R. married his cousin, Anna Eleanor Roosevelt. Together they had six children. Five of them
lived. By this time, Franklin was very involved in politics and was reelected to the state senate of New York.
He became more and more involved in politics. He was nominated as vice president, but he did not win the
election and went back to private life.
While vacationing on Campobello Island, Franklin became sick with polio. This disease took away
the use of his legs, and he would never walk after that. Eleanor encouraged him and supported him in his
efforts to enter politics again. This time he became governor of New York. After being reelected, Franklin
ran for president. F.D.R. was sworn in as president in January 1933. While president, he worked hard to help
overcome the Great Depression. He also led the U.S. into the Second World War. Eleanor was a great
support to Franklin. She also played a great role as first lady of the United States.
61
1. At an early age, Franklin D. Roosevelt (F.D.R.) was taught by………….
A. his parent and-Private tutors B. his parents and teachers at school
C. his parents and private tutors D. his teachers and private tutors
2. What does the word nominated mean in this passage?
A. requested B. ordered C. selected D. pushed
3. After reading the passage, what is a word that could be used to describe F.D.R ?
A. studious B. rugged C. talented D. determined
4. F.D.R.’s time as president could be described as…………..
A. successful and accomplished B. typical and normal
C. supportive yet unsuccessful D. unhappy and unsettled
5. Eleanor played…………..
A. a small role as first lady of the United States
B. a minor part as first lady of the United States
C. an unimportant role as first lady of the United States
D. a great part as first lady of the United States
VII. Choose the correct sentence among A, B, C or D which has the same meaning as the given one
1. What time will it start?
A. How will it start? B. Will is start?
C. What will it start? D. When will it start?
2. This book is more interesting than that book.
A. This book is more beautiful than that book.
B. That book is more boring than this book.
C. This book is cheaper than that book.
D. That book is more expensive than this book.
3. She doesn't have any friends in Ha Noi.
A. She has a few friends in Ha Noi. B. She has few friends in Ha Noi.
C. She has some friends in Ha Noi. D. She has no friends in Ha Noi.
4. Her new school is bigger than her old school.
A. No school is bigger than her new school.
B. Her new school is different from her old school.
C. Her old school is smaller than her new school.
D. Her old school is more beautiful than her new school.
5. Mr. Spencer is not free this afternoon.
A. Mr. Spencer is busy this afternoon.
B. Mr. Spencer is tired this afternoon.
C. Mr. Spencer is out this afternoon.
D. Mr. Spencer is not at home this afternoon.
VIII. Complete the second sentence so that it has the similar meaning to the first one.
1. What children like to do at Tet is to watch fireworks and dragon dance.
Children are keen …………………………………………………………………………………….
2. I think no country in the world is more beautiful than Vietnam.
I think Vietnam ………………………………………………………………………………………
3. My study room has two chairs and one table.
There is……………………………………………………………………………………………….
4. It would be a good idea if students learn in groups to help each other.
Students should ………………………………………………………………………………………
5. I think you should go travelling this summer to relax after a hard working time.
I advise you to ………………………………………………………………………………………..
62
IX. Complete each of the following sentences with the words or phrases given
1. Mrs. Ngoc/going/buy/some toys/children?
………………………………………………………………………………………………………….
2. Many plants/animals/danger/because/we/destroying /them.
………………………………………………………………………………………………………….
3. Ba/usually/play tennis/fall/but/sometimes/sailing.
………………………………………………………………………………………………………….
4. Lan/Ly/go/bookstore/buy/books/now.
………………………………………………………………………………………………………….
5. Sister/have/bike/and/cycle/work/every day.
………………………………………………………………………………………………………….

63
PRACTICE TEST 8
I. Odd one out
1. A. twice B. three C. four D. one
2. A. cold B. summer C. spring D. winter
3. A. badminton B. play C. ride D. go
4. A. volleyball B. basketball C. swimming D. bike
5. A. like B. weather C. season D. activity
II. Choose the word whose underlined part is pronounced differently from that of the others
1. A. hot B. hall C. hour D. hat
2. A. visit B. smile C. thick D. fit
3. A. office B. distance C. nice D. uncle
4. A. last B. taste C. fast D. task
5. A. sight B. cough C. daughter D. bought
III. Choose the best answer to fill in the blank.
1. They had better…………. their homework before watching TV.
A. to finish B. finishing C. finish D. finished
2. Who made a lot of………….in the field of electricity?
A. inventing B. invent C. inventors D. inventions
3. Would you mind………….the hat on the table?
A. having put B. putting C. to put D. put
4. My mother taught me………….to make spring rolls.
A. what B. how C. where D. when
5. I………….a letter from my old friend last week.
A. received B. gave C. took D. sent
6. We haven’t met her………….we left Washington.
A. for B. while C. when D. since
7. They suggest………….a movie tonight.
A. seeing B. having C. reading D. going
8. The students………….their homework 3 days ago.
A. would finish B. finished C. finish D. will finish
9. After smiling at her son………….., she got on the bus.
A. unhappy B. happy C. happily D. very happy
10. In the fire - making contest, two team members had to make a fire in the ………….way.
A. different B. wonderful C. traditional D. original
11. Mrs. Lien………….to Thailand every week.
A. is flying B. flies C. fly D. flew
12. This novel is based………….historic events.
A. on B. in C. to D. from
13. What do you often have………….lunch?
A. of B. to C. at D. for
14. It’s………….to travel around the world.
A. interestedly B. interested C. interesting D. interestingly
15. Our team won the game because we played very………….
A. good B. better C. well D. goody
IV. Identify the mistake in each sentence
1. He led us on a guiding hike along the edge of the canyon.
A B C D
2. This question can answer by most of the students in my class.
64
A B C D
3. I don’t know how to play the game when I was ten.
A B C D
4. Can I help you by your language ?
A B C D
5. They are used to live here. Everything will become easy soon.
A B C D
V. Give the correct form of the words in the brackets in each of the following sentences.
1. We were………….., so we bought something to eat in the shop. (hunger)
2. That is a…………..machine because it doesn’t work any more. (use)
3. They arrived at the destination…………... (safety)
4. Our life is much…………..with the help of modern equipment. (good)
5. If you want to join the club, please fill in this…………..form. (apply)
VI. Complete the passage with the words given in the box
flying portrait finish centuries
sculpture one human about
Leonardo da Vinci lived in Italy in the fifteenth and sixteenth (1)…………. He was a student in Florence,
where he studied painting, (2) …………. and design. He began a lot of paintings, but he didn’t (3)………….
many of them. His picture of the Mona Lisa is the most famous (4) …………. in the world. Leonardo was
interested in many things. He wanted to know (5)………….everything he saw. He examined the (6)…….
body. He thought that the sun didn’t go round the earth. He wrote music. He designed a (7)………….
machine 400 years before the first one flew. Many people didn’t understand his ideas. It is difficult to think
that (8)………….man could do so much.
VII. Read the passage and decide which answer A, B, C or D best fits each space.
Language (1)………….learn words in different ways. Some learners make a (2)………….and put into it the
(3)………….of new words in their mother tongue, and try to learn them by (4)………….. However, others
do not; instead, they write one or two example sentences with each new word in order to (5)………….how to
use the word in the (6)………….way. In order to remember words better, some learners (7)………….write
each word and its use on a small piece of paper and stick it somewhere in their house so as to learn it at any
time.
1. A. people B. students C. pupils D. learners
2. A. lit B. book C. list D. notebook
3. A. meanings B. meaner C. means D. mean
4. A. mind B. heart C. mouth D. head
5. A. forget B. know C. remember D. study
6. A. left B. right C. own D. main
7. A. ever B. even though C. even D. event
VIII. Complete the second sentence so that it has the similar meaning to the first one.
1. To study English well is difficult.
It is ………………………………………………………………………………………………………
2. Mr. Tran was a careless driver.
Mr. Tran………………………………………………………………………………………………….
3. There are nine members in the art club.
There are …………………………………………………………………………………………………
4. The girl is too weak to carry the bag.
The girl is not ……………………………………………………………………………………………
5. The last time I ate this kind of food was 6 months ago.
I haven’t ………………………………………………………………………………………………….
65
IX. Rearrange the following words and punctuation marks to complete the sentences.
1. has/the chores/to/today/Hai/do/himself/.
……………………………………………………………………………………………………………
2. there/my/a/trees/lot/school/on/to/are/the way/of/.
……………………………………………………………………………………………………………
3. clean/the/teacher / me / told / to/blackboard / the /.
……………………………………………………………………………………………………………
4. how/English?/to/me / tell / study / you/Could /.
……………………………………………………………………………………………………………
5. She/my class/is/enough/old/in/not/to be/.
……………………………………………………………………………………………………………

66
PRACTICE TEST 9
I. Odd one out
1. A. bored B. happy C. care D. good
2. A. washing machine B. stove C. fridge D. bridge
3. A. bridge B. palace C. skyscraper D. motor home
4. A. modern B. delicious C. famous D. know
5. A. Asia B. Europe C. America D. African
II. Choose the word whose underlined part is pronounced differently from that of the others
1. A. cruel B. useful C. July D. ruler
2. A. secretary B. school C. position D. else
3. A. mother B. tongue C. some D. today
4. A. both B. enroll C. program D. lost
5. A. pretty B. theater C. different D. student
III. Choose the best answer to fill in the blank.
1. He was happy………….her mother.
A. meet B. meets C. to meet D. will meet
2. Mary is pretty, but not so pretty………….Julia.
A. like B. as C. than D. so
3. He repairs his roof to stop it from………….
A. leaks B. leaking C. leak D. is leaking
4. My father has iust finished………….the newspaper.
A. reads B. read C. reading D. will read
5. I have no time………….with you.
A. to play B. played C. play D. plays
6. He directed that no one………….eat before sunset.
A. should B. would C. must D. could
7. Who are those people? Why are you looking at………….?
A. their B. themselves C. them D. theirs
8. Mr. Johnson washes his car every week,………….?
A. does he B. don’t he C. doesn’t he D. do he
9. My brother will go abroad as soon as he………….his passport.
A. get B. got C. could get D. gets
10. I have a sister but………….brothers.
A. any B. no C. some D. none
11. I………….a letter to my best friend last night.
A. wrote B. am writing C. write D. will write
12. I would like to see you………….a good job on the project.
A. to doing B. to do C. did D. do
13. Sam as well as her sister………….planning to come to our country next month.
A. were B. are C. is D. be
14. Those workers are skillful, so they have………….difficulty in their work.
A. few B. fewer C. less D. at all
15. I have to study very hard………….I can pass the next exam.
A. so that B. such C. therefore D. in order
16. Soldiers must have………….to face danger.
A. so courageous enough B. courageous enough
C. such enough courage D. enough courage
17. The black and white dog………….sitting under the tree at the moment.
67
A. are B. were C. is D. was
18. Joanna often………….playing the piano in her room.
A. practices B. will practice C. practice D. practicing
19. Mary………….in this house since 1978.
A. lived B. lives C. has lived D. is living
20. Janet wants to ………….a phone call to her family.
A. do B. put C. make D. have
IV. Identify the mistake in each sentence
1. Nobody was on the holiday last year.
A B C D
2. Does Nam’s mother go always to work by bus?
A B C D
3. Vegetables often have dirty from the farm on them.
A B C D
4. Could you tell me when the next train leaves and where buy tickets?
A B C D
5. Hoa prefers reading books to play video games.
A B C D
V. Give the correct form of the words in the brackets in each of the following sentences.
1. The…………..have to cook rice in this competition. (participate).
2. Mrs. Quyen and her husband had a…………..time in San Francisco. (wonder)
3. He’s deeply…………..and goes to church twice a week. (religion)
4. Sa Pa is a famous…………..resort in Vietnam. (mountain)
5. She cried…………..when she heard that news. (excite)
VI. Read the passage and decide which answer A, B, C or D best fits each space
A small boy and his father were having a walk in the country when it suddenly (1)………….. to rain very
heavily. They did not have an umbrella with them and there was nowhere to hide from the rain, (2)………….
they soon got wet, and the small boy did not feel happy. Then he turned to his father and (3)………….. to
him "Why does it rain? It isn’t very nice, isn’t it?” "No, it isn’t very nice, but it’s very (4)………….., Tom"
answered his father, "It rains to make fruits and vegetables (5)…………..for us."
1. A. was beginning B. begins C. begin D. began
2. A. because B. although C. so D. despite
3. A. told B. questioned C. asked D. said
4. A. uselessly B. usefully C. useless D. useful
5. A. to grow B. grow C. growing D. grown
VII. Choose the correct sentence among A, B, C or D which has the same meaning as the given one
In the first year of lower secondary school, I had some difficulties in learning English. My
pronunciation of English words was really bad and my English grammar was worse. I did not know how to
improve them. One afternoon after the lesson, my English teacher told me to wait for her outside the
classroom. She took me to the school library and showed me cassettes of pronunciation drills kept in a glass
bookcase. She also told me how to use an English-English dictionary to improve my English grammar. “Now
I think you know what you should do,” she said. I did not know that only one year later, I would win the first
prize in the English Speaking Contest held for secondary school students in my hometown.
1. Which sentence is true?
A. The writer had some difficulties in learning English when he was in primary school.
B. The writer had some difficulties in learning English when he was in lower primary school.
C. The writer had no difficulties in learning English when he was in lower secondary school.
D. The writer had some difficulties in learning English when he was in lower secondary school.
68
2. He was not good at…………….
A. speaking and listening B. pronunciation and speaking
C. grammar and pronunciation D. grammar and listening
3. Which sentence is not true?
A. His grammar was worse than his pronunciation.
B. His English teacher was a woman.
C. He knew how to improve them.
D. He did not know how to improve them.
4. She also told him how to use an English-English dictionary.
A. to improve his skill of listening.
B. to improve his pronunciation of English.
C. to improve his skill of speaking.
D. to improve his English grammar.
5. He won the first prize in the English Speaking Contest.
A. one month later B. four months later
C. ten months later D. twelve months later
VIII. Complete the second sentence so that it has the similar meaning to the first one.
1. It’s nearly two years since we saw Joanna.
We haven’t……………………………………………………………………………………………..
2. I have studied English for three years.
I began………………………………………………………………………………………………….
3. Your backpack is not the same as mine.
My backpack is …………………………………………………………………………………………
4. No one in the group is younger than Jane.
Jane is …………………………………………………………………………………………………..
5. I have a plan to visit my grandparents next weekend.
I am ……………………………………………………………………………………………………..
IX. Complete each of the following sentences with the words or phrases given.
1. It/impossible/understand/his theory
………………………………………………………………………………………………………….
2. I/used/stay/farm/grandparents/parents.
………………………………………………………………………………………………………….
3. The manager/come/the office?
………………………………………………………………………………………………………….
4. Your writing/difficult/read
………………………………………………………………………………………………………….
5. We/extreme/tired/after/trip/so/sleep/bus/way/home
………………………………………………………………………………………………………….

69
PRACTICE TEST 10
I. Odd one out
1. A. dolls B. stamps C. gymnastics D. books
2. A. egg B. pork C. beef D. ham
3. A. service B. charitable C. non-profit D. homeless
4. A. milk B. lemonade C. juice D. sandwich
5. A. glass B. plate C. fork D. bowl
II. Choose the word whose underlined part is pronounced differently from that of the others
1. A. wide B. world C. window D. answer
2. A. put B. burn C. turn D. fur
3. A. concern B. hot C. correct D. control
4. A. sugar B. salt C. soup D. sauce
5. A. absent B. content C. silence D. legend
III. Match a word in column A with its definition in column B
A B
A. To make programs for community radio station.
B. Help toads cross the road when they migrate for the annual
1. Kids theatre volunteer breeding season.
2. Radio producer C. Work with children to do theatre plays and musical
3. Festival volunteer productions. .
4. Toad warden D. Look after pets when elderly owners go into hospital.
5. Tandem bike rider E. Be the front rider on a tandem bike so blind people can enjoy
6. Pet carer recycling.
F. Work on a charity stall at a music festival.

Answers:
1……….. 2.……….. 3.……….. 4.……….. 5.……….. 6………..
IV. Choose the best answer to fill in the blank.
1. Some students are making a lot of………..in the school yard.
A. noisily B. noisy C. noiser D. noise
2. Susan is much………..with children than her sister is.
A. most patient B. patient C. patienter D. more patient
3. What is the………..month of the year in Vietnam?
A. hot B. hotter C. hottest D. more hot
4. They’ve been waiting, and………..has she.
A. either B. too C. neither D. so
5. My teacher always advises me ……….. hard.
A. study B. studying C. studied D. to study
6. The expert says that cycling uses more calories than walking,……….. running use the most calories of all.
A. but B. and C. so D. or
7. She bought a………..of bread so we can make sandwiches.
A. loaf B. bar C. bowl D. slide
8. Mary hates………..at weekends.
A. gone hiking B. went hiking C. go hiking D. going hiking
9. Sports like riding a bicycle………..running use a lot of calories.
A. so B. but C. or D. and
10. Can I help you? Yes, I’d like ……….. wine, please.
70
A. and B. a C. any D. some
11. We’d like to watch the football match, and he would………...
A. neither B. too C. so D. either
12. My dad will………..apple trees, but mv mum doesn’t like this idea.
A. grew B. grow C. to grow D. growing
13. There are many………..in ASEAN countries.
A. attractives B. attracts C. attractions D. attractings
14. Physics is more difficult………..history.
A. from B. as C. at D. than
V. Give the correct form of the verbs in the brackets to complete the sentences
1. Water puppetry (begin)………..in the 11th century.
2. The teachers at our school (give)………..us lots of homework.
3.………..(Mai/ go) to school every day? No, she (go)………..to school from Monday to Friday.
4. Harry Potter and the Goblet of Fire (write)………..by J K Rowling.
5. I think she (not agree)………..with this idea.
VI. There is one mistake in each of the following sentences. Find and correct it.
1. The dress was pink and little white flowers on it.
2. When I am ill, my mom looks at me.
3. When I saw her, I thought she looked happily.
4. Didn’t you understood what I said to you?
5. My mother bought for me a nice schoolbag on the occasion of a new school year.
VII. Read the passage and decide which answer A, B, C or D best fits each space
Dental Health
Teeth are important for chewing food and talking. Without teeth, we would be able to eat only soft
food. It would also be hard for us (1)………..without teeth. Therefore, we should look after our teeth. The
food that we eat can affect our teeth. Too (2)………..sugar can make acid. The acid begins to eat away the
teeth. Cavities in the teeth are holes made by acid. Children love cola drinks. One can of cola (3)………..ten
teaspoons of sugar. We should avoid this kind of food with lots of sugar. Another way we can look after our
teeth is by (4)………..them. The teeth should be brushed at least (5)………..a day. Also, we must give our
teeth special treatment by visiting the dentist. The dentist can look after cavities in our teeth.
1. A. spoke B. speak C. speaking D. to speak
2. A. lots of B. plenty C. many D. much
3. A. has B. have C. is having D. had
4. A. drilling B. filling C. cleaning D. showering
5. A. twices B. twice time C. two times D. two time
VIII. Read the following text and choose the best answer to the questions below.
Not a Minute Lost
Ants live and work together the way people do. They live in ant cities, which they work hard to build.
Every ant has his own work to do. The queen ant lays the eggs. Some ants help by caring for her and bringing
her food. Other ants take care of the babies. Worker ants dig tunnels and build storerooms. They carry seeds
and sand. Other ants work as soldiers. They stand watch, ready to fight. Ants build their cities in many
places. Some build in tree tops. Some build in the ground. Some ants even build their cities in pieces of old
wood. An ant city is always a busy place.
1. The story as a whole is about………….
A. how ants live B. ant eggs C. queen ants D. worker ants
2. The work of the queen ant is to………….
A. stand watch B. take care of the babies
C. lay eggs D. build an ant city
71
3. Ants store their food underground …………..
A. in hollow trees B. in tree tops
C. in storerooms D. in pieces of old wood
4. Each ant…………..
A. lays his own eggs B. can do different jobs in an ant city
C. is ready to fight his enemy D. has his own work to do
5. Soldier ants …………...
A. dig tunnels and build storerooms B. stand watch and ready to fight
C. take care of baby ants D. care for the queen ant
IX. Complete the second sentence so that it has the similar meaning to the first one.
1. How much does it cost you to buy that souvenir in dollars?
How many ………………………………………………………………………………………………..
2. When did you have it?
How long …………………………………………………………………………………………………
3. These Dong Ho paintings are more expensive than those modern paintings.
Those modern paintings are not ………………………………………………………………………….
4. Jane didn’t go fishing yesterday because it rained heavily.
Because of ………………………………………………………………………………………………..
5. This is the first time I have seen him.
I have never …………………………………………………………………………………………….
X. Complete each of the following sentences with the words or phrases given.
1. Volunteers/wash/dirty clothes/classify/before/give/the poor.
…………………………………………………………………………………………………………….
2. your children/go/camp/every summer holiday?
…………………………………………………………………………………………………………….
3. my cousin/give/me/book/gift/next birthday.
…………………………………………………………………………………………………………….
4. I/enjoy/play/sports/because/it/good/health.
…………………………………………………………………………………………………………….
5. he/find/mountain climbing/dangerous/so/he/not take/it.
…………………………………………………………………………………………………………….

72
TRƯỜNG THCS LƯƠNG THẾ VINH
PRACTICE TEST 1
I. MULTIPLE CHOICE
Circle the odd one out. (2 pts)
1. A. hard-working B. clever C. generous D. kindness
2. A. history B. park C. pagoda D. museum
Circle the word which has the underlined part pronounced differently from the others. (3 pts)
3. A. lakes B. works C. poems D. tents
4. A. magazine B. bag C. hard D. family
5. A. expensive B. opposite C. those D. noisy
Circle the best option to complete the sentences. (12 pts)
6. He………….his homework last night, but this morning he left it at home.
A. enjoyed B. completed C. finish D. does
7. You shouldn’t touch the animals on the street………….they may bite or scratch you.
A. when B. because of C. so D. because
8. Listen! He………….the piano upstairs.
A. are playing B. playing C. plays D. is playing
9. What do you………….of your classmates?
A. like B. think C. thought D. see
10. I………….like to fly planes and visit other countries.
A. had B. should C. would D. could
11. Our grandparents live in………….but we live in a big city.
A. country B. apartment C. villa D. the country
12. She is a nurse. She looks………….patients and works in a hospital.
A. of B. after C. on D. up
13. That story is very………….so he wants to read it again.
A. boring B. interested C. excited D. interesting
14. There………….a book and some pens next………….the pencil case.
A. is/to B. are/to C. is/near D. are/near
15. I hope the ………….can repair our car quickly.
A. mechanic B. teacher C. architect D. dentist
16. - Shall we go to Hung temple?
- ………….
A. Wait a minute. I have to look at the map
B. I don’t think so
C. That’s a good idea
D. I don’t want any more
17. - Where’s the cooker? I can’t find it.
- ………….
A. Oh, I see nothing B. It’s over there. It’s on the table
C. Really? I don’t remember it D. It’s not my problem
There is one mistake in each of the following sentences. Find and correct it. (3 pts)
18. My new classmates are very kind with me.
A B C D
19. How many wine is there in the glass ?
A B C D
20. Every day my brother has to do homeworks before going to bed.
73
A B C D
Read the passage and choose the correct answer A, B, C or D. (5 pts)
Stratford
Anyone who visits England would like to spend some time in Stratford-upon-Avon. Located in the
center of England, it is the town that is rich in both history and culture. Its population is about 23,000.
Stratford-upon-Avon is famous because it is the place where the talented playwright Shakespeare was born
and died. When you come there, you can visit his birthplace, and some buildings which used to belong to his
family. There are many other buildings that are worth visiting in the town. Enjoying an evening in Stratford-
upon-Avon is very interesting. There are restaurants that are suitable for all tastes. In the evening, the Royal
Shakespeare Theatre offers interesting plays written by Shakespeare and you have a chance to see some of
the best actors and actresses of England on stage. There are many other kinds of amusement and
entertainment waiting for you.
21. What is Stratford-upon-Avon rich in? - …………..
A. plays B. entertainment
C. population D. history and culture
22. Stratford is the place…………..
A. that does not belong to England
B. where Shakespeare was born and died
C. which is overpopulated
D. which is very rich
23. The buildings in Stratford …………..
A are all in the possession of Shakespeare
B. are old and ugly
C. are worth visiting
D. have been demolished
24. Restaurants in Stratford are…………...
A. suitable for all tastes B. expensive
C. rich D. delicious
25. In the Royal Shakespeare Theatre, you can enjoy…………...
A. actors and actresses B. food of all kinds
C. plays written by Shakespeare D. films
Read the following passage and choose the correct answer A, B, C or D. (5 pts)
The Lions and the Mouse
One day, a lion is sleeping. A mouse runs (26)…………..the lion’s nose and the lion wakes up. The
lion is angry. The mouse is scared. “I’m very sorry,” says the mouse. “Don’t eat me! I can (27)………..you
one day.”
The lion laughs. “That’s funny!” it says. “You are little! How can you help me?” But the lion is not angry
now. The mouse runs away.
One year later, the lion is walking in the jungle. A big net (28) …………..over the lion. The lion opens its
(29) …………..and roars. The mouse hears the lion and it runs to help.
“Don’t move,” says the mouse. “I can help you.”
The mouse chews the net and makes a big hole. The lion is free. “Thank you,” says the lion. “You are (30)
………….., but you are kind.”
26. A. over B. under C. on D. off
27. A. take B. make C. do D. help
28. A. fell B. falling C. falls D. fall
29. A. eyes B. mouth C. arms D. nose
30. A. big B. large C. much D. little
74
II. WRITING (10pts)
Rewrite the following sentences without changing the meaning, using the words given. (5 pts)
31. He often joins in outdoor activities after school.
He often takes ……………………………………………………………………………………………
32. Their school is near the stadium.
Their school isn’t…………………………………………………………………………………………
33. She likes learning English.
She is fond ……………………………………………………………………………………………….
34. Do your classmates ride their bicycles to school every day?
Do your classmates go……………………………………………………………………………………
35. How long is the Red River?
What is……………………………………………………………………………………………………
Complete each of the following sentences with the words or phrases given (3 pts)
36. What kind/information/you/look for/now/?/
…………………………………………………………………………………………………………….
37. Our family/go/back/to/hometown/visit/grandparents/last weekend/./
…………………………………………………………………………………………………………….
38. Linh/practice/listen/by/watch/English cartoons/TV/every day/./
…………………………………………………………………………………………………………….
Rearrange the following words and punctuation marks to complete the sentences. (2 pts)
39. Lan/yesterday/go/was/school/did/because/she/to/sick /./not/
…………………………………………………………………………………………………………….
40. Students/recess/different/in/part/activities/at/take /./
…………………………………………………………………………………………………………….

75
PRACTICE TEST 2
I. MULTIPLE CHOICE
Circle the odd one out. (2 pts)
1. A. first B. second C. third D. four
2. A. see B. look C. meeting D. find
Circle the word which has the underlined part pronounced differently from the others. (3 pts)
3. A. city B. village C. twice D. family
4. A. usually B. Sunday C. mum D. bus
5. A. seaside B. season C. sugar D. sun
Circle the best option to complete the sentences. (12 pts)
6. I live……………my parents in an apartment in the city center.
A. to B. about C. from D. with
7. - What’s your village……………?
- It’s small and quiet.
A. be B. being C. like D. alike
8. Every day, the……………go to the market on foot.
A. village B. villages C. villager D. villagers
9. During rush hour, the streets are very……………
A. crowded B. crowd C. many D. much
10. Our school is……………86 Tran Phu Street.
A. on B. at C. in D. next
11. ……………does your family live? - In Nam Dinh province.
A. Where B. What C. When D. Why
12. -……………you……………your house? - Yes, I like it very much.
A. Do/like B. Would/like C. Should/like D. Could/like
13. Fansipan is the highest……………in Viet Nam.
A. hill B. fall C. mountain D. building
14. There……………a park and some buildings near our neighborhood.
A. are B. is C. be D. being
15. -……………takes you to school every day? - My dad does.
A. Where B. What C. Who D. Why
16. - Do you like living in Ha Noi? - ……………
A. Yes, very much B. Yes, I’d love
C. No, I wouldn’t D. I could like
17. - Nice to meet you. - ……………
A. Nice to meet you, too B. I want to meet you
C. I like you D. Not at all
There is one mistake in each of the following sentences. Find and correct it. (3 pts)
18. His place is near to the city center so he likes it.
A B C D
19. Do you know how much a kilo of rice cost ?
A B C D
20. You can find different dictionaries on the shelfs for reference books.
A B C D
Read the text and then write T (true) or F (false) beside the sentence.
Leo and Lily hear a noise. It’s a fox! The fox is hungry. It wants some dinner. It wants to eat Leo!
Leo tries to fight the fox, but the fox is very big. Lily is in a tree. She pecks a pinecone. The pinecone falls
and hits the fox on the head! Leo escapes and climbs the tree. The fox is angry. It can’t climb trees.
76
Leo and Lily wait in the tree for a long time. At last, the fox goes away. Leo and Lily are safe, but they want
to go home now. Lily flies up very high. She sees the garden! She squawks happily. Lily shows Leo the way
home. But Lily is small and her wings are tired. She sits on Leo’s back and Leo carries Lily home. Leo and
Lily are safe at home in their garden again. They’re friends now. Lily flies around the garden and Leo never
tries to catch her... Well, hardly ever!
21. The fox is full but he still wants some dinner.
22. The pinecone falls and hits the fox on the head.
23. Leo stands there and stares at the fog.
24. Leo and Lily are safe because the fog goes away after a long time.
25. Leo and Lily get on well after in danger.
Read the following passage and mark the letter A, B, C or D to indicate the correct word or phrase
that best fits each of the numbered blanks.
People long ago (26)…………….from place to place or traveled by horse and cart. When they
traveled long distances they went (27) …………….train or ship. It was slow and difficult. Today we can
travel in cars, fast trains, buses, or airplanes (28) …………….to places that are far away. We can travel to
places (29) …………….the ocean or across the world in a day. Travel is fast (30) …………….easy now.
26. A. walk B. walked C. walking D. walks
27. A. by B. in C. on D. into
28. A. go B. going C. to go D. goes
29. A. on B. in C. above D. across
30. A. and B. but C. both D. to
II. WRITING (10pts)
Rewrite the following sentences without changing the meaning, using the words given. (5 pts)
31. What’s her address?
Where ………………………………………………………………………………………………
32. Hai likes his new school.
Hai is fond ………………………………………………………………………………………….
33. His office is far from his house.
His office isn’t ………………………………………………………………………………………
34. There are five people in Peter’s family.
Peter’s family ……………………………………………………………………………………….
35. Would you like a cup of tea?
Do you ………………………………………………………………………………………………
Complete each of the following sentences with the words or phrases given (3 pts)
36. how/he/feel/new/home/?/
……………………………………………………………………………………………………….
37. she/be/happy/because/live/her/uncle/and/aunt/in/Ha Noi.
……………………………………………………………………………………………………….
38. my/classmates/very/nice/kind/me/
……………………………………………………………………………………………………….
Rearrange the following words and punctuation marks to complete the sentences. (2 pts)
39. relatives/the/weekend/our/We/at/often/visit/. /
……………………………………………………………………………………………………….
40. sitting/on/friends/are/sun/turtle/stones/in/the/My/.
……………………………………………………………………………………………………….

77
PRACTICE TEST 3
I. MULTIPLE CHOICE
Circle the word which has the underlined part pronounced differently from the others. (3 pts)
1. A. education B. collection C. question D. celebration
2. A. season B. break C. great D. steak
3. A. thanks B. thirsty C. father D. theater
Circle the odd one out. (2 pts)
4. A. fine B. nice C. good D. bad
5. A. two B. third C. fourth D. first
Circle the best option to complete the sentences. (12 pts)
6. We always need more players. Why don’t you come…………?
A. on B. along C. with D. after
7. Giving a book report before the class is an interesting…………
A. act B. action C. activity D. deed
8. I like swimming and…………does my sister.
A. so B. too C. neither D. also
9. I often ………… in winter.
A. go playing skiing B.go skiing
C. play skiing D.go to ski
10. Some young people are working…………..hospital volunteers.
A. to be B. such as C. like D. as
11. He likes stamps. He is a stamp …………..
A. collect B. collecting C. collector D. collects
12. There is…………..wine in the red cup than in the orange cup.
A. a few B. a little C. less D. fewer
13. Would you like to come to my house for lunch? - …………..
A. Yes, I do B. I’d love to C. Yes, I like D. OK. I’d like
14. She…………..some experiments in Physics class.
A. makes B. learns C. does D. enjoys
15. Royal City is on Nguyen Trai street,…………..the Techcombank and Pico plaza?
A. next to B. near C. between D. in
16. A: Do you have a brother at this school? B:…………..
A. Yes, he’s my brother B No, he’s not
C. Yes, he’s in class 5A D. Yes, he does
17. - Was it your friend’s birthday? - …………..
A. Yes, she was B. Yes, it was C. Yes, it is D. Yes, she is
There is one mistake in each of the following sentences. Find and correct it.(3pts)
18. I will send him the message as soon as he returned.
A B C D
19. A little girl can’t spends all her time listening to stories.
A B C D
20. It’s three kilometers from my house into the park.
A B C D
Read the text and then write T (true) or F (false) beside the sentence. (5 pts)
Hammerfest is a small town in the north of Norway. It is about 1,000 kilometers north Arctic Circle.
Only 9,407 people live here. However, about 175,000 tourists come here every year. In the winter, people
here can do a lot of sports. They go skiing and they have snowmobiles. Children usually play outside, but if
it’s very cold, they play inside on their computer or watch TV. In the evening, they usually stay at home and
78
relax, or go and see friends. But the winter is difficult for old people because it is cold and light for only two
or three hours a day. People even have breakfast, lunch, and dinner in the dark.
In the summer, life in Hammerfest is completely different. It is light for 24 hours a day, and the
weather is sometimes very hot. People are outside every time. They go fishing, walking and have barbecues.
They don’t sleep much and children don’t want to go to bed because they find it isn’t dark.
Life here is quiet and beautiful. The air is very fresh and clean. But the winter is very long and the summer is
very short so people prefer spending their holidays to living here.
Arctic Circle: Vòng Bắc cực
Snowmobiles: xe chạy bằng máy trên tuyết và băng
21. Only 9,470 people live in Hammerfest.
22. About 175,000 tourists spend their holidays in Hammerfest every year
23. In the winter, people even have breakfast, lunch, and dinner in the dark.
24. It is light for 24 hours a day, and the weather is sometimes very hot in the summer in Hammerfest.
25. The winter is very long and the summer is very short so people prefer living to spending their holidays
here.
Read the following passage and choose the correct answer A, B, C or D. (5 pts)
THE OSTRICH
The ostrich is the largest bird in the world, and an adult can be more (26)…………….90 kilos. Most
wild ostriches live (27) …………….southern Africa, but there are only a few of
them left. Like all birds, ostriches have wings, (28) …………….they cannot fly. They use their
wings to help them turn when they are running. Ostriches can run very (29) ……………., from 65 to 90
kilometres an hour, so it is very difficult for other animals to catch them. Baby ostriches are the same size as
chickens and take about 3 years to become adults. Ostriches (30) …………….plants and can live for many
days without water.
26. A. than B. of C. like D. as
27. A. on B. in C. at D. of
28. A. or B. and C. but D. so
29. A. fastly B. fast C. quick D. fastest
30. A. ate B. eats C. eat D. eating
II. WRITING (lOpts)
Rearrange the following words and punctuation marks to complete the sentences. (2 pts)
31. learn/write/literature/about/essays/in/and/we/books/.
……………………………………………………………………………………………………….
32. the/ bell/into/the/students/ten/and/go/past/rings/all/the/ yard/half/,/at/.
……………………………………………………………………………………………………….
Complete each of the following sentences with the words or phrases given (3 pts)
33. you/mind/I/turned on/television?
……………………………………………………………………………………………………….
34. We/not see/her/since/we/be/holiday/Ha Long Bay.
……………………………………………………………………………………………………….
35. I/look/forward/see/you/soon.
……………………………………………………………………………………………………….
Rewrite the following sentences without changing the meaning, using the words given. (5 pts)
36. There are over eight hundred stamps in Nam’s collection.
Nam’s collection ……………………………………………………………………………………..
37. He is a careful driver.
He drives ……………………………………………………………………………………………..
38. What a luxurious car!
79
How …………………………………………………………………………………………………..
39. Finding an apartment in a big city is not easy.
It is…………………………………………………………………………………………………….
40. It takes me about two hours each day to do my homework.
I spend ………………………………………………………………………………………………...

80
PRACTICE TEST 4
I. MULTIPLE CHOICE
Circle the word which has the underlined part pronounced differently from the others. (3 pts)
1. A. far B. vacation C. garden D. party
2. A. day B. lazy C. map D. baby
3. A. easy B. empty C. dryer D. busy
Circle the odd one out. (2 pts)
4. A. dangerous B. sign C. road D. intersection
5. A. orange B. bread C. meat D. rice
Circle the best option to complete the sentences. (12 pts)
6. How……………is your house from school? - It’s about 20 minutes by bike.
A. much B. many C. long D. far
7. Nam has learnt English since, he was four and is now……………good.
A. nearly B. possibly C. quite D. probably
8. It is not easy……………an apartment in Ha Noi.
A. find B. finds C. to find D. finding
9. Schools in Viet Nam are different…………..those in the US.
A. from B. for C. in D. at
10. The bathroom in her house has a sink, a tub and a…………..
A. armchair B. shower C. refrigerator D. stove
11. He usually starts work……………six in the morning.
A. in B. on C. at D. to
12. There is……………wine in the red cup than in the orange cup.
A. a few B. a little C. less D. fewer
13. I never drink……………orange juice for breakfast.
A. a lot of B. much C. A & B D. many
14. Come in and have a……………
A. seat B. sit C. table D. chair
15. If anybody……………question, please ask me after class.
A. has a B. have C. have a D. has
16. - How nice your new schoolbag is!
A. - Thanks a lot. B.- You’re welcome.
C. - I bought it yesterday. D.- It’s a present from my father.
17. - Why do you enjoy learning English?
A. - Because it helps me communicate with international friends.
B. - I’m attending an English club.
C. - I think I’m very good at speaking English.
D. - Because I’m studying in a boarding school.
There is one mistake in each of the following sentences. Find and correct it. (3 pts)
18. My favorite book is different with her favorite one.
A B C D
19. Sharon participates on a lot of competitions and often plays very well in them.
A B C D
20. I think people don’t travel as long as long ago.
A B C D
Read the following passage and choose the correct answer A, B, C or D. (6 pts)
Americans like sport very much. One of the (21)……………popular kinds of sports in autumn is
football. All the high schools and universities have their own teams. In winter the most popular kind of sports
81
(22) ……………basketball. There is usually a match every evening in one school gymnasium or another. In
some parts of the United States there is a lot of (23) …………… and ice. Many people like skiing and
skating.
In the other two (24) ……………, millions of Americans enjoy baseball. The schools have their games in
spring but the most important professional games are (25) ……………during summer. Many people listen to
the games on the radio, watch them on television or read about them (26) ……………the newspapers.
21. A. most B. more C. much D. less
22. A. being B. be C. are D. is
23. A. rain B. snow C. fog D. snows
24. A. seasons B. season C. month D. months
25. A. playing B. plays C. played D. done
26. A. at B. of C. off D. in
Read the following passage and choose the correct answer A, B, C or D. (4 pts)
Television is one of man’s most important means of communication. It brings events and sounds from
around the world into millions of homes. A person with a television set can sit in his house and watch the
president making a speech or visit a foreign country. He can see a war being fought and watch statesmen try
to bring about peace. Through television, home viewers can see and learn about people, places, and things in
faraway lands. TV even takes its viewers out of this world. It brings them coverage of America’s astronauts
as the astronauts explore outer space.
In addition to all these things, television brings its viewers a steady stream of programs that are
designed to entertain. In fact, TV provides many more entertainment programs than any other kind. The
programs include action-packed dramas, light comedies, sporting events and motion pictures.
27. What does television bring us?
A. president making a speech
B. man’s most important means of talking
C. noise from around the earth
D. events and sounds around our globe
28. What can’t viewers do with a television at home?
A. see heavens and hells
B. see a war being fought
C. watch a president giving a talk
D. watch statesmen making attempt to bring about peace
29. Which of the following is not true?
A. By watching TV, people can widen their knowledge.
B. People know events and sounds from a different world.
C. People can watch TV without going out.
D. Television can bring us entertainment.
30. According to the passage, what don’t entertainment programs include?
A. dramas B. motion pictures C. strategies D. musicals
II. WRITING (10pts)
Rewrite the following sentences without changing the meaning, using the words given. (5 pts)
31. It took us five minutes to have a tea cup.
We spent ...................................................................................................................................................
32. We participate in English festival every year.
We take .............................. ......................................................................................................................
33. My sister likes listening to music better than reading books.
My sister prefers ......................................................................................................................................
34. We have a two - month summer vacation.
82
Our summer vacation lasts ......................................................................................................................
35. Our teacher goes to school on foot.
Our teacher .............................................................................................................................................
Complete each of the following sentences with the words or phrases given (3 pts)
36. Kate/meet/friends/once/week/play sports/Sunday/.
.................................................................................................................................................................
37. Books/play/important/role/life/.
.................................................................................................................................................................
38. It/be/better/go/slowly/quickly/carelessly /.
.................................................................................................................................................................
Rearrange the following words and punctuation murks to complete the sentences. (2 pts)
39. the/took/in/fortunately/,/bus/we/end/./right/home/the/
.................................................................................................................................................................
40. exercises/do/you/morning/before/always/breakfast/do/?/
.................................................................................................................................................................

83
PRACTICE TEST 5
I. MULTIPLE CHOICE
Circle the word which has the underlined part pronounced differently from the others. (3 pts)
1. A. interesting B. stripe C. invite D. India
2. A. send B. shelf C. quite D. empty
3. A. holiday B. take C. vacation D. lazy
Circle the odd one out. (2 pts)
4. A. bring B. swim C. take D. vacation
5. A. field B. market C. garden D. farm
Circle the best option to complete the sentences. (12 pts)
6. In…………, we learn about books and write essays.
A. History B. Literature C. Math D. Physics
7. He…………a vacation next week.
A. having B. have C. has D. will have
8. He always…………in the library after school.
A. will study B. studies C. study D. are studying
9. Her group is …………a play for the school anniversary celebrations.
A. doing B. talking C. rehearsing D. playing
10. What’s your…………subject, Lan? - I like History.
A. famous B. like C. favorite D. popular
11. “What time is it?” It…………four fifteen.
A. has B. at C. X D. is
12. They put the chairs…………the middle of their garden.
A. at B. on C. in D. X
13. She is…………intelligent than her classmates.
A. least B. more C. much D. fewer
14. He had a chance to watch a horse…………in his homeland during his summer holidays.
A. trip B. racing C. volunteer D. station
15. I ’m seeing my old classmate next Sunday. He’s a very…………person. He enjoys sports very much.
A. sporty B. serious C. curious D. responsible
16. Would you like some milk?
A. Yes, I do B. No, I don’t like
C. I like much D. No, thanks
17. Nam: What’s the matter with you? You look so sad. Hai:…………
A. Forget it! B. Let me say something,
C. I don’t know the answer. D. Not at all.
There is one mistake in each of the following sentences. Find and correct it. (3 pts)
18 There isn’t many pollution in the country because there isn’t much traffic.
A B C D
19. I don’t want to tell him to my salary.
A B C D
20. If he is not in work, he must be at home.
A B C D
Read the text, and then write T (true) or F (false) beside the sentence.(5 pts)
Think about rules at home. Parents usually make these rules, and they make sure we follow them.
Parents can ask us to wash our hands and clean up. They make other rules about eating unhealthy food,
watching TV, or bedtime. What rules do you follow at home?
There are rules at school, too. Teachers and the principal make the school rules. We should always be
84
polite and take turns when we play games. Librarians help us follow the rules in the school library. We can’t
eat, drink, or be noisy there. Outside school, the crossing guard helps us cross the road safely, and we should
pay attention to the traffic light. These rules help keep us happy and safe. Can you think of more rules at
school?
21. At home, parents can ask us not to wash our hands and clean up.
22. Parents make other rules about eating unhealthy food, watching TV, or bedtime.
23. At school, we should always be polite and take turns when we play games.
24. Teachers and the principal help us follow the rules in the school library.
25. Librarians help us cross the road safely outside school.
Read the passage and choose the correct answer A, B, C or D. (5 pts)
Stamp collecting is an interesting hobby. You can learn many things, such as the geography of a
country from stamps. Postal stamps are a source of interesting facts and important dates about every country
in the world. It makes stamp collecting become very popular.
As you look at the pages of a stamp album, you can learn interesting details of foreign customs, arts,
literature, history and culture. Their colours can make you feel relaxed and happy.
Collecting stamps can become a business. If you are lucky in finding a special stamp, it will bring
you some money besides knowledge and pleasure.
26. Stamp collecting is an interesting hobby because…………..
A. stamps give you interesting facts and important dates about a country
B. you can learn many things such as the geography of a country from stamps
C. stamps give you uninteresting facts and important dates about a country
D. A and B are correct
27. All of the following are true EXCEPT…………...
A. stamp collecting can make you famous
B. you can earn money from your collection if you are lucky
C. stamps can make you know more
D. stamps can make you relaxed and happy
28. According to the passage, it is true to say that…………...
A. stamp collecting is a very popular hobby
B. stamp collectors can earn a lot of money from stamp collecting
C. stamp collecting is a hobby that costs you a lot of money
D. stamp collecting helps you become rich and famous
29. The word “business” in the last paragraph is closest in meaning to…………..
A. the activity of collecting stamps
B. the activity of buying or selling something
C. the activity of buying stamps from other countries
D. the activity of selling stamps from other countries
30. The main idea of the passage is…………...
A. good things from stamp collecting B. famous stamp collectors
C. the history of stamp collecting D. the ways of stamp collecting
II. WRITING (10pts)
Rewrite the following sentences without changing the meaning, using the words given. (5 pts)
31. Vietnamese students have fewer vacations than American ones.
American students ………………………………………………………………………………….
32. It is necessary for you to wash your hands before meals.
You ………………………………………………………………………………………………
33. It is such good weather that they are going for a picnic.
The weather ………………………………………………………………………………………..
85
34. What was Mr. Tan’s weight?
How ………………………………………………………………………………………………..
35. He looks after sick people.
He takes …………………………………………………………………………………………….
Complete each of the following sentences with the words or phrases given (3 pts)
36. we/not see/her/since/we/be/holiday/Ha Long Bay.
………………………………………………………………………………………………………
37. can / tell/how/stadium?
………………………………………………………………………………………………………
38. have/we/music/Monday/and/on/classes/Friday.
………………………………………………………………………………………………………
Rearrange the following words and punctuation marks to complete the sentences. (2 pts)
39. class/student/intelligent/to/in/the/most/said/is/be/the/He/./
………………………………………………………………………………………………………
40. 2nd/birth/on/date/is/of/October/His/./
………………………………………………………………………………………………………

86
PRACTICE TEST 6
Mark the letter A, B, C or D to indicate the word whose underlined part differs from the other three in
pronunciation in each of the following questions.
1. A. table B. amazing C. orange D. favorite
2. A. advice B. delicious C. center D. juice
Mark the letter A, B, C or D to indicate the word that doesn’t belong to the group.
3. A. see B. look C. watch D. know
4. A. cow B. chicken C. buffalo D. egg
Mark the letter A, B, C or D to indicate the underlined part that needs correction in each of the
following questions.
5. I am liking history because it’s an interesting subject.
A B C D
6. After the first three lessons, we have a twenty-minutes break.
A B C D
7. Tim is at the same age as Hoa.
AB C D
Mark the letter A, B, C or D to indicate the most suitable response to complete each of the following
exchanges.
8. “Excuse me, can you tell me where I can catch a bus to London, please?”
-……………
A. Sure, go ahead. B. Ok. Here is the ticket.
C. Sorry, I’m new here myself. D. Yes, please.
9. “It was very kind of you to help me out, Paul?”
Paul :“……………”.
A. I’m glad you like it. B. That was the least I could do.
C. Thanks a million! D. You can say that again.
Mark the letter A, B, C or D to indicate the correct answer to each of the following questions.
10. In America, soccer is the usual word for…………..
A. volleyball B. football C. basketball D. baseball
11.…………..does the plane fly? It flies at 500 miles an hour.
A. How much B. How fast C. How D. How long
12. Hoa’s brother is a…………..He repairs cars in a garage.
A. writer B. mechanic C. journalist D. farmer
13. He looks so…………... His face turns red when he talks with strangers.
A. friendly B. creative C. clever D. shy
14. How many………….. are there in your house?
A. kitchen B. room C. rooms D. living room
15. He wears thick…………... They make him so intelligent.
A. shoulders B. tails C. glasses D. necks
16. She has…………..books than Linh.
A. much B. more C. most D. many
17. She’s very beautiful. She has long black…………..and brown…………..
A. face/fingers B. hair/ eyes C. nose/foots D. arm/ears
18. Let’s…………..to Huong’s house.
A. come B. to come C. comes D. coming
19. What…………..great idea!
A. a B. Ø C. the D. an
Mark the letter A, B, C or D to indicate the word(s) CLOSEST in meaning to the underlined word(s)
87
in each of the following questions.
20. My father helps me repair my bicycle.
A. enjoy B. fix C. play D. clean
21. Talking with friends is a common way of relaxing at recess.
A. good B. popular C. easy D. different
Read the following passage and mark the letter A, B, C or D to indicate the correct word or phrase
that best fits each of the numbered blanks.
Oak City is a great city. There’s so (22)…………… to do! There are cinemas, parks and discos. It’s
got some great clothes and music shops (23) …………… Oak city is very modern and beautiful. There aren’t
any (24)……………buildings and there’s some beautiful (25) ……………around the city. Is there any
pollution in Oak city? There isn’t much pollution because there isn’t much traffic. Everyone travels (26)
……………bike and walks, so there aren’t many accidents. No one works, so everyone has got a lot of free
time. Are there any unfriendly people in Oak city? - No, so come and spend a little time here.
22. A. many B. much C. any D. some
23. A. neither B. either C. so D. too
24. A. pretty B. beautiful C. ugly D. unpleasant
25. A. view B. scenery C. scene D. sight
26. A. in B. by C. so D. with
Read the following passage and mark the letter A, B, C or D to indicate the correct answer to each of
the questions.
Nowadays people are more aware that wildlife all over the world is in danger. Many species of
animals are threatened, and could easily become extinct if we do not make an effort to protect them. There
are many reasons for this. In some cases animals are hunted for fur or for other valuable parts of their bodies.
Some birds, such as parrots, are caught alive , and sold as pets. For many animals and birds, the problem is
that their habit at-the place where they live is disappearing. More land is used for farms, for houses and
industry, and there are fewer open places than there once were. Farmers use powerful chemicals to help them
grow better crops, but these chemicals pollute the environment and harm the wildlife. The most successful
animals on the earth - human beings will soon be the only ones left, unless we can solve this problem.
27. What does the word “extinct” in line two mean?
A. dead
B. no longer in existence
C. suffering from a mental or physical handicap
D. not in a place because of illness
28. The word “this” in line three refers to…………….
A. the fact that many species of animals could easily become extinct
B. the fact that people are more aware that wildlife is in danger
C. wildlife
D. danger
29. According to the passage, one of the reasons that people hunt animals is……………..
A. They want their fur and other valuable parts of their bodies
B. They want to sell them as pets
C. They consider this a sport
D. They want to kill them
30. Which of the following is not true?
A. Human beings are the most successful animals on earth.
B. If we can solve the problem, we will soon be the only ones left.
C. Farmers use chemicals in order to grow better crops.
D. Chemicals pollute the environment.
88
31. What can be the best title of the passage?
A. Protect Our Earth. B. Birds are in Danger.
C. The Threat to the Environment. D. The Most Successful Animals.
Mark the letter A, B, C, or D to indicate the meaningful sentence written by using the word(s) or
phrase(s) given in each of the following questions.
32. I/delighted/ get /your/letter /yesterday.
A. I delighted to get your letter yesterday.
B. I was delighted getting your letter yesterday.
C. I delighted to getting your letter yesterday.
D. I was delighted to get your letter yesterday.
33. The/bell/rang/while/I/have/dinner.
A. The bell rang while I had dinner.
B. The bell rang while I was having dinner.
C. The bell was ringing while I was having dinner.
D. The bell was ringing while I had dinner.
34. I/prefer/live/the country/live/the city.
A. I prefer to live in the country to living in the city.
B. I prefer living in the country than living in the city.
C. I prefer living in the country to live in the city.
D. I prefer living in the country to living in the city.
Mark the letter A, B, C, or D to indicate the sentence that best combines each pair of sentences in the
following questions.
35. Nam is driving a car. He is too young.
A. Nam is not old enough to drive a car.
B. Nam is not young enough to drive a car.
C. Nam is too old to drive a car.
D. Nam is not enough to drive a car.
36. Tuan studies hard. He wants to pass the exam.
A. Tuan studies hard in order to pass the exam.
B. Tuan studies hard so as to pass the exam.
C. Tuan studies hard to pass the exam.
D. All are correct.
37. Some kinds of fish live in fresh water. Others live in the sea.
A. Some kinds of fish live in fresh water, so others live in the sea.
B. Some kinds of fish live in fresh water, for others live in the sea.
C. Some kinds of fish live in fresh water, or others live in the sea.
D. Some kinds of fish live in fresh water, and others live in the sea.
Mark the letter A, B, C, or D to indicate the sentence that is closest in meaning to each of the following
questions.
38. No building in the town is older than this church
A. This chinch is the oldest building in the town.
B. This church is older than any other buildings in the town.
C. This church is the older building in the town.
D. A & B.
39. They have two ten - minute breaks.
A. They have a break that lasts ten minutes.
B. They have ten breaks that last two minutes.
C. They have two breaks that last ten minutes.
89
D. They have a break that lasts two minutes.
40. She finds Geography difficult.
A. She thinks Geography is not difficult.
B. She thinks Geography is not easy.
C. She finds Geography interesting.
D. She finds Geography is easy.

90
PRACTICE TEST 7
Mark the letter A, B, C or D to indicate the word whose underlined part differs from the other three in
pronunciation in each of the following questions.
1. A. rest B. terrible C. expensive D. empty
2. A. sea B. ear C. peace D. tea
Mark the letter A, B, C or D to indicate the word that doesn’t belong to the group.
3. A. thirsty B. danger C. hungry D. tired
4. A. breakfast B. take C. brush D. wash
Mark the letter A, B, C or D to indicate the underlined part that needs correction in each of the
following questions.
5. Mr. Brown is looking of an apartment for his family.
A B C D
6. Hung spends 8 hours to study at school every day.
A B C D
7. Minh enjoys to play volleyball in his free time.
A B C D
Mark the letter A, B, C or D to indicate the most suitable response to complete each of the following
exchanges.
8. “Why don’t we buy her a dress as a birthday present?”
-……………..
A. Let’s do that.
B. Can you say it again.
C. Great idea! There is a nice shop near my house.
D. Both A & B are correct.
9. You can use my bike but take it back before 6p.m.
-……………..
A. Yes, I do. B. Ok. I will.
C. Don’t worry. D. I will keep my promise.
Mark the letter A, B, C or D to indicate the correct answer to each of the following questions.
10.……………..is on the borders of Nepal and Tibet.
A. Everest Mount B. Everest Mountain
C. Mount Everest D. the Mount Everest
11. You should pay more…………….in class.
A. attention B. care C. notice D. part
12. It’s difficult to cross this street because there is too much……………..
A. people B. crowd C. transport D. traffic
13. My favorite subject at school is…………….
A. electron B. electric C. electronics D. electricity
14. Throw the old newspapers…………….
A. forward B. off C. away D. out
15. On Sunday mornings I get up…………….8 o’clock.
A. at B. to C. until D. from
16.…………….does the group ’The Wall’ play?
A. What sort of music B. What sort of the music
C. What’s the sort of music D. What’s the kind of music
17. Let’s go to the pop concert at New College.
-…………….
A. Yes B. All right C. OK, we go D. Yes, let we go
91
18. Does Mr. Brown have…………….children?
A. a few B. any C. some D. the
19. People have become more and more concerned…………….the environment.
A. for B. with C. in D. about
Mark the letter A, B, C or D to indicate the word(s) CLOSEST in meaning to the underlined word(s)
in each of the following questions.
20. I need to go to the chemist’s to buy some medicines.
A. store B. bakery C. drugstore D. post office
21. I don’t think he is such an impolite person.
A. sad B. unhappy C. interesting D. rude
Read the following passage and mark the letter A, B, C or D to indicate the correct word or phrase
that best fits each of the numbered blanks.
“ My name is Stephanie and (22)…………….for a newspaper. I usually get up at 6.00 (23) ……
Mondays, Wednesdays and Fridays because I have my English class before I go to work. On Tuesdays and
Thursdays I get up later. I go to work by train, but on Fridays I drive my car, so I can visit my mother in the
afternoon. When the class (24) ……………at quarter to nine, I go to my office. I have lunch at 2.00 and then
I work (25) ……………about 7.00. In the summer I work different hours because from 15th June we
(26) ……………to the summer timetable. It’s very hot in Rome in August, so most people go on holiday.”
22. A. working B. I’m work C. I work D. worked
23. A. from B. on C. between D. in
24. A. finish B. finishing C. to finish D. finishes
25. A. from B. until C. at D. for
26. A. change B. go C. get D. come
Read the following passage and mark the letter A, B or C to indicate the correct answer to each of the
questions.
Hello, my name is John. I live in an apartment in New York, the USA. It is a big city. It has got a lot
of high buildings, supermarkets, banks and stores. There is much traffic and there are often traffic jams at
rush hours. It is very noisy here. But I enjoy living here because everything is exciting and attractive to me.
Hi. My name is Kate. I am John’s elder cousin. I live in a village in the suburbs of Canberra, Australia. It is
quieter than New York. It hasn’t got many shops, supermarkets or restaurants. There are some farms around
my house. You can see a lot of sheep and kangaroos on the farms. There are not many people, but they are
friendly. I prefer to live in the countryside because I enjoy the fresh air and the peaceful life here.
27. New York has got many high buildings, banks, supermarkets and …………...
A. parks B. post office C. stores
28. It’s very…………….in New York City.
A. boring B. peaceful C. noisy
29. Kate is…………….than John.
A. older B. younger C. taller
30. She lives in a…………….in an area away from Canberra, Australia.
A. village B. town C. city
31. She prefers to live in the countryside because the air is…………….there.
A. fresh B. dirty C. polluted
Mark the letter A, B, C, or D that completes the sentence from the suggested words or phrases.
32. It/be/interesting/read/comics/free time.
A. It’s interesting reading comics in (your) free time.
B. It’s interesting to read comics at (your) free time.
C. It’s interesting to read comics in (your) free time.
D. It’s interesting when read comics in (your) free time.
92
33. What/you/have/breakfast/this morning?
A. What do you have for breakfast this morning?
B. What do you have for breakfast in this morning?
C. What did you have for breakfast in this morning?
D. What did you have for breakfast this morning?
34. Minh/not/want/go out/his friend/now.
A. Minh is not want to go out with his friend now.
B. Minh is not wanting to go out with his friend now.
C. Minh does not want to go out with his friend now.
D. Minh do not want to go out with his friend now.
Mark the letter A, B, C, or D to indicate the sentence that best combines each pair of sentences in the
following questions.
35. He got up late. He missed the school bus.
A. He got up late, so he missed the school bus.
B. Because he got up late, so he missed the school bus.
C. Because he missed the school bus, he got up late.
D. Both A & B are correct.
36. I drink 2 glasses of milk every day. My sister drinks 3 glasses of milk every day.
A. I drink more milk than my sister every day.
B. My sister drink more milk than me every day.
C. I drinks more milk than my sister every day.
D. My sister drinks more milk than me every day.
37. Hoa’s mum doesn’t like fish. She also doesn’t like fish.
A. Hoa and her mum doesn’t like fish. B. Hoa and her mum hate fish.
C. Hoa and her mum don’t like fish. D. Both B & C are correct.
Mark the letter A, B, C, or D to indicate the sentence that is closest in meaning to each of the following
questions.
38. How much are the shoes over there?
A. How much does the shoes over there cost?
B. How much do the shoes over there cost?
C. What the shoes over there cost?
D. What the price of the shoes over there?
39. It’s necessary to brush your teeth after meals.
A. You must to brush your teeth after meals.
B. You should to brush your teeth after meals.
C. You need to brush your teeth after meals.
D. You has to brush your teeth after meals.
40. No one in my team is more careful than him.
A. He is the best careful in my team. B. He is the carefuliest in my team.
C. He is the most careful in my team. D. He is most careful in my team.

93
PRACTICE TEST 8
Mark the letter A, B, C or D to indicate the word whose underlined part differs from the other three in
pronunciation in each of the following questions.
1. A. how B. hour C. hat D. house
2. A. date B. name C. April D. apple
Mark the letter A, B, C or D to indicate the word that doesn’t belong to the group.
3. A. dozen B. packet C. bottle D. box
4. A. onion B. pea C. tomato D. apple
Mark the letter A, B, C or D to indicate the underlined part that needs correction in each of the
following questions.
5. The most suitable apartment is near to the city center.
A B C D
6. Her younger sister doesn’t like do housework.
A B C D
7. There are a large, modern bathroom and a kitchen in the apartment.
A B C D
Mark the letter A, B, C or D to indicate the most suitable response to complete each of the following
exchanges.
8. Hello! How can I help you?
-………….
A. I’m sorry, I can’t.
B. I don’t need help.
C. I’m looking for a watch as a birthday present.
D. both B&c are correct.
9. “Thank you for inviting me.”
-“………….”
A. That’s fine B. Ok
C. Don’t worry D. You are welcome
Mark the letter A, B, C or D to indicate the correct answer to each of the following questions.
10. We use a………….to wash clothes.
A. washing machine B. washer machine
C. dishwasher D. clothes washer
11.………….is a picture on the wall.
A. That B. It C. This D. There
12. Put these dirty socks in the…………..
A. basket for washing B. washing basket
C. washer’s basket D. wash basket
13. What………….you on your last birthday?
A. did your father give B. has your father given
C. does your father give D. had your father given
14. This is the most interesting novel…………
A. I am reading B. I’ve never read C. I’ve ever read D. I read
15. Would you like to come to my house for lunch? -………….
A. Yes, I like B. OK. I’d like C. I’d love to D. Yes, I do
16. He was sitting on the beach when he………… a noise.
A. heard B. was hearing C. hears D. has heard
17. You should always keep your bedroom…………
A. tidily B. tidier C. more tidy D. tidy
94
18. We learn…………to use a computer.
A. how B. what way C. a way D. the way
19. What did Jane…………about her next dinner party?
A. tell B. say C. speak D. talk
Mark the letter A, B, C or D to indicate the word(s) CLOSEST in meaning to the underlined word(s)
in each of the following questions.
20. English is my son’s favorite subject.
A. important B. difficult C. interesting D. beloved
21. I think people shouldn’t say rude words.
A. ugly B. polite C. impolite D. useless
Read the following passage and mark the letter A, B, C or D to indicate the correct word or phrase
that best fits each of the numbered blanks.
Tokyo, which is the (22)…………….city of Japan, is one of the three largest cities in the world. (23)
…………….the twelfth century, Japan’s capital was an island city named Kyoto. Six hundred years later,
many people (24) ……………. Kyoto moved east to a city called Yedo, renamed it Tokyo, and (25) ………
this city the capital of all Japan, which is still is. Today, Tokyo is a (26) …………….place. It is a center for
business and education. It is also a center for religion.
22. A. capital B. town C. country D. street
23. A. On B. In C. At D. Under
24. A. of B. from C. to D. off
25. A. making B. makes C. made D. cause
26. A. crowded B. noisy C. noise D. busy
Read the following passage and mark the letter A, B, C or D to indicate the correct answer to each of
the questions.
London, which is the capital city of the United Kingdom, was founded by the Romans in 43 AD. The
number of people living in London is about seven million. Located in Northern England, on the river
Thames, the city covers an area of 1610 square kilometers. London is famous for many things. Tourists come
from all over the world to visit its historic buildings, such as St. Paul’s Cathedral, and the Houses of
Parliament. They also come to visit its theatres, its museums and its many shops. The most famous shop is
Harrods, where you can buy almost everything. The best thing about London is the parks. There are five of
them in the city centre. They cover a large area of the city; and there are ideal places for people to get away
from noise and busy city life and relax.
27. What is the population of London?
A. Three million. B. Four million.
C. Seven million. D. Seventeen million.
28. What is the most famous shop in London?
A. St. Paul’s Cathedral B. Houses of Parliament
C. Harrods D. Me Donald
29. What does the word its in line 6 refer to?
A. London’s B. United Kingdom’s
C. Northern England D. River Thames’
30. How many parks are there in the city centre?
A. seven B. six C. five D. four
31. What is the most wonderful about London?
A. Theatres B. Museums C. Shops D. Parks
Mark the letter A, B, C, or D that completes the sentence from the suggested words or phrases.
32. It/not/be/convenient/live / far / school.
A. It’s not convenient living far school.
95
B. It’s not convenient living far from school.
C. It’s not convenient to live far from school.
D. It’s not convenient to live far school.
33. How/your father/travel/work/every day?
A. How your father travels to work every day?
B. How is your father travel to work every day?
C. How often does your father travel to work every day?
D. How does your father travel to work every day?
34. Minh/not/want/go out/his friend/now.
A. Minh is not want to go out with his friend now.
B. Minh is not wanting to go out with his friend now.
C. Minh does not want to go out with his friend now.
D. Minh do not want to go out with his friend now.
Mark the letter A, B, C, or D to indicate the sentence that best combines each pair of sentences in the
following questions.
35. We came to visit her last night. She was watching TV.
A. She watched TV while we came to visit her last night.
B. When we came to visit her last night, she was watching TV.
C. She was watching TV when we were coming to visit her last night.
D. Last night we came to visit her, she watched TV.
36. My mother is very busy with her work at the office. She still takes care of us.
A. My mother is so busy with her work at the office that she cannot take good care of us.
B. Because my mother is so busy with her work at the office, she takes good care of us.
C. My mother is so busy with her work at the office to take good care of us.
D. In spite of being very busy with her work at the office, my mother takes good care of us.
37. Jane doesn’t like coffee. Her husband doesn’t, either.
A. Both Jane and her husband likes coffee.
B. Neither Jane or her husband likes coffee.
C. Neither Jane nor her husband likes coffee.
D. Not only Jane but also her husband like coffee.
Mark the letter A, B, C, or D to indicate the sentence that is closest in meaning to each of the following
questions.
38. It’s very important to spend time doing exercises.
A. You should spend time doing exercises.
B. You must spend time doing exercises.
C. Spending time doing exercises is very important to me.
D. Spending time doing exercises is very important.
39. She is a fast runner.
A. She runs fastly. B. She runs fast.
C. She runs faster than me. D. She is the fastest runner.
40. Because my father is tired of living in the city, he wants to live in a quiet village.
A. My father is tired to live in the city and wants to live in a quiet village.
B. My father is tired of living in the city, so he wants to live in a quiet village.
C. My father is tired from living in the city and he wants to live in a quiet village.
D. My father is tired of living in the city and wanting to live in a quiet village.

96
PRACTICE TEST 9
Mark the letter A, B, C or D to indicate the word that doesn’t belong to the group.
1. A. flower B. restaurant C. tree D. park
2. A. favorite B. hungry C. want D. thirsty
Mark the letter A, B, C or D to indicate the word whose underlined part differs from the other three in
pronunciation in each of the following questions.
3. A. author B. biology C. nervous D. comfortable
4. A. hurry B. bus C. sugar D. supper
Mark the letter A, B, C or D to indicate the underlined part that needs correction in each of the
following questions.
5. Hung spends most of his free time playing computer’s games.
A B C D
6. People are living longer so there are more and more many old people .
A B C D
7. One of the world’s largest libraries are the United States’ Library of Congress.
A B C D
Mark the letter A, B, C or D to indicate the most suitable response to complete each of the following
exchanges.
8. How nice your new schoolbag is!
A. I bought it yesterday. B. Thanks a lot.
C. It’s a present from my father. D. You’re welcome.
9. Your room is so beautiful.
- “………….”
A. I’m glad you like it B. No problem
C. My room is very large too D. I think it needs some pictures
Mark the letter A, B, C or D to indicate the correct answer to each of the following questions.
10. Who is…………..girl standing over there?
A. Ø B. those C. the D. a
11. ‘How old is your brother?’
A. His age is 20 years B. He has 20 years of age
C. His age is 20 D. He’s 20 years old
12. Come in and…………..a seat.
A. have B. enjoy C. do D. give
13. Which is the ninth month of the year? - …………...
A. October B. November C. September D. February
14. There will be a farewell party for him…………...the twenty-eighth.
A. at B. for C. on D. in
15. Which sentence is correct?
A. What color does his new bicycle have?
B. What color of his new bicycle is it?
C. What color has his new bicycle?
D. What color is his new bicycle?
16. It’s difficult…………...his fast English.
A. for me understanding B. understanding
C. to understand D. for understanding
17. People have nothing to do in their…………...time.
A. amusement B. leisure C. fun D. entertainment
18.…………..sports do you like best?
97
A. What sort B. What the kind of C. What D. What kind
19. The pizza restaurant is often.………….of their customers.
A. busy B. friendly C. crowded D. full
Mark the letter A, B, C or D to indicate the word(s) CLOSEST in meaning to the underlined word(s)
in each of the following questions.
20. He rides his bicycle to school every day.
A. biking B. cycles C. drives D. flies
21. I have taken up collecting stamps since 2018.
A. taken over B. finished C. begun D. done
Read the following passage and mark the letter A, B, C or D to indicate the correct word or phrase
that best fits each of the numbered blanks.
One of the first novels in the history of literature (22)……………written in England in 1719. It was
Robinson Crusoe by Daniel Defoe. Daniel Defoe was born in London in the family of a rich man. When
Daniel was a schoolboy, he began to write stories. After (23) ……………school, he worked in his father’s
shop and wrote articles for newspapers. Defoe visited many countries and met many people. That helped him
(24) ……………in his writings. In 1719, when Defoe was sixty years old, he wrote the novel Robinson
Crusoe which made him famous. Defoe used in his book a true story about a sailor who (25)……………on
an island for four years. Robinson Crusoe in Defoe’s novel lived on an island for twenty-eight years. People
liked the novel in England and in many other countries, Daniel Defoe wrote other books. (26) ……………his
novel Robinson Crusoe was the most famous. Defoe was not a rich man when he died in 1731.
22. A. were B. has C. have D. was
23. A. leaving B. coming C. going D. staying
24. A. much B. few C. plenty D. many
25. A. visited B. went C. lived D. came
26. A. Because B. So C. However D. And
Read the following passage and mark the letter A, B, C or D to indicate the correct answer to each of
the questions.
Lan and Nga want to know their friends’ favorite freetime activities, so they are asking their friends
about what they like to do in their free time. Many of their friends say that they usually eat and drink as well
as chat with their friends. They like to talk about many things around them: their teachers, their friends, their
families as well as their favorite musicians, singers, soccer players and movie stars. There are some popular
activities after lessons at their school such as skipping rope, reading books and playing sports. At home,
many of their friends say they like to watch TV, listen to music or play computer games. They rarely go to
the movies. Some of the girls like to go shopping. Very few of their friends like to do English exercises on
the Internet. Many of their friends don’t take part in art, music club.
27. What do many of Lan and Nga’s friends usually do in their free time?
A. do English exercises on the Internet
B. go to the movie
C. eat, drink and chat
D. go shopping
28. What do their friends like to talk about?
A. their teachers, friends and families
B. their favorite soccer players and movie stars
C. many tilings around them
D. their favorite musician and singers
29. Which of the following activities is NOT popular with their friends at school?
A. Playing sports B. Reading books
C. Skipping rope D. Take part in art, music club
98
30. Which of the following activities is NOT popular with their friends at home?
A. Do English exercises on the Internet B. Listening to music
C. Playing computer games D. Watching TV
31. What does the passage talk about?
A. Nga and Lan’s friends.
B. Free-time activities of Nga and Lan’s friends.
C. Nga and Lan’s school.
D. Nga and Lan’s pastimes.
Mark the letter A, B, C, or D to indicate the meaningful sentence written by using the word(s) or
phrase(s) given in each of the following questions.
32. Last year/we/spend/week/Ha Long/and/I/not want/leave.
A. Last year we spent a week in Ha Long and I did not want to leave.
B. Last year we spend a week in Ha Long and I did not want to leave.
C. Last year we spent week in Ha Long and I did not want to leave.
D. Last year we spent a week in Ha Long and I did not want leaving.
33. What time/you/go to bed?
A. What time does you go to bed? B. What time do you go to bed?
C. What time are you go to bed? D. What time you go to bed?
34. Ba/usually/play tennis/fall/but/sometimes/sailing.
A. Ba usually play tennis in the fall but he sometimes go sailing.
B. Ba usually playing tennis in the fall but he sometimes go sailing.
C. Ba usually plays tennis in the fall but he sometimes goes sailing.
D. Ba usually plays tennis in the fall but he sometimes go sailing.
Mark the letter A, B, C, or D to indicate the sentence that best cornbifc each pair of sentences in the
following questions.
35. The box is very heavy. You can’t carry it.
A. The box is so heavy for you to carry.
B. The box is too heavy for you to carry.
C. You are not strong enough to carry the box.
D. Both B&C are correct.
36. Phong is intelligent. He is very hard- working too.
A. Phong is intelligent, but he is very hard - working too.
B. Phong is intelligent, but he is not very hard - working.
C. Phong is not only intelligent but also very hard - working.
D. Phong is not very hard- working but he is intelligent.
37. The picture was so popular. I did not think so.
A. The picture was not as popular as I thought.
B. The picture was less popular than I thought.
C. I thought the picture was very popular.
D. The picture was more popular than I thought.
Mark the letter A, B, C, or D to indicate the sentence that is closest in meaning to each of the following
questions.
38. The children laughed a lot because of the funny story.
A. The children laughed because it was funny.
B. The children laughed because the story is funny.
C. The children laughed because of the story funny.
D. The children laughed a lot because the story was funny.
39. He is very rich, but he isn’t very happy.
99
A. Although he is very rich, but he isn’t very happy.
B. Although he is very rich, he isn’t very happy.
C. Although he isn’t very rich, but he is very happy.
D. Because he is very rich, he is very happy.
40. It’s necessary for children to learn to swim.
A. Learn to swim is necessary for children.
B. Learning to swim necessary for children.
C. Learning to swim is necessary for children.
D. Children need learning to swim.

100
PRACTICE TEST 10
Mark the letter A, B, C or D to indicate the word that doesn’t belong to the group.
1. A. lettuce B. carrot C. tomato D. chicken
2. A. drugstore B. bookstore C. bakery D. beautiful
Mark the letter A, B, C or D to indicate the word whose underlined part differs from the other three
in pronunciation in each of the following questions.
3. A. concert B. can C. ocean D. comic
4. A. chamber B. technology C. chemistry D. architect
Mark the letter A, B, C or D to indicate the underlined part that needs correction in each of the
following questions.
5. Will you describe about your apartment ?
A B C D
6. Her new school have a lot of students.
A B C D
7. Hoa’s the new student in our class.
A B C D
Mark the letter A, B, C or D to indicate the most suitable response to complete each of the following
exchanges.
8. “ I’m going to have a holiday in France with my family”
- ……………
A. Do well. B. Lucky you!
C. You must be careful. D. It’s nice of you.
9. “ I feel bored because everything is going wrong” - “……………”
A. Not at all B. It’s a bad thing
C. That’s all D. Things will be better
Mark the letter A, B, C or D to indicate the correct answer to each of the following questions.
10. Is this your first time in Hanoi? - No, it isn’t. It’s……………time.
A. second B. number two C. the second D. twice
11. What do you call your sister’s son? -…………….
A. Cousin B. Nephew C. Niece D. Husband
12. Mai’s new school……………a large…………….
A. have/librarys B. has/library
C. are having/librarys D. is having/library
13. Next summer I am working as a……………teacher in SOS village in Viet Tri City.
A. favorite B. nice C. volunteer D. good
14. Minh is always……………at school, and helps other students with their homework.
A. work hard B. hard C. hard-working D. hardly
15. Please be……………when you run on the edge of the pool.
A. happy B. funny C. careful D. quiet
16. There is a big photo of their family……………their room’s wall.
A. between B. on C. in D. among
17. Mai is……………the phone, chatting……………friends.
A. on - on B. to - to C. to - with D. on - to
18. The boy next to me is very……………He talks all the time, which really annoys me.
A. silent B. kind C. talkative D. lovely
19. My bedroom is……………my parents’ room.
A. under B. next to C. on D. in
Mark the letter A, B, C or D to indicate the word(s) CLOSEST in meaning to the underlined word(s)
101
in each of the following questions.
20. Could you pleased turn off the TV ?
A. shut down B. shut up C. plug in D. turn on
21. Once you’ve finished cleaning , you can go on with your work.
A. examine B. make a note C. consider D. continue
Read the following passage and mark the letter A, B, C or D to indicate the correct word or phrase
that best fits each of the numbered blanks.
My favourite hobby is coin (22)…………... It is also one of the oldest (23) …………...of the world
and is also called ’hobby of the Kings’, as in the old times, it was only done by kings and rich class of the
society. It started for me some years ago when my father usually came back (24) …………...his foreign tours
and (25) …………...me coins of different countries. I began collecting them in my money box and after that
I always waited for my new coins as much I used to wait for my father. Since then, it has become my
favourite hobby. This hobby has given me information about different countries of the world. I feel excited
when showing my collection to (26) …………...people.
22. A. collecting B. collection C. collected D. collectors
23. A. hobby B. hobbies C. game D. games
24. A. after B. in C. from D. of
25. A. gives B. gave C. giving D. give
26. A. other B. another C. others D. difficult
Read the following passage and mark the letter A, B, C or D to indicate the correct answer to each of
the questions.
THE HISTORY OF THE LONDON POLICE
Today there are policemen everywhere, but in 1700 London had no policemen at all. A few old men
used to protect the city streets at night and they were not paid very much. About 300 years ago, London was
starting to get bigger. The city was very dirty and many people were poor. There were so many thieves who
stole money in the streets that people stayed in their homes as much as possible.
In 1750, Henry Fielding started to pay a group of people to stop thieves. They were like policemen
called Bow Street and were Runners’ because they worked near Bow Street. Fifty years later, there were 120
Bow Street Runners, but London had become very big and needed more policemen. So, in 1829, the first
Metropolitan (or London) Police Force was started with 3000 officers. Most of the men worked on foot, but a
few rode horses. Until 1920 all the police in London were men. Today, London police are quite well paid and
for the few police officers who still ride horses, the pay is even better than for the others.
27. 300 years ago, many people ………….
A. came to live in London B. had big houses in London
C. wanted to live in London D. were very rich in London
28. People did not leave their houses because………….
A. the city was not clean B. they were afraid of losing money
C. they had no money D. the city was bigger
29. The Bow Street Runners………….
A. stole money B. stopped people stealing
C. paid people to steal D. gave money to the thieves
30. In 1800, there were…………..
A. enough policemen B. not enough policemen
C. too many policemen D. only few policemen
31. Of the first 3000 Metropolitan policemen,…………...
A. all of them rode horses B. some of them rode horses
C. most of them rode horses D. none of them rode horses
Mark the letter A, B, C, or D to indicate the meaningful sentence written by using the word(s) or
102
phrase(s) given in each of the following questions.
32. Huong/not/usually/go out/her friends/Sunday.
A. Huong isn’t go out with her friends on Sunday.
B. Huong doesn’t go out with her friends in Sunday.
C. Huong doesn’t usually go out with her friends on Sunday.
D. Huong isn’t going out with her friends in Sunday.
33. Last night/they/decide/have/party/their friends.
A. Last night, they decide to have a party with their friends.
B. Last night, they decided to have a party with their friends.
C. Last night, they decide having a party with their friends.
D. Last night, they decided to having a party with their friends.
34. Mai/often/practice/speak/English/her friend/class.
A. Mai often practices speak English with her friend(s) in class.
B. Mai often practices to speak English with her friend(s) in class.
C. Mai often practices speaking English with her friend(s) in class.
D. Mai often practice speaking English with her friend(s) in class.
Mark the letter A, B, C, or D to indicate the sentence that best combines each pair of sentences in the
following questions.
35. Richard is very wealthy. He can afford to buy almost anything he wants.
A. Richard is so wealthy that he can afford to buy almost anything he wants.
B. Although Richard is very wealthy, he can afford to buy almost anything he wants.
C. Richard is very wealthy so that he can afford to buy almost anything he wants.
D. Richard can afford to buy almost anything he wants, but he is very wealthy.
36. Peter likes football. He doesn’t play it too often.
A. Peter likes football so that he doesn’t play it too often.
B. Although Peter likes football, he doesn’t play it too often.
C. Although Peter likes football but he doesn’t play it too often.
D. Peter likes football so he doesn’t play it too often.
37. The town was beautiful. We spent our holiday there last month.
A. The town in there we spent our holiday last month as beautiful.
B. The town was beautiful which we spent our holiday there last month.
C. The town where we spent our holiday last month was beautiful.
D. The town which we spent our holiday there last month was beautiful.
Mark the letter A, B, C, or D to indicate the sentence that is closest in meaning to each of the following
questions.
38. We are used to travelling by bus.
A. We like travelling by bus.
B. We are interested in travelling by bus.
C. We are accustomed to travelling by bus.
D. We are fond of travelling by bus.
39. The house is so large that they can’t paint it in a week.
A. If the house were large, they could paint it in a week.
B. If the house isn’t large, they could paint it in a week.
C. If the house weren’t large, they couldn’t paint it in a week.
D. Unless the house were large, they could paint it in a week.
40. Somebody repaired her bicycle last week.
A. She had to repair her bicycle last week.
B. She had her bicycle repaired last week.
103
C. She had her bicycle repair last week.
D. She had her bicycle to repair last week.

104
HƯỚNG DẪN GIẢI
TRƯỜNG THPT CHUYÊN
HÀ NỘI - AMSTERDAM
PRACTICE TEST 1
I. PHONETICS
1.1. 1.2.
1.D 2. B 3. B 1. A 2. C
II. READING COMPREHENSION
2.1.
1. A 2. C 3. C 4. C 5. A
2.2.
1. B 2. C 3. C 4. B 5. C 6. B 7. D 8. D 9. A 10. A
III. GRAMMAR AND VOCABULARY
3.1.
1. B 2. C 3. D 4. B 5. A 6. D 7. B 8. B 9. B 10. D
3.2. 3.3. a 3.3. b
1. C 2. C 1. D 2. C 1. C 2. A
3.4.
1. traditionally 2. healthier 3. dishonest 4. Deforestation
IV. WRITING
4.1.
1. We spent three hours finding a room for the night.
2. Would you prefer me to stay with you during the holidays?
3. The English test was so difficult that I could not do it well.
4. The police accused Jim of having stolen the money.
5. His illness made it impossible for him to work effectively./His illness made him impossible to work
effectively.
4.2.
1. When it is hot, he goes swimming/for a swim in the river in front of his house.
2. She usually listens to music at night.
3. This coffee is so hot that I cannot/can't drink it.
4. Do you know who is (the) best at English in your grade?
Who do you know is…………..
5. Air pollution is a serious problem in many big cities.
Notes:
2.1.
• in other words (phr): nói cách khác
• to be (in) capable of + doing sth: (không) có khả năng làm gì
• do the leaving = pass away= die: qua đời
2.2.
• carry on + doing sth: tiếp tục làm gì • make demand on (phr): có nhu cầu về
• consequence(n): hậu quả • consumption (n): sự tiêu thụ
• shelter(n): chỗ trú ẩn • either .... or....hoặc....hoặc
• as a result (ph.r.): do đó, kết quả là • run out (of sth) (phr.v): cạn kiệt cái gì
3.1.
• generally speaking (phr): nói chung
105
• provided (conj) = providing: miễn là, với điều kiện
3.3.
• set in (phr.v): bắt đầu (thời tiết, thường là bắt đầu xấu)
• Winter has set in: Mùa đông đã bắt đầu.
• It sets in to rain: Trời bắt đầu mưa.
3.4.
• dishonest (adj): không trung thực • deforestation (n): sự phá rừng
4.1.
• It + takes + sb + time + to do sth = Sb + spend time + doing sth: ai mất bao nhiêu thời gian làm gì
• to accuse sb of + doing sth: buộc tội ai đã làm gì
• make it impossible for sb to do sth: làm cho ai không thể làm gì
• so + adj + that + S+V: .... đến mức
The English test was so difficult that I could not do it well: Bài kiểm tra tiếng Anh khó đến mức tôi không
thể làm tốt được.

PRACTICE TEST 2
I. PHONETICS
1.1. 1.2.
1. D 2. D 1. C 2. A 3. D
II. READING COMPREHENSION
2.1.
1. D 2. B 3. A 4. B 5. B 6. C 7. A 8. B 9. D 10. C
2.2.
1. D 2. A 3. C 4. B 5. C
III. GRAMMAR AND VOCABULARY
3.1.
1. B 2. C 3. D 4. B 5. C 6. D 7. C 8. B 9. C 10. A
3.2.a 3.2.b 3.3.
1. D 2. D 1. B 2. A 1. D 2. A
3.4.
1. popularity. 2. admirable. 3. misunderstood. 4. preservation.
IV. WRITING
4.1.
1. The nearer New Year comes, the busier people are.
2. Neither Peter nor John likes tea.
3. Unless Peter is invited to the party, Mary will not (won't) come.
4. Although the weather was bad, we went swimming.
5. He apologizes for not coming/having come to the meeting yesterday.
4.2.
1. The factory closed because the workers went on strike last week.
2. It is important that your brother (should) stop smoking at once.
Or: It is important for your brother to stop smoking at once.
3. The policeman ordered the children not to play football in/ on the street.
4. She will/ is going to make another attempt to climb that/ the mountain next year.
5. It is/ was wrong of you not to give her a chance to talk.
Notes:
1.1.
106
• passion (n): niềm đam mê • threaten (v): đe dọa
1.2.
incidence(n): phạm vi ảnh hưởng • tendency(n): xu hướng
2.1.
• to be made up of = to be comprised of: gồm có
• evaporation (n): sự bay hơi
• territory(n): lãnh thổ
• precipitation(n): lượng mưa
• moisture(n): hơi ẩm
• absorb (v): hấp thụ
• dune(n): đụn cát
• drizzle(n,v): mưa phùn
• Plants use their roots to absorb moisture from the soil: Cây sử dụng rễ của chúng để hút hơi ẩm từ đất.
• landscape (n): phong cảnh
• atmosphere (n): khí quyển, bầu không khí
2.2.
• attack(n,v): tấn công • evil spirits(n): tà ma
• to keep sb/sth away (phr.v): xua đuổi ai/cái gì
3.1.
• disapprove sb of + doing sth: không tán thành ai làm gì
• criticize sb for + doing sth: chỉ trích ai làm gì
• object sb to + doing sth: phản đối ai làm gì
• would prefer to do sth rather than do sth else: thích làm gì hơn làm gì khác
• show off (phr.v) khoe khoang
• to be used to doing sth (phr): quen làm gì
• attitude (n): thái độ
• punctuality (n): sự đúng giờ
3.2.
• absent-minded (adj): đãng trí
• Old people are often absent-minded: Người già thường hay đãng trí.
• cruelly (adv): một cách độc ác > < gently
• brutally (adv) = cruelly
3.4. misunderstand (v): hiểu lầm
4.1. neither………..nor……….. : không………..mà cũng không………..
apologize (to sb) for (not) doing sth: xin lỗi (ai) vì đã (không) làm gì
4.2
• go on strike (phr) đình công
• at once = immediately (adv): ngay lập tức
• order sb (not) to do sth: ra lệnh, yêu cầu ai (không) làm gì
• It is + important + for sb + to do sth = It is important that sb (should) do sth
• It is important for your brother to stop smoking = It is important that your brother (should) stop smoking.
Điều quan trọng là anh trai bạn nên bỏ thuốc.
• It is/ was wrong of you not to give her a chance to talk. Bạn đã sai khi không cho cô ấy cơ hội nói.

107
PRACTICE 3
I. PHONETICS
1.1. 1.2.
1. A 2. A 3. C 1. D 2. C
II. READING COMPREHENSION
2.1.
1. B 2. B 3. A 4. D 5. C
2.2.
1. A 2. C 3. A 4. C 5. D 6. D 7. C 8. B 9. C 10. C
III. GRAMMAR AND VOCABULARY
3.1.
1. C 2. B 3. B 4. D 5. D 6. A 7. B 8. B 9. A 10. A
3.2.
1. unmanageable 2. outspoken 3. personality 4. economics
3.3. a 3.3. b 3.4.
1. D 2. B 1. A 2. D 1. B 2. C
IV. WRITING
4.1.
1. Tim doesn’t take after his father at all.
2. I wish I hadn’t given Dennis my phone number.
3. Alison asked her mother if there wasn’t anything she wanted from the shops.
4. Oh bother, we’ve run out of milk.
5. Customers are advised to buy their tickets in advance.
4.2.
1. Many people are fond of going to the countryside on/ at the weekends.
2. It took her two hours to finish such a difficult test.
3. I was invited to her party on the occasion of her birthday last week.
4. Tet is a/the time when Vietnamese people celebrate the beginning of the spring. Tet is a/the time for
Vietnamese people to celebrate the beginning of the spring.
5. The telephone was invented by Alexander Graham Bells.
Notes:
2.1.
• aggression (n): sự hung hăng, sự gây gổ • aggressive (adj): hung hang , gây gổ
• Aggressive nations threaten world peace: Những quốc gia hung hăng đe dọa hòa bình thế giới.
• psychological (adj): thuộc về tâm lý • stability (n): sự ổn định
• highlight (v): nhấn mạnh • depressed (adj): chán nản, trầm cảm
2.2.
• human-beings (n): con người
• derive from (phr.v): có nguồn gốc , bắt nguồn từ
• on the other hands (phr): mặt khác
• on behalf of (phr): thay mặt cho
• by the way (phr): à này, tiện thể
3.1.
• take over (phr.v): tiếp quản
• get sth done = have sth done (phr): nhờ làm hộ
• I got/had my car repaired by a mechanic.Tôi đã nhờ một người thợ máy sửa chiếc xe.
• remember doing sth: nhớ đã làm gì
3.4.
108
• try on (phr.v) thử (giày, dép, mũ, nón, trang phục,...)
• Try on the shoes before you buy them: Trước khi mua giày, hãy đi thử đã.
• a matter of life and death (phr): một vấn đề sống còn
• by all means = of course (phr): dĩ nhiên
4.1.
• take after (phr.v): giống
• regret doing sth : ân hận đã làm gì
• on the occasion of (phr): nhân dịp

PRACTICE TEST 4
I. PHONETICS
1.1. 1.2.
1.D 2. D 3. A 1. C 2. A
II. READING COMPREHENSION
2.1.
1. B 2. A 3. C 4. D 5. B 6. A 7. A 8. D 9. B 10. A
2.2.
1. B 2. A 3. A 4. D 5. A
III. GRAMMAR AND VOCABULARY
3.1.
1. A 2. D 3. B 4. D 5. D 6. D 7. C 8. A 9. D 10. B
3.2. a 3.2. b 3.3.
1. B 2. B 3. A 4. C 1. C 2. D
3.4.
1. troublesome 2. speechless 3. impractical 4. generosity
IV. WRITING
4.1.
1. We spent three hours opening the door.
2. In spite of admiring her achievement, I don’t really like her.
In spite of the fact that I admire her achievement,
3. My English friend isn’t used to (using) chopsticks.
4. George told Mary that he had seen the film three times.
5. My brother didn’t use to study as hard as he does now.
4.2.
1. You won’t pass the/your coming exam unless you work hard.
2. James Watt, (who was) a Scottish scientist, invented the steam engine.
3. Sometimes a country refuses to take part in the Olympics.
4. I apologised to him for not being able to arrive on time.
5. Mars, which is 140 million miles, is known as the red planet.
Notes:
2.1.
• to be full (of sb/sth): đầy
• now that (conj): bởi vì, vì rằng
• Now (that) the weather is warm, you can go out = Vì thời tiết đã ấm hơn, anh có thể ra đi.
2.2.
• apart from (prep) = except for: ngoại trừ, trừ ra, ngoài ra
• fancy costumes (n): trang phục dạ hội
109
• shape (n): hình dạng
• shop for sth (phr.v): khảo giá cái gì
3.1.
• contribute to (doing) sth: đóng góp, góp phần vào (làm) việc gì
• Good health contributed to his success: Sức khỏe tốt đã giúp anh ta thành công.
• bring up (phr.v): nuôi nấng
• adj + as + S + V = cho dù ....
• Rich as he was, he felt so unhappy and lonely. Cho dù anh ta giàu, anh ta vẫn cảm thấy rất bất hạnh và cô
đơn.
3.2
• to be proud (of sb/sth): tự hào về
• to be like a dog with two tails (idm): cực kỳ sung sướng
• to be able to do sth = can do sth: có thể làm gì
3.3. I couldn’t agree more./I quite agree. - I agree completely, (phr): Tôi hoàn toàn đồng ý.

PRACTICE TEST 5
I. PHONETICS
1.1. 1.2.
1. A 2. B 1. C 2. A 3. A
II. READING COMPREHENSION
2.1.
1. C 2. D 3. B 4. B 5. C
2.2.
1. D 2. B 3. B 4. C 5. C 6. B 7. C 8. A 9. B 10. A
III. GRAMMAR AND VOCABULARY
3.1.
1. A 2. C 3. D 4. A 5. A 6. C 7. C 8. A 9. C 10. C
3.2.
1. unavoidable 2. fashionably 3. imaginative 4. Curiosity
3.3. a 3.3. b 3.4.
1. C 2. B 3. D 4. C 1. D 2. C
IV. WRITING
4.1.
1. Bill Gates was born in 1955 in Washington State. He grew up in a rich family.
2. His parents sent him to a private school. There he met his business partner, Paul Allen.
3. When they were in the 8th/eighth grade, they wrote programs for business computers.
4. In 1973, Gates was accepted at Harvard University. His parents were happy.
5. Two years later, Gates dropped out of Harvard to work on a computer program with his friend Allen.
6. In 1975, they created a company called Microsoft to sell their product.
7. A few years later, Microsoft became a giant company.
8. By 1990, “Gates was the youngest billionaire of the United States at the age of 34/thirty-four.
9. He achieved his success with a lot of hard work. He was the “King of Software”.
10. By 1997, he was the richest man in the United States.
4.2.
1. If he were strong enough, he could lift the table.
2. Her brother wishes he could speak English as fluently as a native speaker.
3. She isn’t used to driving a car very fast.
4. The city is reported to have increasingly developed in the past few years.
110
5. My sister would rather stay at home and watch TV than go to the cinema.
Notes:
2.1.
• in terms of (phr): xét về mặt • turn up (phr.v)= arrive, appear: đến, xuất hiện
• migrate(v): di cư
• go to some/great/any lengths ( to do sth) (idm): làm bất cứ cái gì có thể, không e dè câu nệ
• Bella would go to any lengths to fulfil her ambition: Bella sẽ làm bất cứ điều gì để hoàn thành tham vọng
của cô.
• desert (v): bỏ mặc • vulnerable (adj): dễ bị tổn thương
• The young birds are very vulnerable to predators: Những con chim non rất dễ bị thú ăn thịt làm hại.
2.2.
• all - round (adj): toàn diện • have doubts about (phr): hoài nghi về
• do sb good (idm): có lợi cho ai • take up (phr.v) = start liking bắt đầu một sở thích
• at least (phr): ít nhất • advantage (n): thuận lợi > < disadvantage (n) bất lợi
3.3.
• offspring (n): con cháu • interference (n): sự can thiệp
4.1. drop out of (phr.v): bỏ học giữa chừng
4.2. prefer doing sth to doing sth else = would rather do sth than do sth else: thích làm gì hơn làm gì khác =
would prefer to do sth rather than do sth else

111
TRƯỜNG THCS NGOẠI NGỮ
PRACTICE TEST 1
Part I.
1. science fiction 3. mother tongue 5. traffic jam
2. brother-in-law 4. box office
Part II.
1. D 2. B 3. A 4. B 5. A 6. B 7. C 8. D 9. D 10. C
Part III.
1. B 2. A 3. I 4. E 5. J 6. G 7. D 8. F 9. C 10. H
Part IV.
1. countries 2. meet 3. differences 4. common 5. pronunciation
6.from 7. other 8. matter 9.languages 10. likely
Part V.
1. A 2. C 3. B 4. D 5. B
Part VI.
1. Sue is looking forward to going to Spain on hoiliday.
2. I haven’t made up my mind/a decision to continue my study in a foreign country
3. It was foolish of him to say that.
4. The car was so expensive that he couldn’t buy it/too expensive for him to buy
5. It came as no surprise to us (to hear) (that) he was/ (had been) successful/(had) succeeded.
Notes:
Part 1.
• mother tongue(n): tiếng mẹ đẻ
• brother-in-law (n): anh/em rể
• box office(n): phòng vé
• income tax(n): thuế thu nhập
Part 2.
• on account of = because of + noun/Ving • according to + n: theo như: do, vì
• under the weather (idm): khó ở, cảm • You look a bit under the weather: Cậu thấy không khỏe trong
người trông có vẻ hơi mệt.
• safe and sound (idm): bình an vô sự
Part 4.
• to be keen on doing sth: mê say, thích làm gì
• The boy is keen on swimming: Cậu bé thích bơi lội.
Part 5.
• to keep in touch (phr): giữ liên lạc • in person (phr): đích thân, trực tiếp
• The winner will be there in person to collect the prize: Người thắng cuộc sẽ đích thân đến đó lãnh thưởng.
Part 6:
• to be excited about doing sth = to be looking forward to doing sth: mong chờ làm gì
• to decide = to make a decision = to make up one’s mind: quyết định
• I can’t decide / make a decision / make up my mind yet: Tôi chưa thể quyết định.
• It comes/ came as no surprise to sb to do sth / that S+V: Ai đó không ngạc nhiên làm gì / rằng / khi ...
• It came as no surprise when Lester got the job. Chẳng có gì đáng ngạc nhiên khi Lester kiếm được việc làm
đó.
PRACTICE TEST 2
Part I.
1. library 3. commercial centre 5. factory
112
2. suburbs 4. environment
Part II.
1. C 2. A 3. A 4. C 5. D 6. C 7. D 8. B 9. D 10. A
Part III.
l.J 2.A 3.F 4.D 5.C 6.G 7.E 8.B 9.I 10.H
Part IV.
l.best 2.mother’s 3. lives 4.but 5.her
6.is 7.have 8. and 9.for 10.age
Part V.
1. B 2. A 3. C 4. D 5. C
Part VI.
1. It’s very important to keep the environment clean.
2. You are certain/ bound/ sure to meet lots of people in your new job.
You are certainly going to meet lots of people in your new job.
3. She said (that) she was working in a restaurant and she didn’t care much for it.
4. Although the weather was bad, they had a wonderful holiday.
5. They suggested that advertisements on TV should be banned.
Notes:
Part 2.
• put sth into practice (phr): thực hiện cái gì, áp dụng cái gì vào thực tiễn
• under pressure (phr): chịu áp lực, bị sức ép
Part 3.
• put on / gain weight (phr): lên cân > < lose weight: sụt cân
• spend time on sth: dành thời gian vào việc gì
Part 5.
• refer to (phr.v): đề cập tới • analyse (v): phân tích
• prospect (n): triển vọng
• There is no prospect of a settlement of the dispute: Không có triển vọng giải quyết được cuộc tranh chấp.
Part 6.
• It is + adj + to do sth
• It is very important to keep the environment clean. Điều rất quan trọng là giữ cho môi trường sạch
• to be certain/bound/sure to do sth: chắc chắn làm gì
• suggest doing sth = suggest that sth (should) be done gợi ý làm gì

PRACTICE TEST 3
Part I.
1 conscious 3. ambulance 5. bandage
2 first-aid 4. scale
Part II.
1. B 2. C 3. A 4. B 5. A 6. D 7. A 8. C 9. C 10. B
Part III.
l.E 2. H 3. A 4. I 5. J 6. B 7. G 8. F 9. C 10. D
Part IV
1. one 2. when 3. like 4. their 5. put
6. door 7. give 8. next 9. are 10. most
Part V.
1. A 2. C 3. D 4. C 5. A
Part VI.
113
1. The bread was too stale for the children to eat.
2. It’s no use/good crying over spilt milk.
3. George was nowhere to be found.
4. If he doesn’t take these pills, he won’t be better.
5. Because it rained heavily/ hard, we were late for school.
Notes:
Part 2. with a view to doing sth = in order to do sth = so as to do sth (phr): nhằm mục đích làm gì
Part 4. dress up (phr.v): diện bảnh, ăn mặc chải chuốt
Part 5.
• instead of sth/doing sth: thay vì cái gì/làm gì
• (start sth) from scratch: (bắt đầu cái gì) từ đầu , từ con số không, từ bàn tay trắng
Part 6.
• too + adj for sb to do sth: quá ... đến nỗi ai đó không thể làm gì
• There is no point/sense (in) doing sth = It’s no use/good doing sth: làm gì vô ích
• There is no point in crying over spilt milk (idm): Thôi đừng tiếc rẻ con gà quạ tha. (Kêu ca bằng thừa)

PRACTICE TEST 4
Part I.
l.C 2.J 3.A 4.I 5.B 6.G 7.D 8.F 9.H 10.E
Part II.
1. C 2. C 3. B 4. A 5. C 6. A 7. C 8. A 9. B 10. D
Part III.
1. Do you still get wet after the repair?
2. Did they give you anything?
3. What do you think of them?
4. Would you like them to come back?
5. Well, they’ll come back soon, I guess. Well, I have to go. Have a good day.
Part IV.
1. energy 2. rural 3. electricity 4.wastes 5. Students
6.save 7.off 8.bulbs 9. natural 10.coal
Part V.
1. D 2. B 3. C 4. D 5. C
Part VI.
1. Unless you study hard, you will get bad marks.
2. Would you mind not playing your music so loudly?
3. Millions of dollars are spent on advertising every year.
4. No one in his team plays football better than / as well as he does / him.
No one in his team is better at (playing) football than he is / him.
No one in his team is as good at (playing ) football as he is / him.
5. Ingenious as the plan may be, it will never work in practice.
Notes:
Part 2.
• to be supportive of + noun: tán thành, ủng hộ , hỗ trợ
• such + a/an + adj + noun + that + s+ V = đến mức
• It was such a hot day that we decided to stay indoors: Hôm đó trời nóng đến mức mà chúng tôi đã quyết
định ở nhà.
Part 5.
114
• have the custom of doing sth (phr): có phong tục làm gì
• culture (n): văn hóa
• accordingly (adv): theo đó
• add up to sth (phr.v) góp thêm vào cái gì
• to be responsible for doing sth: chịu trách nhiệm làm gì
• You are responsible for paying these bills. Bạn chịu trách nhiệm thanh toán những hóa đơn này.
• in exchange for (phr): đế đổi lấy
• in the wee/small hours (idm) = in the early hours of the morning, just after 12 o’clock at night: lúc tảng
sáng
Part 6:
• Would you mind (not) + doing sth? Cảm phiền bạn (không) làm lộ
• spend money on sth tiêu tiền vào việc gì
• ingenious(adj): mưu trí
PRACTICE TEST 5
Part I.
1. water 3. mirror 5. wardrobe
2. cupboard 4. teeth
Part II.
1. A 2. D 3. C 4. A 5. B 6. D 7.C 8. B 9. A 10. C
Part III.
1. c 2. f 3. a 4. d 5. e
Part IV.
l.for 2.so 3.in 4.which 5. where
6.is 7. built 8.have 9. house 10.what
Part V.
1. B 2. C 3. A 4. D 5. B
Part VI.
1. It was not until he came into the light that I recognized him.
2. That old typewriter is not worth repairing.
3. These pictures are the most beautiful (ones) (that) I have ever seen.
4. In spite of not speaking Dutch, Bob decided to settle in Amsterdam.
5. Glasses make him look older.
Notes:
Part 2.
• have no interest in sth (phr): không thích thú cái gì
• so that + s+ V: cốt để
• We must work hard so that we can pass the exam. Chúng ta phải học chăm cốt để có thể thi đỗ.
• used to do sth (phr): trước kia thường làm gì (bây giờ không còn làm nữa)
• Oliver used to go fishing and so did I = Oliver trước kia thường đi câu cá và tôi cũng vậy. (Bây giờ chúng
tôi không còn đi câu cá nữa)
Part 5:
• comparatively (adv): tương đối
• suffer from the pressures of academic work: chịu áp lực bài vở
• cautious approach: phương pháp thận trọng
• opportunity(n): cơ hội
Part 6:
• It was not until ... that s+v ... mãi đến khi...thì ...
• It is pointless to do sth = sth is not worth doing không đáng làm gì/làm gì chỉ vô ích mà thôi.
115
• have/get sth done = nhờ làm hộ
• It was not until he came into the light that I recognized him. Mãi đến khi cậu ta bước vào ánh đèn thì tôi
mới nhận ra cậu ta.
• That old typewriter is not worth repairing. Cái máy đánh chữ cũ đó không đáng sửa.

116
TRƯỜNG THCS CHẤT LƯỢNG CAO
PRACTICE TEST 1
Part I.
1. A 2. B 3. B 4. C 5. D
Part II.
1. A 2. A 3. A 4. B 5. C
Part III.
1. A 2. A 3. C 4. D 5. B
Part IV.
1. c 2. A 3. B 4. B 5. c
Part V.
1. C 2. B 3. B 4. C 5. C
6. B 7. C 8. C 9. D 10. C
11. B 12. D 13. A 14. B 15. A
16. A 17. D 18. C 19. C 20. D
21. B 22. B 23. C 24. B 25. C
Part VI.
1. C 2. A 3. C 4. D 5. D
Notes:
III. It’s + time + since + S + Ved: Đã ... trôi qua kể từ khi
It’s years since I enjoyed myself so much: Đã nhiều năm trôi qua kể từ khi tôi vui vẻ như thế = I haven’t
enjoyed myself so much for years.
sense of humour (phr): khiếu hài hước
make + sb do sth: bắt ai , khiến cho ai làm gì
His sense of humour always makes people laugh: Khiếu hài hước của cậu ta luôn làm mọi người cười.
IV. It is ... that ... Chính ... mà
It is English pronunciation that puzzles me most: Chính cách phát âm tiếng Anh gây rất nhiều khó khăn cho
tôi.
advise sb to do sth: khuyên ai làm gì
give up doing sth = stop doing sth: từ bỏ, thôi làm gì
discourage sb from doing sth: làm nản lòng, can ngăn ai làm gì > < encourage sb to do sth khích lệ ai làm gì
V. It is of great importance to do sth: Làm gì là điều rất quan trọng.
create/make a good impression on sb gây ấn tượng tốt đối với ai
pay attention to sb/ sth(phr): chú ý tới ai/ cái gì
neither = not either: cũng không ( dùng trong câu mang nghĩa phủ định)
Lan did not know the world. Hoa didn’t either = Neither did Hoa.
public transport (n): vận tải công cộng
VI. respond to sb/sth (phr.v): phản ứng với ai/ cái gì
turn out + to do sth (phr.v): hóa ra
result in sth = cause sth= bring about sth= lead to sth: gây ra, dẫn tới cái gì

PRACTICE TEST 2
Part I.
1. D 2. C 3. B 4. C 5. C
Part II.
1. A 2. D 3. D 4. C 5. D
117
Part III.
1. C 2. A 3. B 4. C 5. C
6. C 7. A 8. D 9. A 10. C
Part IV.
1. B 2. A 3. A 4. C 5. C
Part V.
1. B 2. D 3. C 4. A 5. B
Part VI.
1. A 2. B 3. A 4. C 5. B
Part VII.
1. A 2. C 3. C 4. D 5. D
Part VIII.
1. D 2. B 3. B 4. B 5. D
Part IX.
1. D 2. B 3. A 4. A 5. C
Notes:
V. take special care (phr): chăm sóc đặc biệt
for pleasure or for pastime (phr): để cho vui hoặc để tiêu khiển
protect sb from sth (phr): bảo vệ ai khỏi cái gì
hesitate to do sth: do dự làm gì
for the well-being of (phr): vì lợi ích, hạnh phúc, phúc lợi của
take care of = look after = rear (phr): chăm sóc, nuôi dưỡng
VI. find it adj + to do sth thấy làm gì đó ....
I found it wonderful to travel abroad: Tôi thấy đi du lịch nưóc ngoài thật tuyệt vời.
because of + n/ Ving = because + S + V
VII. tell a lie: nói dối > < tell the truth: nói thật
poor me! Tội nghiệp cho tôi !
mind doing sth: bận lòng làm gì
highly appreciate sb/sth/doing sth: đánh giá cao ai/cái gì/làm gì
XIII. dream of doing sth: mơ ước làm gì
congratulate sb on doing sth: chúc mừng ai đã làm gì
get a promotion: thăng tiến, lên chức
stand (v) = put up with: chịu đựng
promise to do sth: hứa làm gì
IX. pay sb/ sth a visit = pay a visit to sb/sth: thăm ai/ nơi nào
remember to do sth: nhớ sẽ phải làm gì (dùng để nhắc nhở ai)
It’s necessary for sb to do sth = Sb + need to do sth: Ai cần làm gì

PRACTICE TEST 3
Part I.
1. A 2. B 3. C 4. A 5. D
Part II.
1. B 2. D 3. D 4. B 5. C
Part III.
1. B 2. A 3. C 4. B 5. D
Part IV.
1. festivities 2. parades 3. costumes 4. dancing 5. fireworks
118
Part V.
1. A 2. B 3. C 4. C 5. C
Part VI.
1A 2. A 3. C 4. C 5. B
Part VII.
1. B 2. B 3. B 4. D 5. D
6. D 7. C 8. A 9. C 10. D
11. D 12. B 13. A 14. A 15. B
Part VIII.
1. D 2. B 3. B 4. A 5. A
Notes:
III. to be dependent on = to depend on: phụ thuộc vào
take place (phr): diễn ra, xảy ra
sense of peace and quiet (phr) cảm giác yên bình và tĩnh lặng
from time to time= occasionally, sometimes, at times, every now and then, once in a while: thỉnh thoảng
V. stay in contact with (phr): tiếp xúc với
tend to do sth = have the tendency of doing sth: có xu hướng làm gì
VII. in the way (phr): giữa đường
turn down (phr.v): vặn nhỏ (âm lượng)
to be obliged to sb for doing sth: Biết ơn ai đã làm gì
I am much obliged to you for helping us: Tôi rất biết ơn ông đã giúp chúng tôi.
Where there is a will, there is a way(idm): có chí thì nên.
VIII. There’s no doubt about it (phr) = It is certainly true: Chắc chắn
have a go (to do sth): thử làm việc gì
in my opinion (phr): theo ý kiến tôi
Congratulations!: Chúc mừng!

PRACTICE TEST 4
Part I.
1. C 2. D 3. E 4. A 5. B
Part II.
1. D 2. B 3. D 4. A 5. D
Part III.
1. A 2. B 3. C 4. B 5. B
6. B 7. D 8. A 9. C 10. A
Part IV.
1. too 2. much 3. down 4. are 5. lots
6. animals 7. on 8. grow 9. there 10. danger
Part V.
1. D 2. A 3. B 4. C 5. D
Part VI.
1. I ate in the hotel the first night but now I usually go out for my meals.
2. I will show you all of the photos I have taken when I get back.
3. Today I have been to an exciting art gallery with beautiful pictures.
4. I discovered a market with lots of stalls selling everything.
5. The restaurants are great and I can get a meal for less than £25.
Part VII.
119
1. geographical → geographically
2. from → in
3. knowing → known
4. influent → influence
5. much → many
6. larger → largest
7. French → France
8. Vietnam → Vietnamese
9. aspect → aspects
10. person → people
Part VIII.
1. round/ around 3. under 5. over
2. to 4. with
Notes:
I. an electric shock (phr): cú điện giật
cut off (phr): cắt (nguồn cung cấp)
in an attempt to do sth (phr) với nỗ lực làm gì
get connected to sb (phr): liên lạc với ai
on average(phr): tính trung bình
III. It is said that = People say that + S+V ... Người ta nói rằng ...
persuade sb to do sth: thuyết phục ai làm gì
middle-aged (adj): trung niên
IV. to be in danger: gặp nguy hiểm
V. once upon a time(phr): Ngày xửa ngày xưa
turn sth into sth (phr): biến cái gì thành cái gì
reward sb for doing sth: thưởng cho ai vì đã làm gì
do sb a favour: làm cho ai việc gì, giúp ai
fateful day (n): ngày định mệnh
take away the curse of sth (phr): mang đi lời nguyền về cái gì
weep aloud (phr); khóc to
in despair (phr): tuyệt vọng
VII. result from ... (phr.v) do ... mà ra
densely- populous (adj): đông dân
VIII. show sb round/around (phr.v): dẫn ai đi thăm nơi nào
live/come up to one’s expectations: đáp ứng mong đợi của ai.
to be brought under control (phr): được kiểm soát
provide sb with sth(phr): cung cấp cho ai cái gì
get over (phr.v): bình phục

PRACTICE TEST 5
Part I.
1. B 2. C 3. D
Part II.
1. D 2. A
Part III.
1. B 2. A 3. B 4. C 5. D
Part IV.
120
1. age 5. disabled 9. best sellers
2. voluntary service 6. playwright 10. speciality
3. comedies 7. constructed
4. hobby 8. poet
Part V.
1. A 2. A 3. D 4. C 5. C
Part VI.
1. B 2. B 3. C 4. D 5. C
6. C 7. C 8. D 9. D 10. B
Part VII.
1. D 2. B 3. C 4. D 5. C
Part VIII.
1. valuable 3. Scientifically 5. variety
2. simple 4. better
Part IX.
1. beginning 3. living 5. had
2. visited 4. (should) leave
Notes:
III. see enormous changes (phr): chứng kiến những thay đổi lớn
have access to (phr): truy cập
bring many benefits to (phr): mang lại nhiều lợi ích cho
in addition (phr): thêm vào đó
in conclusion (phr): sau cùng, cuối cùng
avoid damaging impacts to: tránh ảnh hưởng tai hại đối với
to be likely to do sth: có thể làm gì
She is very likely to come: Rất có thể cô ta sẽ đến.
IV. the same age as: cùng tuổi với
become involved in: tham gia vào
V. not only ... but also: không những ... mà còn (động từ hòa hợp với danh từ, đại từ đứng sau but also)
Not only his wife but also his parents are staying with him.
Không chỉ vợ mà cả bố mẹ đang ở cùng anh ấy.
a large number of + danh từ đếm được = rất nhiều (a large number of people, houses, cars, ...)
a large amount of + danh từ không đếm được = 1 số lượng lớn: (a large amount of work, money...)
VII. should have + PII (past participle) = lẽ ra nên (chỉ sự nuối tiếc)
I should have cooked more food: Lẽ ra tôi nên nấu thêm đồ ăn. (Thực tế tôi đã nấu không đủ đồ ăn)
VIII. Scientifically, there are no such so-called ghosts: Xét về mặt khoa học, không có cái được gọi là ma
như vậy.

PRACTICE TEST 6
Part I. Odd one out
1. B 2. A 3. C 4. A 5. B
Part II. Choose the word whose underlined part is pronounced differently from that of the others
1. C 2. B 3. D 4. A 5. D
Part III. Choose the best answer to fill in the blank.
1. C 2. A 3. A 4. B 5. D 6. B 7. A 8. C 9. C 10. D
11. C 12. A 13. D 14. D 15. A
Part IV. Identify the mistake in each sentence
121
1. B 2. B 3. D 4. C 5. B
Part V. Read the text. Some of the lines are incorrect. Find them and write the correct answers in the
space provided. (If a line is correct, put a tick √).
Line 1: live → lives Line 5: to → and, teach→teaches
Line 2: is → has, in → at On → In, listens → reads
Line 3: √ Line 7: go → goes
Line 4: √ Line 8: √
Part VI. Read the passage and decide which answer A, B, C or D best fits each space.
1. A 2. A 3. C 4. B 5. D
Part VII. Read the following text and choose the best answer for the questions below.
1. B 2. D 3. A 4. B 5. A
Part VIII. Choose the correct sentence among A, B, C or D which has the same meaning as the given
one.
1. A 2. C 3. A 4. B 5. C
Part IX. Complete the second sentence so that it has the similar meaning to the first one.
1. Nam is sitting to the right of Huong in the classroom at the moment.
2. There is a desk, an armchair, two computers and a printer in my father’s working room.
3. On the table is his computer.
4. There is a big super-market next to my house.
5. Because Jane is intelligent, kind-hearted and helpful, many people in my class love her.
Notes
Part IV: 1. children - số nhiều của child
2. excited + about: hào hứng/hứng thú về
3. Every Wednesday: thứ tư hàng tuần
4. will + V → will see
5. five bridges
Part VI: Cultural life: đời sống văn hóa
take place = happen = occur (v): diễn ra
typical (adj) + of: tiêu biểu, đặc trưng về
pray (v) + for: cầu nguyện cho
Part VII: be bound to V: chắc chắn sẽ làm gì
predator (n) = carnivore (n): động vật ăn thịt

PRACTICE TEST 7
Part I. Choose the word that is stressed differently from that of the other words.
1. D 2. A 3. C 4. D 5. B
Part II. Choose the word whose underlined part is pronounced differently from that of the others.
1. B 2. A 3. D 4. D 5. C
Part III. Choose the best answer to fill in the blank.
1. C 2. C 3. B 4. B 5. A 6. A 7. D 8. B 9. B 10. C
11. A 12. D 13. A 14. D 15. D 16. B 17. D 18. A 19. C 20. B
Part IV. Identify the mistake in each sentence.
1. A 2. C 3. B 4. C 5. C
Part V. Read the passage and decide which answer A, B, C or D best fits each space
1. A 2. C 3. B 4. D 5. A
Part VI. Read the passage and decide which answer A, B, C or D best fits each space.
1. C 2. A 3. D 4. A 5. D
Part VII. Choose the correct sentence among A, B, C or D which has the same meaning as the given
122
one
1. D 2. B 3. D 4. C 5. A
Part VIII. Complete the second sentence so that it has the similar meaning to the first one.
1. Children are keen on watching fireworks and dragon dance at Tet.
2. I think Vietnam is the most beautiful country in the world.
3. There is one table and two chairs in my study room.
4. Students should learn in groups to help each other.
3. I advise you to go travelling this summer to relax after a hard working time.
Part IX. Complete each of the following sentences with the words or phrases given
1. Is Mrs. Ngoc going to buy some toys for her children?
2. Many plants and animals are in danger because we are destroying them.
3. Ba usually plays tennis in the fall but sometimes he goes sailing.
4. Lan and Ly are going to the bookstore to buy some books now.
5. My sister has a bike and (she) cycles to work every day.
Notes
Part III. 4. pay for: trả tiền cho
6. hasty (adj): hấp tấp, vội vàng
draw a conclusion: rút ra kết luận, đưa ra kết luận
7. In the middle of all the panic: trong cơn hoảng loạn
10. desert (v): bỏ rơi
11. remind sb to do sth: nhắc ai làm gì
remind sb of/about sth/Ving: gợi cho ai nhớ về cái gì/việc gì; nhắc nhở ai về cái gì/việc gì
12. baptism (n): lễ rửa tội
christen (v): làm lễ rửa tội; đặt tên thánh (trong lễ rửa tội; đặt tên cho ai/cái gì rate of exchange: tỉ giá hối
đoái (tỉ giá trao đổi ngoại tệ)
19. He who pays the piper calls the tune. (Ai trả tiền thì người ấy có quyền.)
pay the piper (idiom): đứng ra chịu mọi phí tổn
Part IV: 1. are → is
2. on TV the phone the radio; in the newspaper/ magazine/book
3. restaurants
4. talkative (adj): hay nói
never trong câu phải sửa thành always
5. watching → to watch
Part V: burrow (n): cái hang
prickly bush: bụi cây gai
deter (v) sb/sth from sb/sth: ngăn ai/cái gì khỏi ai/cái gì
multiple litters of babies each year: nhiều lứa sinh con mỗi năm
Part VI: a prep school = a preparatory school: trường Trung học dân lập ở Mỹ
nominate (v): đề cử, đề nghị = request (v)
polio (n): bệnh viêm tủy xám
be/become sick with polio: bị/nhiễm bệnh viêm tủy xám
governor (n): thống đốc
studious (adj): hiếu học
talented (adj): có tài
rugged (adj): gian truân, vất vả, khổ hạnh
determined (adj): quả quyết, quyết tâm
Part VIII: 1. be keen on + Ving/N
2. advise sb to V: khuyên ai làm gì
123
PRACTICE TEST 8
Part I. Choose the word that is stressed differently from that of the other words.
1.A 2. A 3. A 4. D 5. A
Part II. Choose the word whose underlined part is pronounced differently from that of the others
1. C 2. B 3. D 4. B 5. B
Part III. Choose the best answer to fill in the blank.
1. C 2. D 3. B 4. B 5. A 6. D 7. A 8. B 9. C 10. C
11. B 12. A 13. D 14. C 15. C
Part IV. Identify the mistake in each sentence
1. C 2. B 3. A 4. B 5. A
Part V. Give the correct form of the words in the brackets in each of the following sentences.
1.hungry 2. useless 3. safely 4. better 5. application
Part VI. Complete the passage with the words given in the box
1. centuries 2. sculpture 3. finish 4. one
5. about 6. human 7. flying 8. portrait
Part VII. Choose the correct sentence among A, B, C or D which has the same meaning as the given
one
1. D 2. C 3. A 4. B 5. C 6. B 7. C
Part VIII. Complete the second sentence so that it has the similar meaning to the first one.
1. It is difficult to study English well.
2. Mr Tran drove carelessly.
3. The art club has nine members.
4. The girl is not strong enough to carry the bag.
5. I haven’t eaten this kind of food for 6 months.
Part IX. Complete each of the following sentences with the words or phrases given
1. Today Hai has to do the chores himself.
2. There are a lot of trees on the way to my school.
3. The teacher told me to clean the blackboard.
4. Could you tell me how to study English?
5. She is not old enough to be in my class.
Note
Part IV: 1. guiding → guided
a guided hike: cuộc đi bộ đường dài có hướng dẫn viên
2. don’t → didn’t
3. by → in
in + language: bằng tiếng gì
5. be/get used to + living: quen với việc gì
Part V: 1. hunger (n): tình trạng đói; (v): cảm thấy đói
hungry (adj): đói
hunger + for/after (Phr.v): khao khát, ước mong
2. use (n): sự sử dụng, cách dùng (v): sử dụng, dùng
useful (adj): có tác dụng, hữu ích
useless (adj): vô tác dụng, vô ích
3. safety (n): sự an toàn safe (adj): an toàn
unsafe (adj): không an toàn
4. So sánh hơn của good là better
5. apply (v): ứng dụng, áp dụng
application form: Đơn xin việc
124
Part VI: century (n): thế kỉ
sculpture (n): nghệ thuật điêu khắc, tác phẩm điêu khắc
sculpture (v): điêu khắc, chạm trổ
human body: cơ thể người portrait (n): chân dung

PRACTICE TEST 9
Part I. Odd one out.
1. C 2. D 3. A 4. D 5. D
Part II. Choose the word whose underlined part is pronounced differently from that of the others.
1. B 2. C 3. D 4. D 5. A
Part III. Choose the best answer to fill in the blank.
1. C 2. B 3. B 4. C 5. A 6. A 7. C 8. C 9. D 10. B
11. A 12. D 13. C 14. C 15. A 16. B 17. C 18. A 19. C 20. C
Part IV. Identify the mistake in each sentence
1. B 2. B 3. C 4. D 5. D
Part V. Give the correct form of the words in the brackets in each of the following sentences.
1. participants 2. wonderful 3. religious 4. mountainous 5. excitedly
Part VI. Read the passage and decide which answer A, B, C or D best fits each space.
1. D 2. C 3. D 4. D 5. B
Part VII. Choose the correct sentence among A, B, C or D which has the same meaning as the given
one
1. D 2. C 3. C 4. D 5. D
Part VIII. Complete the second sentence so that it has the similar meaning to the first one.
1. We haven’t seen Joanna for nearly two years.
2. I began studying English three years ago.
3. My backpack is different from yours.
4. Jane is the youngest in the group.
5. I am going to visit my grandparents next weekend.
Part IX. Complete each of the following sentences with the words or phrases given
1. It’s impossible to understand his theory.
2. I used to stay on a farm with my grandparents and my parents.
3. Has the manager come to the office yet?
4. Your writing is difficult to read.
5. We were extremely tired after the trip, so we slept on the bus on the way home.
Notes
Part III: 1. be adj + to V
3. stop sth from +Ving/N: ngăn cái gì khỏi việc gì/cái gì
4. finish + Ving
5. have time + to V: có thời gian làm gì
12. see somebody do sth: Chứng kiến ai làm việc gì đó (chứng kiến toàn bộ quá trình xảy ra sự việc).
See somebody doing: Chứng kiến ai đó đang làm gì, nhưng không phải chứng kiến cả quá trình xảy ra sự
việc.
15. so that + Clause (S + V): chỉ mục đích (để...)
in order + to V
16. courageous (adj): can đảm, dũng cảm
face (v): đối mặt với
adj + enough + to V: đủ ... để làm gì/ đủ ... có thể làm gì
Part IV: 1. on holiday: có ngày nghỉ
125
2. trạng từ tần suất + V: always go
3. dirty (adj): bẩn thỉu
dirt (n): bụi bẩn
4. D.buy sửa thành to buy
5. prefer + Ving to + Ving: thích việc gì hơn việc gì
D. play sửa thành playing
Part V: 1. participate (v) + in: tham gia vào
participant (n): người tham gia
participation (n) + in sth: sự tham gia, sự tham dự vào cái gì
2. wonder (n): kỳ quan, điều kỳ diệu
wonderful (adj): tuyệt vời, kỳ diệu
3. religion (n): tôn giáo
religious (adj): thuộc tôn giáo, sùng đạo
4. mountainous (adj): thuộc miền núi
5. excitedly (adv): một cách phấn khích
Part VI: rain heavily: mưa rào, mưa nặng hạt
make sth/sb + V
Ví dụ: Rain makes fruits and vegetables grow for us.
Part VII: pronunciation drills: những bài tặp phát âm
improve (v): cải thiện
Part VIII: different from + N: khác
Part IX: used + to V: thường làm gì trong quá khứ (thói quen trong quá khứ)

PRACTICE TEST 10
Part I. Odd one out.
1. C 2. A 3. A 4. D 5. C
Part II. Choose the word whose underlined part is pronounced differently from that of the others.
1. D 2. A 3. B 4. A 5. B
Part III. Match a word in column A with its definition in column B
1. C 2. A 3. F 4. B 5. E 6. D
Part IV. Choose the best answer to fill in the blank.
1. D 2. D 3. C 4. D 5. D 6. A 7. A 8. D 9. C 10. D
11. B 12. B 13. C 14. D
Part V. Give the correct form of the verbs in the brackets to complete the sentences
1. began
2. give
3. Does Mai go - goes
4. was written
5. won’t agree
Part VI. There is one mistake in each of the following sentences. Find and correct it.
1. and → with 2. at → after 3. happily → happy
4. understood → understand 5. bought for → bought
Part VII. Read the passage and decide which answer A, B, C or D best fits each space
1. D 2 D 3. A 4. C 5. C
Part VIII. Read the following text and choose the best answer to the questions below.
1. A 2. C 3. C 4. D 5. B
Part IX. Complete the second sentence so that it has the similar meaning to the first one.
1. How many dollars does it cost you to buy that souvenir?
126
2. How long have you had it?
3. Those modern paintings are not as expensive as these Dong Ho paintings.
4. Because of heavy rain, Jane didn’t go fishing yesterday.
5. I have never seen him before.
Part X. Complete each of the following sentences with the words or phrases given
1. The volunteers wash dirty clothes and classify them before giving them to the poor.
2. Do your children go camping every summer holiday?
3. My cousin will give me a book as a gift on my next birthday.
4. I enjoy playing sports because it’s good for my health.
5. He finds mountain climbing dangerous so he doesn’t take it up.
Note
Part III: Tandem bike: xe đạp hai yên
community radio station: đài phát thanh cho cộng đồng
a charity stall: phòng từ thiện
Part IV: 5. advise sb to do sth: khuyên ai làm gì
Part VI: 2. look after: chăm sóc
5. buy sb sth = buy sth for sb: mua cái gì cho ai
be able + to V: có thể
affect (v): ảnh hưỏng
cavity (n): lỗ răng sâu
special treatment: điều trị đặc biệt
Part VII: be able + to V: có thể
affect (v): ảnh hưởng
cavity (n): lỗ răng sâu
special treatment: điều trị đặc biệt
Part VIII: worker ant: kiến thợ
queen ant: kiến chúa
Part X: classify (v): phân loại
take up: bắt đầu

127
TRƯỜNG THCS LƯƠNG THẾ VINH
PRACTICE TEST 1
I. MULTIPLE CHOICE
Circle the odd one out.
1. D 2. A
Circle the word which has the underlined part pronounced differently from the others.
3. C 4. C 5. A
Circle the best option to complete the sentences.
6. B 7. D 8. D 9. B 10. C 11. D
12. B 13. D 14. A 15. A 16. C 17. B
There is one mistake in each of the following sentences. Find and correct it.
18. D → to 19. A → How much 20. C → homework
Read the passage and choose the correct answer A, B, C or D.
21. D 22. B 23. C 24. A 25. C
Read the following passage and choose the correct answer A, B, C or D.
26. A 27. D 28.C 29. B 30. D
II. WRITING
Rewrite the following sentences without changing the meaning, using the words given
31. He often takes part in outdoor activities after school.
32. Their school isn’t far from the stadium.
33. She is fond of learning English.
34. Do your classmates go to school by bicycle bike every day?
35. What is the length of the Red River?
Complete each of the following sentences with the words or phrases given
36. What kind of information are you looking for now?
37. Our family went back to our hometown to visit our grandparents last weekend.
38. Linh practices listening by watching English cartoons on TV every day.
Rearrange the following words and punctuation marks to complete the sentences.
39. Lan did not go to school yesterday because she was sick.
40. Students take part in different activities at recess.
Note
7. because 4- Clause (S + V + O)
because of + N/Ving
12. look after: chăm sóc, trông nom
18. kind to sb: tốt bụng với ai
31. take part in = participate in = join (in): tham gia vào
32. near + place: gần ... >< far from + place: xa ...
33. be fond of + N/Ving: thích cái gì/việc gì
34. ride their bicycles to school = go to school by bicycle/bike
36. look for: tìm kiếm
40. at recess = at break: vào giờ giải lao, giờ ra chơi

PRACTICE TEST 2
I. MULTIPLE CHOICE
Circle the odd one out.
1. D 2. C
Circle the word which has the underlined part pronounced differently from the others.
128
3. C 4. A 5. C
Circle the best option to complete the sentences.
6. D 7. C 8. D 9. A 10. B 11. A
12. A 13. C 14. B 15. C 16. A 17. A
There is one mistake in each of the following sentences. Find and correct it.
18. B → near/close to 19. D → costs 20. B → the shelves
Read the text and then write T (true) or F (false) beside the sentence.
21. F 22. T 23. F 24. T 25. T
Read the following passage and mark the letter A, B, C or D to indicate the correct word or phrase
that best fits each of the numbered blanks.
26. B 27. A 28. C 29. D 30. A
II. WRITING
Rewrite the following sentences without changing the meaning, using the words given
31. Where does she live?
32. Hai is fond of his new school.
33. His office isn’t near his house.
34. Peter’s family has five people.
35. Do you want a cup of tea?
Complete each of the following sentences with the words or phrases given
36. How does he feel about his new home?
37. She is happy because she lives with her uncle and aunt in Ha Noi.
38. My classmates are very nice and kind to me.
Rearrange the following words and punctuation marks to complete the sentences.
39. We often visit our relatives at the weekend.
40. My friends are sitting on turtle stones in the sun.
Notes
6 live with sb: Sống cùng ai đó
7. What's ... like? Như thế nào?
9 crowd (n): đám đông → crowded (adj): đông đúc
10. at + Number + street (at 86 Tran Phu Street ) on + street (on Tran Phu street).
15. take sb to somewhere: đưa ai đến nơi nào đó
32. be fond of + N/Ving = + enjoy/fancy + N/Ving: thích cái gì/việc gì + like + N/Ving/to V
35. would like + N/to V = want + N/to V

PRACTICE TEST 3
I. MULTIPLE CHOICE
Circle the word which has the underlined part pronounced differently from the others.
1. C 2. A 3. C
Circle the odd one out.
4. D 5. A
Circle the best option to complete the sentences.
6.B 7. C 8. A 9. B 10. D 11. C
12.C 13. B 14. C 15. C 16. C 17. B
There is one mistake in each of the following sentences. Find and correct it.
18. D→returns 19. B → spend 20. D → to
Read the text and then write T (true) or F (false) beside the sentence.
21. F 22. T 23. T 24. T 25. F
Read the following passage and choose the correct answer A, B, C or D.
129
26. A 27. B 28. C 29. B 30. C
II. WRITING
Rearrange the following words and punctuation marks to complete the sentences.
31. We learn about books and write essays in litterature.
32. At half past ten, the bell rings and all the student go into the yard.
Complete each of the following sentences with the words or phrases given
33. Would you mind if I turned on television?
34. We haven’t seen her since we were on holiday in Ha Long Bay.
35. I’m looking forward to seeing you soon.
Rewrite the following sentences without changing the meaning, using the words given.
36. Nam’s collection has over eight hundred stamps.
37. He drives carefully.
38. How luxurious the car is!
39. It is difficult to find an apartment in a big city. / not easy to find an apartment in a big city.
40. I spend about two hours each day doing my homework.
Note
6. come along: đến (tham gia)
8. Đồng tình khẳng định: So + auxiliary verb (do/does...) + S.
S + auxiliary verb (do/does...) /V ..., too
Đồng tình phủ định: Neither + auxiliary verb (do/does...) + S.
S + negative auxiliary verb (don’t/doesn’t... )+ V, either
20. from + place to + place: từ đâu tới...
33. Would you mind if + Clause S + V(simple past)
= Do you mind I + Clause S + V(simple present) → Xin phép ai làm gì
35. look forward to + N/Ving: mong đợi cái gì/việc gì
38. What + (a/an) + adj + N(s)! = How + adj + the + N(s) + be! -> câu cảm thán
39. It is + adj + to V.
40. It takes sb + time + to V: Ai đó mất bao lâu để làm gì.
S + spend + time + Ving: Ai dành thời gian bao lâu để làm gì.

PRACTICE TEST 4
I. MULTIPLE CHOICE
Circle the word which has the underlined part pronounced differently from the others.
1. B 2. C 3. C
Circle the odd one out.
4. A 5. A
Circle the best option to complete the sentences.
6. D 7. C 8. C 9. A 10. B 11. C 12. C 13. C 14. A 15. A
16.A 17. A
There is one mistake in each of the following sentences. Find and correct it.
18. C → from 19. A → in 20. B → won’t
Read the following passage and chose the correct answer A, B, C or D.
21.A 22. D 23. B 24. A 25. C 26. D
Read the passage and choose the correct answer A, B, C or D.
27.D 28. A 29. B 30. C
II. WRITING
Rewrite the following sentences without changing the meaning, using the words given.
31. We spent five minutes having a tea cup.
130
32. We take part in English festival every year.
33. My sister prefers listening to music to reading books.
34. Our summer vacation lasts for two months.
35. Our teacher walks to school.
Complete each of the following sentences with the words or phrases given
36. Kate meets her friends once a week to play sports on Sunday.
37. Books play an important role in our life.
38. It is better to go slowly than to go quickly and carelessly.
Rearrange the following words and punctuation marks to complete the sentences.
39. Fortunately, we took the right bus home in the end.
40. Do you always do morning exercises before breakfast?
Note
6. Hỏi khoảng cách:
How far is + place + from + place?
= How far is it from place to place? (Từ nơi nào tới nơi nào bao xa?)
19. be different + from: khác
27 - 30: means of communication: phương tiện giao tiếp
make a speech: phát biểu statesman (n): chính khách
coverage (n): tin tức, việc đưa tin về những sự kiện
motion picture: phim điện ảnh
action-packed drama: phim tâm lý hành động
31. It takes sb + time + to V → ai dành thời gian bao lâu để làm gì
S + spend + time + Ving → ai mất bao lâu để làm gì
32. participate in + N/Ving = take part in + N/Ving + join in + N/Ving:tham gia vào cái gì/tổ chức nào/việc

33. prefer + N/Ving to N/Ving: thích ai/cái gì/việc gì hơn ai/cái gì/việc gì
34. last (v): kéo dài (về mặt thời gian)
35. go to + place on foot = walk to + place

PRACTICE TEST 5
I. MULTIPLE CHOICE
Circle the word which has the underlined part pronounced differently from the others.
1.B 2. C 3. C
Circle the odd one out.
4. D 5. B
Circle the best option to complete the sentences.
6. B 7. D 8. B 9. C 10. C 11. D
12. C 13. B 14. B 15. A 16. D 17. D
There is one mistake in each of the following sentences. Find and correct it.
18. A → much 19. D → about 20. B → at
Read the following passage and choose the correct answer A, B, C or D.
21. F 22. T 23. T 24. F 25. F
Read the passage and choose the correct answer A, B, C or D.
26.D 27. A 28. A 29. B 30. A
II. WRITING
Rewrite the following sentences without changing the meaning, using the words given.
31. American students have more vacations than Vietnamese ones.
32. You need to wash your hands before meals.
131
You have to wash your hands before meals.
33. The weather is so good that they are going for a picnic.
34. How heavy was Mr Tan?
How much did Mr Tan weigh?
35. He takes care of sick people.
Complete each of the following sentences with the words or phrases given
36. We haven’t seen her since we were on holiday in Ha Long Bay.
37. Can you tell me how to get to the stadium?
38. We have music classes on Monday and Friday.
Rearrange the following words and punctuation marks to complete the sentences.
39. He is said to be the most intelligent student in the class.
40. His date of birth is on 2nd of October.
Notes
12. in the middle of + N: ở giữa
16. No, thanks → đáp lại lời mời khi muốn từ chối
17. not at all: không có gì
18. pollution (sự ô nhiễm là danh từ không đếm được nên dùng much + pollution.
19. tell sb about sth: kể cho ai về cái gì
20. at work: ở nơi làm việc
31. few/a few: vài, một ít → so sánh hơn là fewer: ít hơn
one (ones): là đại từ thay thế danh từ đã được nhắc tới trước đó.
Ví dụ: Vietnamese ones = Vietnamese students
American ones = American students
32. It is necessary for sb + to V
<=> S + need/have + to V: ai đó cần phải làm gì
33. such + (a/an) + adj + N that + Clause (S + V...)
<=> so + adj + that + Clause (S + V...): đến nỗi mà
34. Hỏi cân nặng: How heavy + be + S?
= How much + auxiliary verb (do/does...) + S + weigh?
= What + be (is) one’s weight/the weight of + N?
35. look after = take care of: chăm sóc

PRACTICE TEST 6
Mark the letter A,B,C or D to indicate the word whose underlined part differs from the other three in
pronunciation in each of the following questions.
1. C 2. B
Mark the letter A, B, C or D to indicate the word that doesn’t belong to the group.
3. D 4. D
Mark the letter A, B, C or D to indicate the underlined part that needs correction in each of the
following questions.
5.A 6. D 7. B
Mark the letter A, B, C or D to indicate the most suitable response to complete each of the following
exchanges.
8. A 9. C
Mark the letter A, B, C or D to indicate the correct answer to each of the following questions.
10. B 11. B 12. B 13. D 14. C 15. C 16. B 17. B 18. A 19. A
Mark the letter A, B, C or D to indicate the word(s) CLOSEST in meaning to the underlined word(s)
in each of the following questions.
132
20. B 21. B
Read the following passage and mark the letter A, B, C or D to indicate the correct word or phrase
that best fits each of the numbered blanks.
22. B 23. C 24. C 25. B 26. B
Read the following passage and mark the letter A, B, C or D to indicate the correct answer to each of
the questions.
27. B 28. A 29. A 30. B 31. B
Mark the letter A, B, C, or D to indicate the meaniful sentence written by using the word(s) or
phrase(s) given in each of the following questions.
32. D 33. B 34. D
Mark the letter A, B, C, or D to indicate the sentence that best combines each pair of sentences in the
following questions.
35. A 36. D 37. D
Mark the letter A, B, C, or D to indicate the sentence that is closest in meaning to each of the following
questions.
38.D 39.C 40.B
Notes
9. That was the least I could do: Câu trả lời lịch sự đáp lại lời cảm ơn/sự biết ơn của người khác đối với bạn.
You can say that again: Câu nói thẻ hiện rằng bạn hoàn toàn đồng ý với người khác về việc gì đó.
Thanks a million = Thanh you very much
18. Let’s + V: hãy làm gì
20. repair (v) = fix (v): sửa chữa
21. common = popular (adj): phổ biến
25. scenery: phong cảnh, cảnh đẹp
(the general appearance of the natural environment, especially when it is beautiful) scene: quang cảnh, cảnh
cụ thể (a view or picture of a place, event, or activity)
27 - 31: in danger: lâm nguy, gặp nguy hiểm
make an effort: nỗ lực
32. S + be + adj + to V
34. prefer + Ving to + Ving: thích việc gì hơn việc gì
35. adj + enough + to V: đủ ... để ...
too + adj + to V: quá ... không thể...
36. in order to V = so as + to V = to V → chỉ mục đích (để ...)
in order that + Clause (S + V...)
= so that + Clause (S 4- V...) → chỉ mục đích (để...)
40. S + find + N + adj: Ai đó nhận thấy cái gì như thế nào
S + think + N/S + be + adj: Ai đó nghĩ rằng cái gì như thế nào

PRACTICE TEST 7
Mark the letter A,B,C or D to indicate the word whose underlined part differs from the other three in
pronunciation in each of the following questions.
1. C 2. B
Mark the letter A, B, C or D to indicate the word that doesn’t belong to the group.
3. B 4. A
Mark the letter A, B, C or D to indicate the underlined part that needs correction in each of the
following questions.
5. B 6. C 7.A
Mark the letter A, B, C or D to indicate the most suitable response to complete each of the following
133
exchanges.
8. B 9. C
Mark the letter A, B, C or D to indicate the correct answer to each of the following questions.
10. C 11. A 12. D 13. C 14. C 15. A 16. A 17. B 18. B 19. D
Mark the letter A, B, C or D to indicate the word(s) CLOSEST in meaning to the underlined word(s)
in each of the following questions.
20. C 21. D
Read the following passage and mark the letter A, B, C or D to indicate the correct word or phrase
that best fits each of the numbered blanks.
22. C 23. B 24. D 25. B 26. A
Read the following passage and mark the letter A, B, C or D to indicate the correct answer to each of
the questions.
27. C 28. C 29. A 30. A 31. A
Mark the letter A, B, C, or D to indicate the meaniful sentence written by using the word(s) or
phrase(s) given in each of the following questions.
32. C 33. D 34. C
Mark the letter A, B, C, or D to indicate the sentence that best combines each pair of sentences in the
following questions.
35. A 36. D 37. D
Mark the letter A, B, C, or D to indicate the sentence that is closest in meaning to each of the following
questions.
Notes
5. look for: tìm kiếm → sửa of thành for
6. spend + time + Ving → sửa to study thành studying
7. enjoy + Ving → sửa to play thành playing
10. Mount Everest: Núi Everest
11. pay attention + to sth: chú ý vào cái gì
13. electron (n): điện tử
electric (adj): liên quan đến điện, chạy bằng điện (n): dòng điện
electronics (n): điện tử học electricity (n): điện lực
14. throw away: vứt đi
throw sth away: vứt cái gì đi
15. trước thời gian là giờ dùng giới từ “at”.
Ví dụ: at 8 o’clock
16. What sort of music... = what kind of music...? loại âm nhạc nào... ?
19. concerned (adj) + about sth/sb: quan tâm tới cái gì/ai
20. chemist’s = drugstore: hiệu thuốc
21. impolite (adj) = rude (adj): bất lịch sự, thô lỗ >< polite (adj): lịch sự
22. work for: làm việc cho (cơ quan/tổ chức/công ty nào đó)
25. until + time: đến tận khi
26. change (sth) to/into sth: đổi( từ cái này) sang cái khác
27-31: elder cousin: anh/chị họ
“elder” là dạng so sánh hơn của “old” và được dùng cho thành viên trong gia đình (elder brother/sister: anh
trai/chị gái)
fresh (adj): trong lành → fresh air: bầu không khí trong lành
32. in (one’s) free time: vào thời gian rảnh
35. Because he got up late, he missed the school bus.
→ “because” (bởi vì) dùng chỉ nguyên nhân
134
He got up late, so he missed the school bus.
→ “so” (do đó, bởi vậy, cho nên) dùng chỉ kết quả
39. be necessary + to V → cần phải làm gì
<=> S + need + to V: Ai cần làm gì

PRACTICE TEST 8
Mark the letter A,B,C or D to indicate the word whose underlined part differs from the other three in
pronunciation in each of the following questions.
1. B 2. D
Mark the letter A, B, C or D to indicate the word that doesn’t belong to the group.
3. A 4. D
Mark the letter A, B, C or D to indicate the underlined part that needs correction in each of the
following questions.
5. D 6. D 7. A
Mark the letter A, B, C or D to indicate the most suitable response to complete each of the following
exchanges.
8. C 9. D
Mark the letter A, B, C or D to indicate the correct answer to each of the following questions.
10. A 11. D 12. B 13. A 14. C 15. C 16. A 17. D 18. A 19. D
Mark the letter A, B, C or D to indicate the word(s) CLOSEST in meaning to the underlined word(s)
in each of the following questions.
20. D 21. C
Read the following passage and mark the letter A, B, C or D to indicate the correct word or phrase
that best fits each of the numbered blanks.
22. A 23. B 24. B 25. C 26. D
Read the following passage and mark the letter A, B, C or D to indicate the correct answer to each of
the questions.
27.C 28. C 29. A 30. C 31. D
Mark the letter A, B, C, or D to indicate the meaniful sentence written by using the word(s) or
phrase(s) given in each of the following questions.
32. C 33. D 34. C
Mark the letter A, B, C, or D to indicate the sentence that best combines each pair of sentences in the
following questions.
35. B 36. D 37. C
Mark the letter A, B, C, or D to indicate the sentence that is closest in meaning to each of the following
questions.
38. D 39. B 40. B
Notes
5. near = next to/close to: gần, cạnh
6. like + Ving/to V
9. “You are welcome.” Được dùng để đáp lại lời cảm ơn.
14. Mệnh đề sau so sánh hơn nhất được dùng thì hiện tại hoàn thành.
Ví dụ: This is the most interesting novel I’ve ever read.
17. keep sth + adj
Ví dụ: You should always keep your bedroom tidy. (Bạn nên luôn giữ cho phòng ngủ gọn gàng)
19. talk about sth: kể về cái gì
20. favourite/favorite = beloved (adj): được ưa chuông, được yêu mến
26. a busy place: một nơi tấp nập
135
a center for business and education: trung tâm thương mại và giáo dục
a center for religion: trung tâm tôn giáo
36. in spite of + N/Ving: mặc dù
37. neither + N/Pronoun nor + N/Pronoun + V... cả ... lẫn đều không ...
Neither Jane nor her husband likes coffee. (Cả Jane lẫn chồng cô ấy đều không thích cà phê.)
39. fast (adj), (adv): nhanh
40. be tired of + N/Ving: chán cái gì/việc gì

PRACTICE TEST 9
Mark the letter A, B, C or D to indicate the word that doesn’t belong to the group.
1. B 2. C
Mark the letter A, B, C or D to indicate the word whose underlined part differs from the other three in
pronunciation in each of the following questions.
3. B 4. C
Mark the letter A, B, C or D to indicate the underlined part that needs correction in each of the
following questions.
5. D 6. C 7. C
Mark the letter A, B, C or D to indicate the most suitable response to complete each of the following
exchanges.
8. C 9. A
Mark the letter A, B, C or D to indicate the correct answer to each of the following questions.
10. C 11. D 12. A 13. C 14. C 15. D 16. C 17. B 18. C 19. D
Mark the letter A, B, C or D to indicate the word(s) CLOSEST in meaning to the underlined word(s)
in each of the following questions.
20. B 21. C
Read the following passage and mark the letter A, B, C or D to indicate the correct word or phrase
that best fits each of the numbered blanks.
22. D 23. A 24. A 25. C 26. C
Read the following passage and mark the letter A, B, C or D to indicate the correct answer to each of
the questions.
27. C 28. C 29. D 30. A 31. B
Mark the letter A, B, C, or D to indicate the meaningful sentence written by using the word(s) or
phrase(s) given in each of the following questions.
32. A 33. B 34. C
Mark the letter A, B, C, or D to indicate the sentence that best combines each pair of sentences in the
following questions.
35. D 36. C 37. D
Mark the letter A, B, C, or D to indicate the sentence that is closest in meaning to each of the following
questions.
38. D 39. B 40. C
Notes
5. computer game: trò chơi trên máy tính
6. more and more old people: ngày càng nhiều người già
12. have a seat = sit down/ take a seat.
“Have a seat”: A polite directive to sit down. (Một cách nói lịch sự khi mời người khấc ngồi xuống)
17. leisure time = spare time = free time: thời gian rảnh
19. full of +Ns
136
crowded with + Ns → đầy cái gì
20. go to + place by bike = ride one’s bicycle/bike to + place = cycle (one’s bicycle/ bike) to + place
21. take up = start/begin: bắt đầu
36. not only + adj/N but also + adj/N: không những... mà còn...

PRACTICE TEST 10
Mark the letter A, B, C or D to indicate the word that doesn’t belong to the group.
1. D 2. D
Mark the letter A, B, C or D to indicate the word whose underlined part differs from the other three in
pronunciation in each of the following questions.
3. C 4. A
Mark the letter A, B, C or D to indicate the underlined part that needs correction in each of the
following questions.
5.C 6. B 7. B
Mark the letter A, B, C or D to indicate the most suitable response to complete each of the following
exchanges.
8. B 9. D
Mark the letter A, B, C or D to indicate the correct answer to each of the following questions.
10. C 11. A 12. B 13. C 14. C 15. C
16. B 17. D 18. C 19. B
Mark the letter A, B, C or D to indicate the word(s) CLOSEST in meaning to the underlined word(s)
in each of the following questions.
20. B 21. C
Read the following passage and mark the letter A, B, C or D to indicate the correct word or phrase
that best fits each of the numbered blanks.
22. A 23. B 24. C 25. B 26. A
Read the following passage and mark the letter A, B, C or D to indicate the correct answer to each of
the questions.
27. A 28. C 29. B 30. B 31. B
Mark the letter A, B, C, or D to indicate the meaniful sentence written by using the word(s) or
phrase(s) given in each of the following questions.
32. C 33. B 34. C
Mark the letter A, B, C, or D to indicate the sentence that best combines each pair of sentences in the
following questions.
35. A 36. B 37. C
Mark the letter A, B, C, or D to indicate the sentence that is closest in meaning to each of the following
questions.
Notes
5. describe sth: miêu tả cái gì
7. “the” là mạo từ xác định, “the” sửa thành “a”
13. volunteer teacher: giáo viên tình nguyện
17. on the phone: nói chuyện điện thoại
chat with/to sb: tán gẫu với ai
20. turn off = shut down: tắt đi
21. go on = continue: tiếp tục
22-26: rich class: tầng lớp giàu có
33. decide to V: quyết định làm gì
34. practice + Ving: luyện làm gì
137
35. so + adj that + Clause (S + V): ... đến nỗi mà ...
38. be/get used to + Ving/N = be accustomed to + Ving/N
39. Câu điều kiện loại 2: Không có thật ở hiện tại. Sau Subject trong mệnh đề if dùng “were” cho tất cả các
ngôi.
Ví dụ: If the house were not large, they could paint it in a week.
40. The sai khiến (Causative): have sth done
She had her bicycle repaired last week.

138

You might also like